ML14163A386

From kanterella
Jump to navigation Jump to search

Final Written Examination with Answer Key (405-5 Format) (Folder 3)
ML14163A386
Person / Time
Site: Oyster Creek
Issue date: 05/13/2014
From: Todd Fish
Operations Branch I
To: Ridosh J
Exelon Nuclear Generation Corp
Shared Package
ML13333A261 List:
References
TAC U01893
Download: ML14163A386 (276)


Text

EXAMINATION ANSWER KEY ILT 13~1 NRC Written Exam (RO) 1 10: 13*1 NRO 01 Points: 1.00 The plant was at rated power when a fuel failure occurs at 1:00pm.

At 1:10 pm the following indications were observed in the Control Room:

  • CHARMS channel 1 indicates 52 R/hr
  • CHARMS channel 2 indicates 48 R/hr
  • Off-Gas Radiation Monitors indicate 1300 rnr/hr Which of the following automatic actuation signals have already occurred as of 1:10 pm?
1. Reactor Building Ventilation Isolation
2. Standby Gas Treatment System initiation
3. Off-Gas Isolation
4. Primary Containment Vent & Purge Valve Isolation A. 1, 2, 3 and 4 B. 1 and 2 ONLY C. 3 and 4 ONLY D. 4 ONLY Answer: 0 Answer Explanation Knowledge and Ability Reference Information Importance Rating K&A RO SRO 212000 RPS K1.05
  • Knowledge of the physical connections and/or cause- effect relationships between REACTOR 3.3 3.6 PROTECTION SYSTEM and the following: Process radiation monitoring system Level RO I Tier I 2 I Group I 1 General RAP-10F1c RAP 10F4k References OCS OPS ILT 13-1 EXAM Page: 1 of 201 13 May 2014

EXAMINATION ANSWER KEY ILT 13*1 NRC Written Exam (RO)

D is Correct. If either Channel I or II of CHARMS sense 45 Rlhr or greater, with no time delay, torus/drywell vent & purge valve isolation occurs. Off-gas isolation occurs when off-gas rad monitors sense 1000 mr/hr after a 15 minute time delay and only 10 minutes have elapsed.

A is Incorrect. RB ventilation isolation and SGTS initiation Explanation setpoint has not been exceeded and therefore has not occurred.

B is Incorrect. Reactor Building exhaust monitors must sense 9

+I* 1 mr/hr or a lo-lo RPV water level or high drywell pressure must occur in order for these to occur.

C is Incorrect. Off-gas isolation has not occurred since 15 minutes has not yet elapsed.

Lesson 2621.828.0.033A, Plant Radiation Monitoring Systems Plan 00819 State any automatic actions initiated by the ARM system.

Learning State which monitors provide these actions and the setpoints.

Objective/ 00837 List or identify the auto isolation setpoints for process RAD Monitor system: CHARMS, Sewage Rad monitor, RB Ventilation Exhaust, Off-Gas Air Ejector Radiation Monitors, and Containment Airborne Particulate and Gaseous Radiation Monitoring System (CAPGRMS).

References ILT: None LORT: Open Provided Question Source (New, Modified, Bank) Bank Previous 2 NRC Exams (ILT Only) No Memory or Comprehension Fundamental 1 :I Cognitive or Analysis Knowledge Level NUREG 1021 Appendix B: interlocks, setpoints, or system (singular) response 55.41b 7 55.43b 10CFR55 Design, components, and functions of control and safety systems, Content including instrumentation, signals, interlocks, failure modes, and automatic and manual features.

Justification for LORT questions with KIA values*< 3.0 J N/A Time to Complete: 1-2 minutes Point Value: 1 System I D No.: 212000 PRA: No Safety Function(s): 7 IZI ILT OCSOPS ILT 13-1 EXAM Page: 2 of 201 13 May 2014

EXAMINATION ANSWER KEY ILT 13~1 NRC Written Exam (RO)

I Category(s) (LORT Only): I N/A lo LORT OCS OPS ILT 13-1 EXAM Page: 3of201 13May2014

EXAMINATION ANSWER KEY ILT 13~1 NRC Written Exam (RO) 2 ID: 13*1 NRO 02 Points: 1.00 Reactor power is 50% while starting up after an outage. Control rod withdrawals are in progress.

The following annunciator then alarmed:

  • IRM HI-HI/INOP The Operator reports that IRM 15 indicates upscale. While investigating the alarm, the following annunciator alarmed:
  • APRM DNSCL The Operator reports that APRM 5 indicates downscale.

Which of the following states the correct Operator action(s) to continue the startup?

A Bypass IRM 15 ONLY.

B. Bypass APRM 5 ONLY.

C. Bypass IRM 15 AND reset the 1/2 scram.

D. Bypass APRM 5 AND reset the 1/2 scram.

Answer: D Answer Explanation Knowledge and Ability Reference Information Importance Rating K&A RO SRO 215005 APRM I LPRM K1.02 - Knowledge of the physical connections and/or cause- effect relationships between AVERAGE POWER 3.7 3.7 RANGE MONITOR/LOCAL POWER RANGE MONITOR SYSTEM and the following: IRM Level RO I Tier I 2 I GroupJ 1 General RAP-H7a GE 237E566 References RAP-G4f OCS OPS ILT 13-1 EXAM Page: 4 of 201 13 May 2014

EXAMINATION ANSWER KEY ILT 13~1 NRC Written Exam (RO)

Dis Correct. The mode switch has been placed in Run.

Generally, this bypasses aiiiRM trips. But when an APRM goes downscale while in RUN, with its companion IRM upscale, then a 1/2 scram occurs. Both APRM 5 downscale and its companion IRM 15 upscale have occurred. To correct the situation, APRM 5 must be bypassed and then the 1/2 scram reset.

Explanation A & B are Incorrect but plausible. If IRM 15 or APRM 5 were bypassed only, then the 1/2 scram would remain.

Cis Incorrect but plausible. If IRM 15 was bypassed, the 1/2 scram can be reset. But this will still leave a rodblock from the downscale ARRM 5.

Lesson 2621.828.0.0029, Nuclear Instrumentation Plan NIS-1 0442: Given the system logic/electrical drawings, describe the system bypass or reset logic and return the system to normal Learning or standby condition.

Objective/

References ILT: None LORT: Open Provided Question Source (New, Modified, Bank) Bank Previous 2 NRC Exams (ILT Only) No Memory or Comprehension Fundamental 1:I Cognitive or Analysis Knowledge Level NUREG 1021 Appendix B: Interlocks, setpoints, or system (singular) response 55.41b 7 55.43b 10CFR55 Design, components, and functions of control and safety systems, Content including instrumentation, signals, interlocks, failure modes, and automatic and manual features.

Justification for LORT questions with KIA values< 3.0 I NIA Time to Complete: 1-2 minutes Point Value: 1 System 10 No.: 215005 PRA: No Safety Function(s): 7 ~ ILT Category(s) (LORT Only): N/A 0 LORT OCS OPS ILT 13-1 EXAM Page: 5 of 201 13 May 2014

EXAMINATION ANSWER KEY ILT 13~1 NRC Written Exam (RO) 3 10: 13*1 NRO 03 Points: 1.00 The plant is at rated power. Which of the following lists ALL of the power supplies available to power the listed pump motors following a plant scram? (Assume NO operator action)

Containment Spray Pump 51 C Core Spray Pump NZ01 A A. S/U Transformer SB S/U Transformer SB EDG 2 EDG 2 B. S/U Transformer SA SBO Transformer EDG 1 S/U Transformer SB FACT EDG 1 C. S/U Transformer SA S/U Transformer SA EDG 1 EDG 2 D. S/U Transformer SB S/U Transformer SA EDG 2 EDG 1 Answer: D Answer Explanation Knowledge and Ability Reference Information Importance Rating K&A RO SRO 209001 LPCS K2.01

  • Knowledge of electrical power supplies to the 3.0 3.1 following: Pump power Level RO I Tier I 2 I Group I 1 General BR 3000 References BR 3002 OCS OPS ILT 13-1 EXAM Page: 6 of 201 13 May 2014

EXAMINATION ANSWER KEY ILT 13-1 NRC Written Exam (RO)

D is Correct. Containment Spray Pump 51 C is powered from 480 VAC USS 1B2, which is powered from 4160 VAC Bus 1D. Core Spray Pump NZ01 A is powered from 4160 VAC Bus 1C (which examines the applicant's knowledge of KIA 209001 K2.01).

At power, Bus 1Dis powered from the auxiliary transformer.

When the plant trips, Bus 1D will automatically be powered from the Startup Transformer SB, or Bank 6. If bank 6 should fail, EDG Explanation 2 will auto start and load onto Bus 1D and 1B2. Bus 1C will automatically be powered from the Startup Transformer SA, or Bank 5. If bank 5 should fail, EDG 1 will auto start and load onto Bus 1C.

All distractors are Incorrect but plausible. The FRCT (Forked River Combustion Turbine) and the Station Blackout transformer (SBO) can be started under loss of power events, but loading it onto station loads require manual actions.

Lesson 2621.828.0.0009, Containment Spray/ESW Systems Plan CNS-10453: Explain or describe how this system is interrelated with other plant systems.

Learning CNS-1 0445: Given a set of system indications or data, evaluate Objective/ and interpret them to determine limits, trends and system status.

References ILT: None LORT: Open Provided Question Source (New, Modified, Bank) Bank Previous 2 NRC Exams (ILT Only) No Memory or Comprehension Fundamental 1:F Cognitive or Analysis Knowledge Level NUREG 1021 Appendix B: Facts 55.41b 7 55.43b 10CFR55 Design, components, and functions of control and safety systems, Content including instrumentation, signals, interlocks, failure modes, and automatic and manual features.

Justification for LORT questions with KIA values< 3.0 I N/A Time to Complete: 1*2 minutes Point Value: 1 System ID No.: 209001 PRA: No Safety Function(s): 2&4 1ZJ ILT Category(s) (LORT Only): N/A 0 LORT OCSOPS ILT 13-1 EXAM Page: 7 of 201 13 May 2014

EXAMINATION ANSWER KEY ILT 13~1 NRC Written Exam (RO) 4 ID: 13*1 NRO 04 Points: 1.00 The plant is at rated power with EDG-2 out of service.

lAW Tech Spec 3.7, Auxiliary Electrical Power, a loss of which of the following components will require the plant to be placed in the COLD SHUTDOWN CONDITION?

A. Containment Spray Pump 1-2 B. Core Spray Main Pump NZ01 C C. Core Spray Booster Pump NZ03B D. Emergency Service Water Pump 1-3 Answer: A Answer Explanation Knowledge and Ability Reference Information Importance Rating K&A RO SAO 264000 EDGs 2.2.40- Equipment Control: Ability to apply technical 3.4 4.7 specifications for a system.

Level RO I Tier I 2 I Group I 1 General TS 3.7.C.2.c References A is Correct. lAW Tech Spec 3.7.C.2.c, with one diesel generator out of service, none of the engineered safety equipment fed by the operational diesel may be out of service or the plant must be placed in the COLD SHUTDOWN CONDITION. Containment Spray Explanation Pump 1-2 is fed from EDG-1 via USS-1A2.

All distractors are Incorrect but plausible since they are all engineered safety equipment, however they are all powered via the out of service diesel.

OCSOPS ILT 13-1 EXAM Page: 8 of 201 13 May 2014

EXAMINATION ANSWER KEY ILT 13~1 NRC Written Exam (RO)

Lesson 2621.828.0.0013, Emergency Diesel Generators Plan EDG-1 0451, Referencing plant Technical Specifications (* from Learning memory for Initial Candidates) and given a set of plant Objective/ conditions, determine, as applicable, the:

a) Definitions*

b) Safety Limits and Bases*

c) Limiting Safety System Settings and Bases*

d) Limiting Conditions for Operation and Applicability e) LCO Action Requirements (SRO ONLY) f) Surveillance Requirements (SRO ONLY) g) Design Features, Containment, Auxiliary Equipment, Administrative Controls, and Appendix B Environmental Technical Specifications (SRO ONLY) h) Bases for Surveillance Requirements, Design Features, Containment, Auxiliary Equipment, Administrative Controls, and Appendix B Environmental Technical Specifications. (SRO ONLY)*

References ILT: None LORT: Open Provided Question Source (New, Modified, Bank) New Previous 2 NRC Exams (ILT Only) No Memory or Comprehension Fundamental 2:DR Cognitive or Analysis Knowledge Level NUREG 1021 Appendix B: Describing or recognizing Relationships 55.41b 10 55.43b 10CFR55 Content Administrative, normal, abnormal, and emergency operating procedures for the facility.

Justification for LORT questions with KIA values< 3.0 I NIA Time to Complete: 1-2 minutes Point Value: 1 System I D No.: 264000 PRA: No Safety Function(s): 6 181

-ILT Category(s) (LORT Only): N/A 0 LORT OCS OPS ILT 13-1 EXAM Page: 9 of 201 13 May 2014

EXAMINATION ANSWER KEY ILT 13~1 NRC Written Exam (RO) 5 ID: 13*1 NRO 05 Points: 1.00 The plant was at rated power when LPRM 28-33A (input to APRM 1) failed resulting in APRM 1 indicating 87.5%. All other LPRMs and APRMs indicate normally.

Which of the following other indications are correct? Assume that the APRM Gain is 1.000.

A. APRM 1 DNSCL OR INOP light will be ON (on Panel 4F) AND a rodblock is present.

B. LPRM 28-33A local analog meter on Panel 4F full core display will have a red back-light.

C. LPRM 28-33A amber light on Panel 4F *full core display will be OFF AND a rodblock is present.

D. LPRM 28-33A local analog meter on Panel 4F full core display indicates downscale.

Answer: D Answer Explanation Knowledge and Ability Referenc!Jnformation Importance Rating K&A RO SRO 215005 APRM I LPRM K3.05

  • Knowledge of the effect that a loss or malfunction of the AVERAGE POWER RANGE MONITOR/LOCAL 3.8 3.8 POWER RANGE MONITOR SYSTEM will have on following: Reactor power indication Level RO I Tier I 2 J GroupJ 1 General RAP-G4f, G7f References OCSOPS ILT 13-1 EXAM Page: 10 of 201 13 May 2014

EXAMINATION ANSWER KEY ILT 13~1 NRC Written Exam (RO)

D is Correct. APRM 1 will read 100% at full power and will read 87.5 when 1 LPRM goes downscale (700/8) [given the gain is 1.000]. Therefore, the LPRM has failed downscale. When the LPRM fails downscale, several things happen: the associated APRM produces a rodblock; the amber light (on full core display) goes ON; the APRM reading goes down; and, the local reactor power meter of the full core display (individual LPRM readings) will go downscale. Therefore, there will be a rod block and the local meter will read downscale.

Explanation A is Incorrect but plausible. The APRM will neither be INOP nor downscale.

B is Incorrect but plausible. There is no red backlighting for the associated LPRM, the red backlighting is on control rod indications.

C is Incorrect but plausible. Since the local reactor power indication for the LPRM failed downscale, its amber light will be ON, not OFF.

Lesson 2621.828.0.0029, Nuclear Instrumentation Plan Learning NIS-1 0444, Describe the interlock signals and setpoints for the Objective/ affected system components and expected system response including power loss or failed components.

References ILT: None LORT: Open Provided Question Source (New, Modified, Bank) Bank Previous 2 NRC Exams (ILT Only) Yes Memory or Comprehension Fundamental 3:SPK Cognitive or Analysis Knowledge Level NUREG 1021 Appendix B: Solve a Problem using Knowledge and its meaning 55.41b 7 55.43b 10CFR55 Design, components, and functions of control and safety systems, Content including instrumentation, signals, interlocks, failure modes, and automatic and manual features.

Justification for LORT questions with KIA values< 3.0 I N/A OCS OPS ILT 13-1 EXAM Page: 11 of 201 13 May 2014

EXAMINATION ANSWER KEY ILT 13-1 NRC Written Exam (RO)

Time to Complete: 1-2 minutes Point Value: 1 System 10 No.: 215005 PRA: No Safety Function(s): 7 IZI ILT Category(s) (LORT Only): N/A 0 LORT OCS OPS ILT 13-1 EXAM Page: 12 of 201 13 May 2014

EXAMINATION ANSWER KEY ILT 13~1 NRC Written Exam (RO) 6 ID: 13*1 NRO 06 Points: 1.00 If IRM 11 loses power while in Range 6 during a Reactor Startup, IRM 11 at Panel 4F will indicate ...

A. upscale at 40/125.

B. downscale at 0.

C. have no indication (black out).

D. the same value as when it failed.

Answer: B Answer Explanation Knowledge and Ability Reference Information Importance Rating K&A RO SRO 2150031RM K3.04

  • Knowledge of the effect that a loss or malfunction 3.6 3.6 of the INTERMEDIATE RANGE MONITOR (IRM) SYSTEM will have on following: Reactor power indication Level RO I Tier I 2 I Group I 1 General RAP-9XF8d References RAP*9XF7d 8 is Correct. The IRM detectors are gas filled ion chambers. The DC voltage to the detector is used to attract the ion pairs which are produced from ionization by the incoming charged particle. If this applied voltage fails to zero, there is no longer an applied force to cause the ion pairs to collect at the electrodes. Since the Explanation number of ion pairs collected is proportional to the detector output, the detector output will fail downscale since the collected ion pairs fails to zero.

All distractors are Incorrect but plausible if the applicant does not recall or understand how the detector works.

OCS OPS ILT 13-1 EXAM Page: 13 of 201 13 May 2014

EXAMINATION ANSWER KEY ILT 13~1 NRC Written Exam (RO)

Lesson 2621.828.0.0029, Nuclear Instrumentation Plan NIS-1 0444, Describe the interlock signals and setpoints for the Learning affected system components and expected system response Objective/ including power loss or failed components.

References ILT: None LORT: Open Provided Question Source (New, Modified, Bank) New Previous 2 NRC Exams (ILT Only) No Memory or Comprehension Fundamental 2:DR Cognitive or Analysis Knowledge Level NUREG 1021 Appendix B: Describing or recognizing Relationships 55.41b 7 55.43b 10CFR55 Design, components, and functions of control and safety systems, Content including instrumentation, signals, interlocks, failure modes, and automatic and manual features.

Justification for LORT questions with KIA values< 3.0 I NIA Time to Complete: 1-2 minutes Point Value: 1 System ID No.: 215003

- PRA: No Safety Function(s): 7 ~ ILT Category(s) (LORT Only): N/A 0 LORT OCS OPS ILT 13-1 EXAM Page: 14 of 201 13 May 2014

EXAMINATION ANSWER KEY ILT 13~1 NRC Written Exam (RO) 7 10: 13*1 NRO 07 Points: 1.00 The plant was at rated power when the following annunciator came into alarm:

  • N2 COMPR FAIL Which of the following describes the N2 Line Pressure setpoint that results in this alarm AND as a result, which air system is now supplying Drywell air loads?

N2 Line Pressure Supplying Drywell Loads A. < 75 psig Service Air B. < 65 psig Instrument Air

c. < 65 psig Service Air D. < 75 psig Instrument Air Answer: B Answer Explanation Knowledge and Ability Reference Information Importance Rating K&A RO SRO 300000 Instrument Air K4.02 *Knowledge of (INSTRUMENT AIR SYSTEM) design 3.0 3.0 feature(s) and or interlocks which provide for the following: Cross-over to other air systems Level RO I Tier I 2 I Group I 1 General RAP-C3g References OCS OPS ILT 13-1 EXAM Page: 15 of 201 13 May 2014

EXAMINATION ANSWER KEY IL T 13~1 NRC Written Exam (RO)

B is Correct. The setpoint for the N2 COMPR FAIL is< 65 psig.

This alarm also automatically opens V-6-1 000, Drywell N2 Supply Shutoff Valve to North Air (SV1 ), and shuts V-6-1 004, Drywell N2 Supply Shutoff Valve To North Air (SV2). This aligns Instrument Air to supply the Drywell N2 Air system loads through V-6-395, Instrument Air Isolation to the Drywell.

A is Incorrect but plausible. The setpoint for the N2 COMPR FAIL Explanation alarm is < 65#, not < 75#. The Drywell Air loads will be supplied by Instrument Air, not Service Air.

C is Incorrect but plausible. The Drywell Air loads will be supplied by Instrument Air, not Service Air.

D is Incorrect but plausible. The setpoint for the N2 COMPR FAIL alarm is < 65#, not < 75#.

Lesson 2621.828.0.030, Nuclear Steam Supply System Plan NSS-1029, Given a drawing of the NSSS, trace the flowpaths and locate the major components associated with the system, and Learning explain its operation within the system.

Objective/

References ILT: None LORT: Open Provided Question Source (New, Modified, Bank) Bank Previous 2 NRC Exams (ILT Only) No Memory or Comprehension Fundamental 2:RI Cognitive or Analysis Knowledge Level NUREG 1021 Appendix B: Recognizing interaction between systems (plural), including consequences and implications 10CFR55 55.41b 7 55.43b I

Design, components, and functions of control and safety systems, Content including instrumentation, signals, interlocks, failure modes, and automatic and manual features.

Justification for LORT questions with KIA values< 3.0 I NIA Time to Complete: 1-2 minutes Point Value: 1 System 10 No.: 300000 PRA: No Safety Function(s): 8 ~ ILT Category(s) (LORT Only): N/A 0 LORT OCS OPS ILT 13-1 EXAM Page: 16 of 201 13 May 2014

EXAMINATION ANSWER KEY ILT 13~1 NRC Written Exam (RO) 8 10: 13*1 NRO 08 Points: 1.00 The plant was at rated power when an event occurred. The Operator reports that RPV water level is 80" and rising slowly.

One minute later, the following annunciator alarmed:

  • COND B FLOW HI POSSIBLE RUPTURE With NO Operator action, which of the following states the final positions of the Isolation Condenser System Valves (Steam and Condensate Return) and associated Vent Valves?

A. System A Valves open System B Valves closed Vent Valves open Vent Valves closed B. System A Valves open System B Valves closed Vent Valves closed Vent Valves closed

c. System A Valves open System B Valves open Vent Valves open Vent Valves closed D. System A Valves open System B Valves closed Vent Valves closed Vent Valves open Answer: B Answer Explanation Knowledge and Ability Reference Information Importance Rating K&A RO SRO 207000 Isolation (Emergency) Condenser A3.03 - Ability to monitor automatic operations of the 3.5 3.7 ISOLATION (EMERGENCY) CONDENSER including:

Reactor water level: BWR-2,3 Level RO I Tier I 2 I Group J 1 General RAP-C3b EMG-SP1 307 References RAP-C1a OCS OPS ILT 13-1 EXAM Page: 17 of 201 13 May 2014

EXAMINATION ANSWER KEY ILT 13*1 NRC Written Exam (RO)

The plant was at rated power when an event resulted in an RPV water level of 80 As water level lowers from its normal155 .. ,

11 both Isolation Condensers will auto initiate at 90 11 (1 condensate valve in each loop goes open to initiate the condensers, and the normally open vent valves go closed). So far, all System valves are open and all vent valves go closed.

B is Correct. When the Possible Rupture B comes in, this will close all System valves in System B only, and the vent valves Explanation remain closed. Thus, System A valves are open with their associated vent valves closed. System B valves are closed with the vent valves closed.

The other distractors provide various positions. If the applicant does not know the initiation signals and their impact on the system/vent valves, or does not know how the Possible Rupture annunciator impacts the Isolation Condensers, than all other answers are plausible.

Lesson 2621.828.0.0023, Isolation Condensers Plan ICS-2030, Describe the Isolation Condenser design feature(s)

Learning and/or interlocks (including signals and setpoints) which provide Objective/ for the following: a. Automatic system initiation, b. Automatic system isolation References ILT: None LORT: Open Provided Question Source (New, Modified, Bank) Bank Previous 2 NRC Exams (ILT Only) No Memory or Comprehension Fundamental 3:PEO Cognitive or Analysis Knowledge Level NUREG 1021 Appendix B: Predict an Event or Qutcome 55.41b 7 55.43b 10CFR55 Design, components, and functions of control and safety systems, Content including instrumentation, signals, interlocks, failure modes, and automatic and manual features.

Justification for LORT questions with KIA values< 3.0 I N/A Time to Complete: 1-2 minutes Point Value: 1 System ID No.: 207000 PRA: No Safety Function(s): 4 1ZJ ILT OCS OPS ILT 13-1 EXAM Page: 18 of 201 13 May 2014

EXAMINATION ANSWER KEY ILT 13~1 NRC Written Exam (RO)

I Category(s) (LORT Only): I N/A lo LORT OCS OPS ILT 13-1 EXAM Page: 19 of 201 13 May 2014

EXAMINATION ANSWER KEY ILT 13~1 NRC Written Exam (RO) 9 ID: 13-1 NRO 09 Points: 1.00 The plant is at rated power with a normal 125 VDC lineup when the following annunciator came into alarm:

  • U-6-f, STATION BAT/CHG- C- C BAT H2 HI-HI It is confirmed that ALL 'C' Battery Room ventilation is lost and cannot be restarted.

Which of the following states (1) the U-6-f alarm setpoint AND (2) the action required by RAP-U6f due to a complete loss of ventilation?

NOTE: LEL- Lower Explosive Limit A. (1) 0.8% (20% of LEL)

(2) Secure from charging the 'C' battery, Install a portable fan, and monitor the 'C' Battery Room H2 concentration at least once every 4 hrs B. (1) 1.6% (40% of LEL)

(2) Secure from charging the 'C' battery, Install a portable fan, and monitor the 'C' Battery Room H2 concentration at least once every 4 hrs C. (1) 1.6% (40% of LEL)

(2) Open the 'C' Battery Room fire door, install a portable fan, and post a continuous fire watch at the 'C' Battery Room D. (1) 0.8% (20% of LEL)

(2) Open the 'C' Battery Room fire door, install a portable fan, and post a continuous fire watch at the 'C' Battery Room Answer: C Answer Explanation Knowledge and Ability Reference Information Importance Rating K&A RO SRO 263000 DC Electrical Distribution K5.01 - Knowledge of the operational implications of the following concepts as they apply to D.C. ELECTRICAL 2.6 2.9 DISTRIBUTION : Hydrogen generation during battery charging Level RO I Tier I 2 I Group I 1 General 331 340.1 RAP-U6f, U7f References OCS OPS ILT 13-1 EXAM Page: 20 of 201 13 May 2014

EXAMINATION ANSWER KEY ILT 13~1 NRC Written Exam (RO)

Cis Correct. Procedure 331 and 340.1 requires battery room ventilation be in service prior when the batteries are in service to prevent hydrogen accumulation which is an explosive hazard.

The C Battery H2 Hi-Hi alarm comes in at 1.6% (40% of LEL). The correct action lAW the RAP for a complete loss of ventilation is to open the 'C' Battery Room fire door, install a portable fan, and post a continuous fire watch at the 'C' Battery Room.

Explanation All distractors are Incorrect but plausible since the C Bat H2 Hi alarm comes in at 0.8% (20% of LEl). In addition, there are numerous 4 hr compensatory actions required for different plant events. The applicant may confuse one of these for the actions required by RAP-U6f. A normal125 VDC lineup is stated in the question stem which assumes all batteries are on a float charge.

Lesson 2621.828.0.0012, DC Distribution Plan DCD-10447, Given normal operating procedures and documents Learning for the system, describe or interpret the procedural steps.

Objective/

References ILT: None LORT: Open Provided Question Source (New, Modified, Bank) Bank Previous 2 NRC Exams (ILT Only) Yes Memory or Comprehension Fundamental 1:P Cognitive or Analysis Knowledge Level NUREG 1021 Appendix B: Procedure steps and cautions 55.41b 10 55.43b 10CFR55 Content Administrative, normal, abnormal, and emergency operating procedures for the facility.

Justification for LORT questions with KIA values< 3.0 I NIA Time to Complete: 1*2 minutes Point Value: 1 System ID No.: 263000 PRA: No Safety Function(s): 6 ~ ILT Category(s) (LORT Only): N/A 0 LORT OCS OPS ILT 13-1 EXAM Page: 21 of 201 13 May 2014

EXAMINATION ANSWER KEY ILT 13*1 NRC Written Exam (RO) 10 ID: 13*1 NRO 10 Points: 1.00 The plant was at rated power, with the following components in service:

  • TBCCW Pumps 1-1 and 1*3 The following annunciator alarmed:
  • 460V STATION POWER -1A2 DC LOST While investigating this event, a LOCA occurred.

Which of the following actions can be performed from the CONTROL ROOM, if required?

A. Initiate Containment Spray Pump 51C in DW SPRAY mode.

B. Initiate Containment Spray Pump 51 Bin TORUS CLG mode.

C. Maximize CAD injection into the RPV with both CAD Pumps.

D. Make up to the RPV with CORE SPRAY pumps NZ01 D & NZ03D Answer: A Answer Explanation Knowledge and Ability Reference Information Importance Rating K&A RO SRO 262001 AC Electrical Distribution K5.02 - Knowledge of the operational implications of the 2.6 2.9 following concepts as they apply to A. C. ELECTRICAL DISTRIBUTION: Breaker control Level RO I Tier I 2 J Group I 1 General BR 3002 sh. 2 RAP-U3d References OCS OPS ILT 13-1 EXAM Page: 22 of 201 13 May 2014

EXAMINATION ANSWER KEY ILT 13-1 NRC Written Exam (RO)

B and D are Incorrect but plausible. The initial annunciator describes a loss of DC power to 480 Bus 1A2. Thus there is no ability of control room operators to operate any breaker from the control room on Bus 1A2 due to breaker control power being lost.

This includes the following: core spray booster pump NZ03D; containment spray pump 51 B; and CRD pump A. Since the core spray and containment spray breakers would be open when the Explanation LOCA occurred, then these pumps cannot be started.

C is Incorrect but plausible since both CRD pumps cannot be started.

A is Correct. Containment Spray Pump 51 C is on 480 VAC Bus 1B2 which is not affected by the DC loss.

Lesson 2621.828.0.0012, DC Distribution Plan DCD-10445, Given a set of system indications or data, evaluate Learning and interpret them to determine limits, trends, and system status.

Objective/

References ILT: None LORT: Open Provided Question Source (New, Modified, Bank) Bank Previous 2 NRC Exams (ILT Only) No Memory or Comprehension Fundamental 1:I Cognitive or Analysis Knowledge Level NUREG 1021 Appendix B: Interlocks, setpoints, or system (singular) response 55.41b 7 55.43b 10CFR55 Design, components, and functions of control and safety systems, Content including instrumentation, signals, interlocks, failure modes, and automatic and manual features.

Justification for LORT questions with KIA values< 3.0 I N/A Time to Complete: 1-2 minutes Point Value: 1 System ID No.: 262001 PRA: No Safety Function(s): 6 IZI ILT Category(s) (LORT Only): N/A D LORT OCS OPS ILT 13-1 EXAM Page: 23 of 201 13 May 2014

EXAMINATION ANSWER KEY ILT 13~1 NRC Written Exam (RO) 11 ID: 13-1 NRO 11 Points: 1.00 The plant is in cold shutdown and is cooling down with the Shutdown Cooling System.

Present plant conditions are as follows:

  • RPV water level is in the normal band
  • RBCCW Pumps 1-1 and 1-2 are in-service
  • All Recirculation Pumps are in-service
  • LKOUT RELAY 86/S1A TRIP lAW 305, Shutdown Cooling System Operation, which ONE of the following is required?

A. Start Shutdown Cooling Pump A.

B. Manually re-start RBCCW Pump 1-1.

C. Close Recirculation Pump E discharge valve.

D. Shutdown the Reactor Water Cleanup System.

Answer: C Answer Explanation Knowledge and Ability Reference Information Importance Rating K&A RO SRO 205000 Shutdown Cooling K6.01 - Knowledge of the effect that a loss or malfunction of the following will have on the SHUTDOWN COOLING 3.3 3.4 SYSTEM (RHR SHUTDOWN COOLING MODE) : A.C.

electrical power Level RO I Tier I 2 I Group I 1 General 305 ABN-3 237E798 References TS Table 3.1.1 OCS OPS ILT 13*1 EXAM Page: 24 of 201 13 May 2014

EXAMINATION ANSWER KEY ILT 13*1 NRC Written Exam (RO)

C is Correct. The plant is in cold shutdown and is cooling down with SOC, when power is lost to 4160 VAC Bus 1A (the provided alarm shows a lockout and loss of startup transformer 1A, which was currently powering 4160 Bus 1A). This will also remove power from 4160 VAC Bus 1C, and EOG1 will auto start and load onto Bus 1C.

The SOC system takes a suction on recirculation pump E suction line, and discharges to recirculation pump E discharge line.

When the recirculation pump is in service, there are no problems.

But when recirculation pump E is tripped, then SOC can be short-cycled through the recirculation loop, unless the recirculation loop discharge valve is closed. This action is prescribed in the SOC normal procedure. The discharge valve does have power to close under the given conditions.

lAW 305, P&L 4.2.11, when 4160 VAC Bus 1A is lost, power to recirculation pump E is lost, and its discharge valve must be manually closed to prevent short-cycling in SOC.

Explanation B is Incorrect but plausible. When Bus 1C power is lost, power is also lost to 460 VAC USS Bus 1A2, which powers RBCCW Pump 1-1. But when EOG1 starts and loads, then RBCCW Pump 1-1 will auto start after about 166 seconds. Thus, there is no need to manually restart the RBCCW pump.

A is Incorrect but plausible. Since SOC pumps B and C (initially running) are powered from 460 VAC USS Bus 1B2, which is not impacted from the initial Bus 1A2 power loss, then the same number of SOC pumps and RBCCW pumps will be running after 166 seconds. Thus, the amount of cooling to the RPV has not been diminished. There is no need to start another SOC pump.

0 is Incorrect but plausible. In the loss of SOC ABN, if SOC flow is inadequate, the procedure states to reduce the load on RBCCW such as RWCU. Also, the RWCU pump that was initially running has tripped due to the initial loss of power. Even if there was a need to reduce the load on RBCCW, the RWCU pump has already tripped.

Lesson 2621.828.0.0045, Shutdown Cooling System Plan SOC-10447, Given normal operating procedures and documents Learning for the system, describe or interpret the procedural steps.

Objective/

OCS OPS ILT 13-1 EXAM Page: 25 of 201 13 May 2014

EXAMINATION ANSWER KEY ILT 13-1 NRC Written Exam (RO)

References ILT: None LORT: Open Provided Question Source (New, Modified, Bank) Bank Previous 2 NRC Exams (ILT Only) No Memory or Comprehension Fundamental 3:SPK Cognitive or Analysis Knowledge Level NUREG 1021 Appendix B: Solve a Problem using Knowledge and its meaning 55.41b 7 55.43b 10CFR55 Design, components, and functions of control and safety systems, Content including instrumentation, signals, interlocks, failure modes, and automatic and manual features.

Justification for LORT questions with KIA values< 3.0 I N/A Time to Complete: 1-2 minutes Point Value: 1 System ID No.: 205000 PRA: No Safety Function(s): 4 r8i ILT Category(s) (LORT Only): N/A 0 LORT OCS OPS ILT 13-1 EXAM Page: 26 of 201 13 May 2014

EXAMINATION ANSWER KEY ILT 13-1 NRC Written Exam (RO) 12 10: 13*1 NRO 12 Points: 1.00 The plant was at rated power with EDG-1 running in parallel to 4160 VAC Bus 1C. An event then occurred and the following annunciator came into and is locked into alarm:

  • T-7-b, EDG 1 DAY TNK LVL HI I LO I LUBE OIL FAIL An operator sent out to investigate reports the following:
  • EDG-1 fuel oil tank level is in the NORMAL band
  • EDG-1 Lube Oil Circ Pump pressure is NORMAL
  • EDG-1 Turbo Oil Pump pressure is NORMAL With annunciator T-7-b still in alarm, which ONE of the following states the status of the Turbo Oil Pumps and action required, if any, by the RAP?

The EDG-1 ...

A. DC Turbo Oil Pump is running and EDG-1 may remain in operation.

B. AC Turbo Oil Pump is running and EDG-1 may remain in operation.

C. DC Turbo Oil Pump is running and EDG-1 must be manually tripped.

D. AC Turbo Oil Pump is running and EDG-1 must be manually tripped.

Answer: A Answer Explanation Knowledge and Ability Reference Information Importance Rating K&A RO SRO 264000 EDGs K6.03- Knowledge of the effect that a loss or malfunction 3.5 3.7 of the following will have on the EMERGENCY GENERATORS (DIESEUJET): Lube oil pumps Level RO I Tier I 2 I Group I 1 General RAP-T7b References OCSOPS ILT 13-1 EXAM Page: 27 of 201 13 May 2014

EXAMINATION ANSWER KEY ILT 13-1 NRC Written Exam (RO)

A is Correct. The question describes a condition where annunciator T-7-b came in and is locked into alarm. This alarm comes in on hi/low fuel oil day tank level, low Lube Oil Circ Pump pressure, or low Turbo Oil Pressure. With the operator reporting all these initiating conditions in the normal band, the only condition that would cause annunciator T-7-b to come in and to be locked in is if the AC Turbo Oil Pump tripped and the DC Turbo Oil Pump was now running. This alarm doesn't clear until the AC Turbo Oil Pump is restored to service. The action required by the RAP is to notify the Work Week Manager to repair the AC Turbo Oil Pump. EDG-1 may continue to operate per the RAP in this condition.

B is Incorrect but plausible if the applicant cannot diagnose the Explanation condition or recall which pump should be running. Since Turbo Lube Oil Pressure is now normal, they it's plausible that the AC Turbo Oil Pump is operating and EDG-1 may remain operating.

Cis Incorrect but plausible. It is correct that the DC Turbo Oil Pump is operating for these conditions, however the applicant may believe the RAP requires the EDG tripped due to the alarm condition.

D is Incorrect but plausible if the applicant cannot diagnose the condition or recall which pump should be running. The applicant may believe the RAP requires the EDG tripped due to the alarm condition.

Lesson 2621.828.0.0013, Emergency Diesel Generators Plan EDG-1 0444, Describe the interlock signals and setpoints for the Learning affected system components and expected system response Objective/ including power loss or failed components.

References ILT: None LORT: Open Provided Question Source (New, Modified, Bank) New Previous 2 NRC Exams (ILT Only) No Memory or Comprehension Fundamental 3:PEO Cognitive Knowledge or Analysis Level NUREG 1021 Appendix B: Predict an Event or .Qutcome OCS OPS ILT 13-1 EXAM Page: 28 of 201 13 May 2014

EXAMINATION ANSWER KEY ILT 13*1 NRC Written Exam (RO) 10CFR55 55.41b I 7 I 55.43b I

Design, components, and functions of control and safety systems, Content including instrumentation, signals, interlocks, failure modes, and automatic and manual features.

Justification for LORT questions with KIA values< 3.0 I NIA Time to Complete: 1-2 minutes Point Value: 1 System I D No.: 264000 PRA: No Safety Function(s): 6 ~ ILT Category(s) (LORT Only): N/A 0 LORT OCSOPS ILT 13-1 EXAM Page: 29 of 201 13 May 2014

EXAMINATION ANSWER KEY ILT 13~ 1 NRC Written Exam (RO) 13 10: 13-1 NRO 13 Points: 1.00 The plant is at rated power. The following Panel 4F indications are observed:

PIIIIIIIQIIIIII~I 0 1 I IIIIIIIIIIIIIII BliP 111111111 z 3 4 ~

X 1CIII OALI.O NS:

il' 1'1'1'1'1'1'1'1'1'111'1'11111 0 300 £00 900 1200 1500 tUllYUt$1 An event then occurred which resulted in an Electric ATWS. Actions required by RPV Control- with ATWS are being implemented by the crew.

lAW the EOP User's Guide, which of the following Panel 4F indications is the FIRST to indicate the reactor will remain shutdown under HOT STANDBY conditions?

OCS OPS ILT 13-1 EXAM Page: 30 of 201 13 May 2014

EXAMINATION ANSWER KEY ILT 13~ 1 NRC Written Exam (RO)

A.

IIJIJIIIIIIIIIIIIIIIIIIIIIIIII 0 300 600 900 1200 1500 JIICUU Ef'SI OCSOPS ILT 13-1 EXAM Page: 31 of 201 13 May 2014

EXAMINATION ANSWER KEY ILT 13~ 1 NRC Written Exam (RO)

B.

STANDBY LIQUID CONTROL

)IIIII I~ IIIIIIIIIIIIIIIIIIIIIIIIIIIIIIJIIIIIIIIII 0 1 2 3 4 5 X tiDlOALLONS I: lllllllllllllllllllllllllllllll 0 300 600 800 1200 1500

,_.11\tUIIII:USl FIRE OFF SYS 2

OCS OPS ILT 13-1 EXAM Page: 32 of 20 1 13 May 2014

EXAMINATION ANSWER KEY ILT 13"1 NRC Written Exam (RO) c.

0 l~llllll 1lllllllllqlllllll 2

llllllljUIIJ 111111111 3 5

~1~0AUONS OCSOPSILT13-1 EXAM Page: 33 of 201 13 May 2014

EXAMINATION ANSWER KEY ILT 13~1 NRC Written Exam (RO)

D.

0 1

~2 t lllllllllllllllll IIIIIIIIIIIIIIIUII 111111111 3 4 s X 11DJOALLONS r JliiJI~~~~~~~~~~~~~ ~~~ ~~~~~~ ~~ ~

0 300 600 00 1200 1500

_--..,...- , !._!!SUlC~ll FIRE FIRE SYS OFF SYS 1 2 Answer: A OCS OPS ILT 13-1 EXAM Page: 34 of 201 13 May 2014

EXAMINATION ANSWER KEY ILT 13-1 NRC Written Exam (RO)

Answer Explanation Knowledge and Ability Reference Information Importance Rating K&A RO SRO 211000 SLC A1.01 -Ability to predict and/or monitor changes in 3.6 3.7 parameters associated with operating the STANDBY LIQUID CONTROL SYSTEM controls including: Tank level Level RO I Tier I 2 I Group I 1 General RPVC- with ATWS EOP User's Guide References EOP A is Correct. The EOP Users Guide provides the following: The Hot Shutdown Boron Weight (HSBW) is defined to be the weight of soluble boron that, if injected into the RPV and mixed uniformly, will maintain the Reactor shut down under hot standby conditions. An amount of 750 gallons of boron injected from the Boron Tank has been determined to be the HSBW. Since the initial tank level was 1750 gal, 1000 gal left in the tank indicates the required 750 gal to reach the HSBW was injected. The student must Analyze and Interpret the indications provided to determine the correct answer (Comprehension or Analysis).

B is Incorrect but plausible. SLC tank indicates 750 gal left in the tank. The Hot Shutdown Boron Weight (HSBW) is achieved when 750 gallons of boron has been injected (1750 gal -750 gal=

Explanation 1000 gal). It is plausible the applicant may confuse the requirement that 750 gal must be injected in the tank, NOT that 750 gal is left in the tank.

C is Incorrect but plausible. SLC tank indicates 150 gal. This value corresponds to the Cold Shutdown Boron Weight (CSBW),

where the reactor will remain shutdown under ALL conditions regardless of control rod position.

D is Incorrect. SLC tank indicates 0 gal. It is true the HSBW has been injected at this point however the question asks which is the FIRST of the four SLC tank indications the applicant can call the reactor shutdown under hot standby conditions. The applicant may not recall the HSBW requirement and believe all boron within the tank must be injected.

Lesson 2621.845.0.0053, RPV Control -with ATWS Plan EWA-2257, Given the EOP, describe in detail each step/statement, OCS OPS ILT 13-1 EXAM Page: 35 of 201 13 May 2014

EXAMINATION ANSWER KEY ILT 13~1 NRC Written Exam (RO)

Lesson 2621.845.0.0053, RPV Control - with ATWS Plan EWA-2257, Given the EOP, describe in detail each step/statement, including the technical basis, and how to verify or perform each Learning step.

Objective/

References ILT: None LORT: Open Provided Question Source (New, Modified, Bank) Modified Previous 2 NRC Exams (ILT Only) No Memory or Comprehension Fundamental 3:SPK Cognitive or Analysis Knowledge Level NUREG 1021 Appendix 8: .Solve a Problem using Knowledge and its meaning 55.41b 10 .S5 . 43b 10CFR55 Content Administrative, normal, abnormal, and emergency operating procedures for the facility.

Justification for LORT questions with KIA values< a.o I N/A Time to Complete: 1-2 minutes Point Value: 1 System ID No.: 211000 PRA: No Safety Function(s): 1 ~ ILT Category(s) (LORT Only): N/A 0 LC>RT OCS OPS ILT 13-1 EXAM Page: 36 of 201 13 May 2014

EXAMINATION ANSWER KEY ILT 13~1 NRC Written Exam (RO) 14 10: 13*1 NRO 14 Points: 1.00 The following plant conditions and sequence of events occur:

  • The plant is operating at 50% power
  • Master Feed Controller is set at 163"
  • Reactor water level is 163"
  • Reactor pressure is 1019 psig At T =0 seconds, a manual scram is inserted and a Hydraulic ATWS occurs.

At T = 15 seconds the following plant conditions exist

  • Reactor power is 25%
  • Reactor water level has lowered to 152"
  • Reactor pressure is 1015 psig With NO operator action, how will the feedwater control system respond to maintain level?

Reactor water level will be automatically controlled at a ...

A. 142" setpoint using the low flow regulating valves B. 163" setpoint using the low flow regulating valves C. 163" setpoint using the main feed regulating valves D. 142" setpoint using the main feed regulating valves Answer: 0 Answer Explanation Knowledge and Ability Reference Information Importance Rating K&A RO SRO 259002 Reactor Water Level Control A1.01 -Ability to predict and/or monitor changes in parameters associated with operating the REACTOR 3.8 3.8 WATER LEVEL CONTROL SYSTEM controls including:

Reactor water level OCS OPS ILT 13-1 EXAM Page: 37 of 201 13 May 2014

EXAMINATION ANSWER KEY ILT 13-1 NRC Written Exam (RO)

Level RO I Tier I 2 I Group I 1 General MDD-OC-625-B Div I References D is Correct. A normal scram is processed through RPS to Feedwater Control and validated by a 10 11 drop in level. With no operator action, the Reactor Feedwater Control System will control RPV level at 142 11

  • A is Incorrect. This distractor is plausible if the applicant believes the Low Flow Valves will be regulating flow for these conditions. With no operator action, the MFRVs will still be Explanation controlling level.

Band Care Incorrect. These distractors are plausible if the applicant believes the post scram level control setpoint was not processed due to the Hydraulic ATWS, however since a >10 11 drop in level did occur, a normal scram was processed by RPS and the Reactor Feedwater Control System. With no operator action, the MFRVs will still be controlling level.

Lesson 2621.828.0.0018, Feedwater Control System Plan FWC-1 0446, Identify and explain system operating controls I indications under all plant operating conditions.

Learning Objective/

References ILT: None LORT: Open Provided Question Source (New, Modified, Bank) Modified Previous 2 NRC Exams (ILT Only) No Memory or Comprehension Fundamental 3:PEO Cognitive or Analysis Knowledge Level NUREG 1021 Appendix 8: Predict an Event or .Qutcome 55.41b 7 55.43b 10CFR55 Design, components, and functions of control and safety systems, Content including instrumentation, signals, interlocks, failure modes, and automatic and manual features.

Justification for LORT questions with KIA values< 3.0 N/A Time to Complete: 1-2 minutes Point Value: 1 System I D No.: 259002 PRA: No Safety Function(s): 2 ~ ILT OCS OPS ILT 13-1 EXAM Page: 38 of 201 13 May 2014

EXAMINATION ANSWER KEY ILT 13~1 NRC Written Exam (RO)

I Category(s) (LORT Only): I NIA lo LORT OCS OPS ILT 13-1 EXAM Page: 39 of 201 13 May 2014

EXAMINATION ANSWER KEY ILT 13~1 NRC Written Exam (RO) 15 10: 13*1 NRO 15 Points: 1.00 Given the following:

  • A plant startup is in progress
  • The reactor mode switch is in STARTUP
  • SRM 22 is bypassed due to a failed detector
  • I AM's 11 ~ 16 and 18 are indicating 25% on Range 8
  • IRM 17 is indicating 75% on Range 7
  • SRM 23 experiences an INOP condition due to a degraded power supply Which statement below describes how this impacts the reactor startup?
1. A withdraw rod block _ _ _ __
2. The reactor startup _ _ _ __

A. (1) is generated (2) can continue because IRM 17 can be switched to Range 8 B. (1) is generated (2) CANNOT continue because the rodblock cannot be bypassed C. (1) is NOT generated (2) can continue because only two SAM's are required to be operable during a reactor startup D. (1) is NOT generated (2) CANNOT continue because more than two SAM's are required to be operable during a reactor startup Answer: A Answer Explanation Knowledge and Ability Reference Information Importance Rating K&A RO SAO 215004 Source Range Monitor A2.01 -Ability to (a) predict the impacts of the following on the SOURCE RANGE MONITOR (SAM) SYSTEM; and (b) 2.7 2.9 based on those predictions, use procedures to correct, control, or mitigate the consequences of those abnormal conditions or operations: Power supply degrade~

Level I RO I Tier I 2 I Group I 1 OCS OPS ILT 13-1 EXAM Page: 40 of 201 13 May 2014

EXAMINATION ANSWER KEY ILT 13~1 NRC Written Exam (RO)

General 201 401.4; 402.3 RAP-G4d References A is Correct. A rod block is generated due to the SRM 23 INOP condition and not ALL of the correlating IRMs (15, 16, 17 and 18) are on or above Range 8. Since IRM 17 is at the top of the 25-75%

band, it can be switched to Range 8, as directed by Procedures 201 and 402.3. This will bypass the SRM 23 rod block, allowing control rod withdrawal to continue.

B is Incorrect but plausible. The first statement is correct.

However, while it is true that one SRM is already bypassed, switching IRM 17 to range 8 automatically bypasses all SRM rod block functions, which will allow control rod withdrawal to Explanation continue. This is allowed (directed) by Procedures 201 and 402.3.

C is Incorrect but plausible. The first statement is incorrect because a withdraw rod block IS generated. The second statement is correct, although insignificant for the given conditions.

Dis Incorrect but plausible. The first statement is incorrect because a withdraw rod block IS generated. The second statement is also incorrect in that Procedure 201 only requires two operable SRM's during a reactor startup (until aiiiRM's are on Range 8 or above).

Lesson 2621.828.0.0029 Nuclear Instrumentation Plan Learning NIS-1 0444 Describe the interlock signals and setpoints for the Objective/ affected system components and expected system response including power loss or failed components.

References ILT: None LORT: Open Provided Question Source (New, Modified, Bank) Bank Previous 2 NRC Exams (ILT Only) No Memory or Comprehension Fundamental 3:SPK Cognitive Knowledge or Analysis Level NUREG 1021 Appendix B: Solve a Problem using Knowledge and its meaning OCS OPS ILT 13-1 EXAM Page: 41 of 201 13 May 2014

EXAMINATION ANSWER KEY ILT 13~1 NRC Written Exam (RO) 10CFR55 55.41b I 10 I 55.43b I

Content Administrative, normal, abnormal, and emergency operating procedures for the facility.

Justification for LORT questions with KIA values< 3.0 I NIA Time to Complete: 1-2 minutes Point Value: 1 System 10 No.: 215004 PRA: No Safety Function(s): 7 ~ IL'T Category(s) (LORT Only): N/A 0 LORT OCS OPS ILT 13-1 EXAM Page: 42 of 201 13 May 2014

EXAMINATION ANSWER KEY ILT 13~1 NRC Written Exam (RO) 16 10: 13-1 NRO 16 Points: 1.00 The plant was at rated power when the following annunciator alarmed:

  • RBCCW- SURGE TANK LVL HI/LO The RB Operator reports the RBCCW Surge Tank indicates 1" by sightglass.

lAW the RAP, which of the following actions is correct?

A. Scram the reactor lAW ABN-1 due do the loss of NPSH.

B. Confirm proper operation of the Surge Tank makeup valve.

C. Manually fill the Surge Tank from the Demineralized Water System utilizing the makeup valve bypass.

D. Manually fill the Surge Tank from the Condensate Transfer System utilizing the normal makeup valve.

Answer: B Answer Explanation Knowledge and Ability Reference Information Importance Rating K&A RO SRO 400000 Component Cooling Water A2.02 - Ability to (a) predict the impacts of the following on the CCWS and (b) based on those predictions, use 2.8 3.0 procedures to correct, control, or mitigate the consequences of those abnormal operation: High/low surge tank level Level RO I Tier I 2 I Group I 1 General 334 RAP-C5c References OCSOPS ILT 13-1 EXAM Page: 43 of 201 13 May 2014

EXAMINATION ANSWER KEY ILT 13~1 NRC Written Exam (RO)

B is Correct. The alarm response in the question stem shows that 1" is low level in the surge tank, and directs the operator to check automatic operation of the mlu surge tank valve.

A is Incorrect but plausible. ABN-19, RBCCW Failure Response requires a manual scram IF there is a Major Unisolable RBCCW leak (leak exceeding the makeup capacity to the RBCCW surge tank AND a leak which cannot be isolated quickly AND a leak which results in imminent loss of the RBCCW system due to loss of NPSH to RBCCW pumps). No indications are given which Explanation indicate a major unisolable leak.

Cis Incorrect but plausible. The RAP says to correct as required.

One way would be to manually open the makeup valve, which is supplied from Demineralized Water System. There is no makeup valve bypass therefore this choice is Incorrect.

Dis Incorrect but plausible since this is a correct action, however the surge tank makeup is from Demineralized Water, not Condensate Transfer.

Lesson 2621.828.0.035, RBCCW System Plan RBC-00058, State the RBCCW System alarms and setpoints.

Learning Objective/

References ILT: None LORT: Open Provided Question Source (New, Modified, Bank) Bank Previous 2 NRC Exams (ILT Only) No Memory or Comprehension Fundamental 1:P Cognitive or Analysis Knowledge Level NUREG 1021 Appendix B: Procedure steps and cautions 55.41b 10 55.43b 10CFR55 Content Administrative, normal, abnormal, and emergency operating procedures for the facility.

Justification for LORT questions with KIA values< 3.0 I N/A Time to Complete: 1-2 minutes Point Value: 1 System I D No.: 400000 PRA: No OCS OPS ILT 13*1 EXAM Page: 44 of 201 13 May 2014

EXAMINATION ANSWER KEY ILT 13-1 NRC Written Exam (RO)

Safety Function(s): 8 181 ILT Category(s) (LORT Only): N/A 0 LORT OCS OPS ILT 13-1 EXAM Page: 45 of 201 13 May 2014

EXAMINATION ANSWER KEY ILT 13-1 NRC Written Exam (RO) 17 10: 13*1 NRO 17 Points: 1.00 The plant was at rated power when a small LOCA occurred. Present plant conditions are as follows:

  • RPV water level lowered to 122" and now indicates 146" and rising slowly
  • RPV pressure indicates 925 psig and lowering slowly

CORE SPRAY- SYSTEM 1 AUTOSTART

  • CORE SPRAY- SYSTEM 2 AUTOST ART One minute later, the Operator notes the following valves indicate RED light ON:
  • RBCCW Isolation Valves, V-5-147, V-5-167, and V-5-166
  • Reactor Recirc Sample Valves, V-24-29 and V-24-30
  • Drywell N2 Makeup Isolation Valves, V-23-17 and V-23-18
  • Drywell Instrument Air/N 2 Isolation Valve, V-6-395 Which of the following valve manipulations is required to maintain Primary Containment integrity?

Manually close t h e - - - - - - - - - - - - - - - - valve(s).

A. RBCCW Isolation B. Reactor Recirc Sample C. Drywell N2 Makeup Isolation D. Drywell Instrument Air/N 2 Isolation Answer: C Answer Explanation Knowledge and Ability Reference Information Importance Rating K&A RO SAO 223002 PCIS/Nuclear Steam Supply Shutoff K4.03 *Knowledge of PRIMARY CONTAINMENT ISOLATION SYSTEM/NUCLEAR STEAM SUPPLY SHUT* 3.5 3.6 OFF design feature(s) and/or interlocks which provide for the following: Manual initiation capability: Plant-Specific Level I RO I Tier I 2 I Group I 1 OCS OPS ILT 13-1 EXAM Page: 46 of 201 13 May 2014

EXAMINATION ANSWER KEY ILT 13~1 NRC Written Exam (RO)

General RAP-B1e RAP-B1f 312.9 References C is Correct. The plant was at rated power when a LOCA occurred resulting in a high OW pressure condition and a containment isolation signal (>2.9 psig). Even though OW pressure is not provided, the core spray alarms come in from either a high OW pressure OR lo-lo RPV water level (86"). Since RPV water level only went down to 122", then core spray must have started from high OW pressure. All the valves listed are primary containment isolation valves. Of the valves listed, only the Orywell N2 Makeup Isolation should have closed on a high OW pressure signal and did not. This valve should be manually closed.

Explanation A is Incorrect but plausible. The RBCCW isolation valves will close on a high OW pressure plus an RPV water level lo-lo signal (86") OR an RPV water level lo-lo-lo (64.6"). But, the stem says that RPV water level only lowered to 122". Thus, RBCCW should not have isolated.

B is Incorrect but plausible. The Reactor Recirc Sample valves close on an MSIV isolation signal, which is not a high OW pressure signal and should be open.

0 is Incorrect but plausible. The Orywelllnstrument Air/N 2 Isolation also closes on an MSIV isolation signal, which is not a high OW pressure signal and should be open.

Lesson 2621.828.0.0032, Primary Containment System Plan PCS-390, Verbally describe (with or without support materials),

Learning the Primary Containment System Fans and isolation valves.

Objective/

References ILT: None LORT: Open Provided Question Source (New, Modified, Bank) Bank Previous 2 NRC Exams (ILT Only) No Memory or Comprehension Fundamental 3:SPK Cognitive or Analysis Knowledge Level NUREG 1021 Appendix B: .Solve a f.roblem using Knowledge and its meaning 10CFR55 55.41b 7 55.43b Content OCSOPS ILT 13-1 EXAM Page: 47 of 201 13 May 2014

EXAMINATION ANSWER KEY ILT 13~1 NRC Written Exam (RO)

Design, components, and functions of control and safety systems, including instrumentation, signals, interlocks, failure modes, and automatic and manual features.

Justification for LORT questions with KIA values< 3.0 I NIA Time to Complete: 1-2 minutes Point Value: 1 System I D No.: 223002 PRA: No Safety Function(s): 5 IZI ILT Category(s) (LORT Only): NIA 0 LORT OCS OPS ILT 13-1 EXAM Page: 48 of 201 13 May 2014

EXAMINATION ANSWER KEY ILT 13-1 NRC Written Exam (RO) 18 ID: 13*1 NRO 18 Points: 1.00 Given the following plant conditions:

  • A Loss of Coolant Accident (LOCA) is in progress
  • Drywell pressure is 3.8 psig and rising
  • RPV level is 122" TAF and lowering

A. 100 seconds after RPV level lowers to the Lo-Lo set point.

B. 120 seconds after RPV level lowers to the Lo-Lo setpoint.

C. 100 seconds after RPV level lowers to the Lo-Lo-Lo setpoint.

D. 120 seconds after RPV level lowers to the Lo-Lo-Lo setpoint.

Answer: D Answer Explanation Knowledge and Ability Reference Information Importance Rating K&A RO SRO 218000 ADS A3.09 - Ability to monitor automatic operations of the 4.1 4.2 AUTOMATIC DEPRESSURIZATION SYSTEM including:

Reactor vessel water level Level RO I Tier I 2 I Group I 1 General RAP B-1-g References OCS OPS ILT 13*1 EXAM Page: 49 of 201 13 May 2014

EXAMINATION ANSWER KEY ILT 13~1 NRC Written Exam (RO)

D is the correct answer. In the automatic depressurization mode, the ADS is actuated on simultaneous occurrence of high drywell pressure(> 2.9 psig), lo-lo-lo reactor water level (64.6" TAF), and core spray system operation as verified by a differential pressure across the core spray booster pump (DP > about 30 psid with no time delay). There are two timers (time delay relays) in each ADS logic channel (4 total). To initiate ADS, both timers in one channel must time out. The actual time delay is set at 105 Explanation seconds. Most general materials refer to a 120 second time delay which makes this number a good answer.

All distractors are Incorrect but plausible if the applicant does not recall the RPV level that ADS actuates on or the time it takes to time out. Level must drop to the Lo-Lo-Lo setpoint, not the Lo-Lo setpoint and timers must time out for 120 seconds (1 05 seconds setpoint), not 100 seconds.

Lesson 2621.828.0.0005, Automatic Depressurization System Plan ADS-10357, List the initiation signals that are required to cause Learning an ADS actuation.

Objective/

References ILT: None LORT: Open Provided Question Source (New, Modified, Bank) Bank Previous 2 NRC Exams (ILT Only) No Memory or Comprehension Fundamental 1:I Cognitive or Analysis Knowledge Level NUREG 1021 Appendix B: interlocks, setpoints, or system (singular) response 55.41b 7 55.43b 10CFR55 Design, components, and functions of control and safety systems, Content including instrumentation, signals, interlocks, failure modes, and automatic and manual features.

Justification for LORT questions with KIA values< 3.0 I N/A Time to Complete: 1*2 minutes Point Value: 1 System ID No.: 218000 PRA: No Safety Function(s): 3 1Z1 ILT Category(s) (LORT Only): NIA 0 LORT OCS OPS ILT 13-1 EXAM Page: 50 of 201 13 May 2014

EXAMINATION ANSWER KEY ILT 13~1 NRC Written Exam (RO) 19 10: 13-1 NRO 19 Points: 1.00 The plant was at rated power when the following occurs:

  • VMCC-1 82 temporarily lost power Which ONE of the following statements is correct when power is restored to VMCC-182?

Power to CIP-3 will automatically transfer from ...

A. DC-B to VMCC-1 82 with NO time delay.

B. VMCC-1 A2 to VMCC-1 82 with NO time delay.

C. DC-B to VMCC-1 82 after a 2-m in time delay.

D. VMCC-1A2 to VMCC-1 82 after a 2-m in time delay.

Answer: C Answer Explanation Knowledge and Ability Reference Information Importance Rating K&A RO SRO 262002 UPS (AC/DC)

A4.01 - Ability to manually operate and/or monitor in the 2.8 3.1 control room: Transfer from alternative source to preferred source Level RO I Tier I 2 I Group I 1 General 339 References OCS OPS I LT 13-1 EXAM Page: 51 of 201 13 May 2014

EXAMINATION ANSWER KEY ILT 13~1 NRC Written Exam (RO)

C is Correct. The question stem describes a condition where power was lost to Vital Motor Control Center 1B2 (VMCC-1 B2).

This is the Normal source of power to Continuous Instrument Panel (CIP) -3 via a rotary inverter. When power was lost to VMCC-1 B2, power to the rotary inverter was automatically transferred to DC*B, which powers CIP-3. The other alternate source of power to CIP-3 is directly via VMCC-1A2. lAW 339, Vital Power Systems, when power is restored to the AC source of power to the rotary inverter, power will automatically transfer from the DC source back to the AC source following a 2-min time delay. The UPS for Oyster Creek is CIP-3. The operator will monitor and verify this automatic transfer from the Main Control Explanation Room once power is restored to VMCC-1 B2.

A is Incorrect but plausible if the applicant does not recall there is a time delay associated with the automatic transfer.

B and D are Incorrect. These distractors are plausible since VMCC-1A2 is an alternate source of power to CIP-3, but the second alternate source (CIP-3 will automatically transfer to VMCC-1A2 if power is lost to both VMCC-1B2 and DC-B). They may also not recall there*s a time delay associated with the transfer.

Lesson 2621.828.0.0056, Vital AC Distribution Plan VAC-10441, Given the system logic/electrical drawings, describe Learning the system trip signals, setpoints and expected system response Objective/ including power loss or failed components.

References ILT: None LORT: Open Provided Question Source (New, Modified, Bank) New Previous 2 NRC Exams (ILT Only) No Memory or Comprehension Fundamental 3:PEO Cognitive Knowledge or Analysis Level NUREG 1021 Appendix B: Predict an .Event or .Outcome 55.41b 7 55.43b 10CFR55 Design, components, and functions of control and safety systems, Content including instrumentation, signals, interlocks, failure modes, and automatic and manual features.

OCS OPS ILT 13-1 EXAM Page: 52 of 201 13 May 2014

EXAMINATION ANSWER KEY ILT 13~1 NRC Written Exam (RO)

Justification for LORT questions with KIA values< 3.0 I NIA Time to Complete: 1*2 minutes Point Value: 1 System I D No.: 262002 PRA: No Safety Function(s): 6 IZI ILT Category(s) (LORT Only): NIA 0 LORT OCS OPS ILT 13-1 EXAM Page: 53 of 201 13 May 2014

EXAMINATION ANSWER KEY ILT 13-1 NRC Written Exam (RO) 20 10: 13*1 NRO 20 Points: 1.00 The plant is at rated power.

Which of the following three plant indications together show that EMRV NR1 08A is LEAKING BY?

A. Torus water AUTO DEPRESS VALVE Indicated feed flow temperature NR108A red light ON and LESS than indicated rising green light OFF steam flow B. Torus water AUTO DEPRESS VALVE EMRV tail pipe temperature NR108A red light OFF and temperature above rising green light ON normal

c. MWe lowering Annunciator EMRV OPEN NR108A indicates in in alarm the VALVE CLOSED REGION D. MWe lowering EM RV tail pipe Indicated feed flow temperature above normal LESS than indicated steam flow Answer: B Answer Explanation Knowledge and Ability Reference Information Importance Rating K&A RO SRO 239002 SRVs A4.04
  • Ability to manually operate and/or monitor in the 4.3 4.3 control room: Suppression pool temperature Level RO I Tier I 2 I Group I 1 General ABN-40 BR 2002 sh. 1 RAP*B3g References OCS OPS ILT 13*1 EXAM Page: 54 of 201 13 May 2014

EXAMINATION ANSWER KEY ILT 13~1 NRC Written Exam (RO)

B is Correct and A is Incorrect. The EMRVs relieve to the torus.

When the valve leaks by, steam will go to the torus and the EMRV tailpipe temperature will rise, and torus water will heatup. The valves are normally closed and leak-by can only occur when the valve is closed. A closed valve is indicated by the red light OFF and the green light ON. Therefore, answer A is incorrect (red light ON). Only answer B has the correct indications.

Explanation Answer Cis Incorrect but plausible since EMRV OPEN annunciator comes in from the EMRV pilot valve limit switch, which when activates, means that the valve is open.

Answer Dis Incorrect but plausible if the applicant does not understand that steam flow would be less than feed flow (not feed < steam).

Lesson 2621.828.0.0005, Automatic Depressurization System Plan ADS-00375, Given ADS alarms and indications, evaluate them in Learning terms of limits and trends.

Objective/

References ILT: None LORT: Open Provided Question Source (New, Modified, Bank) Bank Previous 2 NRC Exams (ILT Only) No Memory or Comprehension Fundamental 2:DR Cognitive or Analysis Knowledge Level NUREG 1021 Appendix B: Describing or recognizing Relationships 55.41b 5 55.43b Facility operating characteristics during steady state and transient 10CFR55 conditions, including coolant chemistry, causes and effects of Content temperature, pressure and reactivity changes, effects of load changes, and operating limitations and reasons for these operating characteristics.

Justification for LORT questions with KIA values< 3.0 I N/A Time to Complete: 1*2 minutes Point Value: 1 System ID No.: 239002 PRA: No Safety Function(s): 3 ~ ILT Category(s) (LORT Only): N/A 0 LORT OCS OPS ILT 13-1 EXAM Page: 55 of 201 13 May 2014

EXAMINATION ANSWER KEY ILT 13~1 NRC Written Exam (RO) 21 10: 13*1 NRO 21 Points: 1.00 Which of the following describes the correct EMRV ALTERNATE Power Supply?

EMBV EMBV EMRV EMRV EMRV A .e. .c D E A DC-0 DC-F DC-0 DC-F DC-0 B. DC-F DC-F DC-0 DC-0 DC-F

c. DC-F DC-0 DC-F DC-D DC-F D. DC-0 DC-0 DC-F DC-F DC-0 Answer: C Answer Explanation Knowledge and Ability Reference Information Importance Rating K&A RO SRO 239002 SRVs K2.01 -Knowledge of electrical power supplies to the 2.8 3.2 following: SRV solenoids Level RO I Tier I 2 I Group I 1 General RAP-B5g RAP-9XF6d References OCSOPS ILT 13-1 EXAM Page: 56 of 201 13 May 2014

EXAMINATION ANSWER KEY ILT 13~1 NRC Written Exam (RO)

C is Correct. The Alternate power supplies for the EMRVs are as follows: EMRV A, C, E is DC*F; EMRV B & D is DC-D. The Normal power supplies for the EMRVs are as follows: EMRV A, C, E is DC-D; EMRVs B & D is DC-F.

A is Incorrect. This distractor is plausible if the student reverses the Normal and Alternate power supplies to the EMRVs.

B is Incorrect. There is other logic in the plant where A, B, & E are grouped together (i.e. A, B, E EMRVs are on the South main steam header and B & D EMRVs are on the North main steam header. Another example is A, B, & E Recirc pumps trip on high Explanation RPV pressure <1 0.5seconds). This distractor is plausible if the student does not recall the correct Alternate power supply grouping to the EMRVs.

D is Incorrect. There is other logic in the plant where A, B, & E are grouped together (i.e. A, B, E EMBVs are on the South main steam header and B & D EMRVs are on the North main steam header. Another example is A, B, & E Recirc pumps trip on high RPV pressure <1 0.5seconds). This distractor is plausible if the student does not recall the correct Alternate power supply grouping to the EMRVs and reverses the Normal and Alternate power supplies to the EMRVs.

Lesson 2621.828.0.0005, Automatic Depressurization System Plan 369, State how the following systems interrelate with ADS: 1.

Vessel and Primary Containment Instrumentation 2. Core spray 3.

Learning NSSS 4. Vital AC Power 5. 125V DC Power Objective/

References ILT: None LORT: Open Provided Question Source (New, Modified, Bank) Bank Previous 2 NRC Exams (ILT Only) No Memory or Comprehension Fundamental 1 :F Cognitive Knowledge or Analysis Level NUREG 1021 Appendix B: Facts 55.41b 7 55.43b 10CFR55 Design, components, and functions of control and safety systems, Content including instrumentation, signals, interlocks, failure modes, and automatic and manual features.

L-------~-----------------------------*...-------------------~

OCS OPS ILT 13-1 EXAM Page: 57 of 201 13 May 2014

EXAMINATION ANSWER KEY ILT 13~1 NRC Written Exam (RO)

Justification for LORT questions with KIA values< 3.0 I NIA Time to Complete: 1*2 minutes Point Value: 1 System ID No.: 239002 PRA: No Safety Function(s): 3 181 ILT Category(s) (LORT Only): N/A 0 LORT OCS OPS ILT 13-1 EXAM Page: 58 of 201 13 May 2014

EXAMINATION ANSWER KEY ILT 13~1 NRC Written Exam (RO) 22 10: 13-1 NRO 22 Points: 1.00 The plant is at rated power. On your Control Room tour during turnover, you note the change in status in the Standby Gas Treatment System (SGTS) as shown below:

INFORMATIO Tripping the feeder breaker to _ _ _ _ for maintenance will impact the LCO for the SGTS.

A. USS 1A2 B. USS 182 C. USS 183 D. VMCC 182 Answer: B Answer Explanation Knowledge and Ability Reference Information Importance Rating K&A RO T SRO OCS OPS ILT 13-1 EXAM Page: 59 of 201 13May2014

EXAMINATION ANSWER KEY ILT 13~1 NRC Written Exarn (RO) 261000 SGTS 2.2.42- Equipment Control:: Ability to recognize system 3.9 4.6 parameters that are entry-level conditions for Technical Specifications.

Level RO I Tier I 2 I Group I 1 General TS 3.5.B.7 330 BR 3002 sh. 2 References B is Correct. The plant is at rated power with indications showing that SGTS Fan 1 is inoperable with its breaker open and tagged out of service. TS 3.5.B.6 allows a 7-day LCO with one SGTS train inoperable. If both trains were inoperable, then TS 3.5.B.7 will require a 24-hour shutdown.

Explanation SGTS fan B (EF 1*9) is powered from MCC 1B24, which is fed from USS 1B2. Thus, troubleshooting on the feeder breaker to USS 1B2 has the potential to de-energize USS 1B2, which would inop the only remaining SGTS fan, and TS 3.5.B.7 would apply for SGTS.

All other answers are plausible sources of power but incorrect.

Lesson 2621.812.0.0042, Secondary Containment & SGTS Plan SBG-10445, Given a set of system indications or data, evaluate and interpret them to determine limits, trends and system status.

Learning Objective/

References ILT: None LORT: Open Provided Question Source (New, Modified, Bank) Bank Previous 2 NRC Exams (ILT Only) No Memory or Comprehension Fundamental 2:DR Cognitive or Analysis Knowledge Level NUREG 1021 Appendix B: Describing or recognizing Relationships 55.41b 10 55.43b 10CFR55 Content Administrative, normal, abnormal, and emergency operating procedures for the facility.

Justification for LORT questions with KIA values< 3.0 N/A Time to Complete: 1*2 minutes Point Value: 1 System ID No.: 261000 PRA: No OCS OPS ILT 13-1 EXAM Page: 60 of 201 13 May 2014

EXAMINATION ANSWER KEY ILT 13~1 NRC Written Exam (RO)

Safety Function(s): 9 IZI ILT Category(s) (LORT Only): N/A 0 LORT OCS OPS ILT 13*1 EXAM Page: 61 of 201 13 May 2014

EXAMINATION ANSWER KEY ILT 13~1 NRC Written Exam (RO) 23 10: 13-1 NRO 23 Points: 1.00 The plant is at rated power. An event then occurred and the following annunciator came into alarm:

  • FCS/RFCS- DUAL LINK FAILURE Which ONE of the following describes the effect on the Digital Feedwater Control System (DFCS) AND RPV Water Level?

DFCS (1) and RPV Water Level (2)

A. (1) transfers to the Moore Stations (2) drops until operator action is taken B. (1) does NOT transfer to the Moore Stations (2) drops until operator action is taken C. (1) transfers to the Moore Stations (2) remains constant since DFCS functions to maintain last known setpoint D. (1) does NOT transfer to the Moore Stations (2) remains constant since DFCS functions to maintain last known setpoint Answer: D Answer Explanation Knowledge and Ability Reference Information Importance Rating K&A RO SRO 259002 Reactor Water Level Control K1.04 - Knowledge of the physical connections and/or cause- effect relationships between REACTOR WATER 3.5 3.6 LEVEL CONTROL SYSTEM and the following: Reactor feedwater flow Level RO I Tier I 2 J Group J 1 General RAP-J2c, J1c References OCS OPS ILT 13-1 EXAM Page: 62 of 201 13 May 2014

EXAMINATION ANSWER KEY ILT 13~1 NRC Written Exam (RO)

Dis Correct. A Dual Link failure alarm informs the control room operators that the Moore Stations are disabled and the DCCs continue to function normally based on the last settings obtained from the Moore stations. As a result, the FRV's will continue to maintain Feedwater Flow constant and RPV water level at the same setpoint so there is no impact on level.

A is Incorrect but plausible. Moore stations are disabled upon a Explanation dual link failure. This would be the expected condition for a dual computer failure. Level will remain the same.

8 is Incorrect but plausible since it is the correct DCFS response, but incorrect RPV water level response.

C is Incorrect but plausible since it is the incorrect DFCS response, but correct RPV water level response.

Lesson 2621.828.0.0018, Feedwater Control System Plan FWC-1 0449, State the function and interpretation of system alarms, alone and in combination, as applicable in accordance Learning with the system RAPS.

Objective/

References ILT: None LORT: Open Provided Question Source (New, Modified, Bank) Modified Previous 2 NRC Exams (ILT Only)

- No Memory or Comprehension Fundamental 3:PEO Cognitive or Analysis Knowledge Level NUREG 1021 Appendix 8: Predict an Event or Qutcome 55.41b 7 55.43b 10CFR55 Design, components, and functions of control and safety systems, Content including instrumentation, signals, interlocks, failure modes, and automatic and manual features.

Justification for LORT questions with KIA values< 3.0 I N/A Time to Complete: 1*2 minutes Point Value: 1 System I D No.: 259002 PRA: No Safety Function(s): 2 ~ ILT Category(s) (LORT Only): NIA 0 LORT OCS OPS ILT 13*1 EXAM Page: 63 of 201 13 May 2014

EXAMINATION ANSWER KEY ILT 13~1 NRC Written Exam (RO) 24 10: 13-1 NRO 24 Points: 1.00 The plant was at rated power with the following air compressor indications:

  • Air Compressor 1 is running in Lead
  • Air Compressor 2 is in Standby
  • Air Compressor 3 is tagged out of service The following annunciator then alarmed:
  • COMPR 1 TRIP The SAO has directed a re-start of the compressor.

lAW 334, Instrument and Service Air, which of the following is correct to place Air Compressor 1 back in LEAD from the Control Room?

A. Reset the breaker locally. Then place the COMPRESSOR 1 control switch to START.

B. Reset the breaker locally. Then hold the COMPRESSOR 1 control switch in START for 3-5 seconds.

C. The local reset at the compressor must be pressed once and then wait for the start logic to be satisfied. Then place the COMPRESSOR 1 control switch to START.

D. The local reset at the compressor must be pressed twice and then wait for the start logic to be satisfied. Then hold the COMPRESSOR 1 control switch in START for 3-5 seconds.

Answer: D Answer Explanation Knowledge and Ability Reference Information Importance Rating K&A RO SRO 300000 Instrument Air K4.01 - Knowledge of INSTRUMENT AIR SYSTEM design 2.8 2.9 feature(s) and or interlocks which provide for the following: Manual/automatic transfers of control Level RO l Tier I 2 I Group J 1 General RAP-M4a 334 References RAP-M5a OCS OPS ILT 13-1 EXAM Page: 64 of 201 13 May 2014

EXAMINATION ANSWER KEY ILT 13~1 NRC Written Exam (RO)

D is Correct. The plant is at power with air compressor 1 is lead, air compressor 2 is standby, and air compressor 3 is tagged out of service. An annunciator alarms which describes a trip of compressor 1. The trip is spurious is and the SRO directs a restart of the compressor. lAW the procedure, to place air compressor 1 back in lead, the local reset at the compressor must be pressed twice and then wait for the start logic to be satisfied. Then hold the COMPRESSOR 1 control switch in START for 3*5 seconds.

Explanation A & B are Incorrect but plausible. There is another annunciator, COMPR 1 BREAKER TRIP, which opens the air compressor breaker on overload only. This will require actions at the breaker to restart the air compressor. But this is not the annunciator given in the question.

Answer C is Incorrect but plausible since the air compressor switch must be held for 3*5 seconds, not just placed in start, and the reset must be pressed twice.

Lesson 2621.828.0.0043, Service, Instrument & Breathing Air Plan Learning CAS-10447, Given normal operating procedures and documents Objective/ for the system, describe or interpret the procedural steps.

References ILT: None LORT: Open Provided Question Source (New, Modified, Bank) Bank Previous 2 NRC Exams (ILT Only) No Memory or Comprehension Fundamental 1:P Cognitive or Analysis Knowledge Level NUREG 1021 Appendix B: Procedure steps and cautions 55.41b 10 55.43b 10CFR55 Content Administrative, normal, abnormal, and emergency operating procedures for the facility.

Justification for LORT questions with KIA values< 3.0 I N/A Time to Complete: 1-2 minutes Point Value: 1 System I D No.: 300000

- PRA: No OCS OPS I LT 13-1 EXAM Page: 65 of 201 13 May 2014

EXAMINATION ANSWER KEY ILT 13-1 NRC Written Exam (RO)

Safety Function(s): 8 181 ILT Category(s) (LORT Only): NIA 0 LORT OCS OPS ILT 13-1 EXAM Page: 66 of 201 13 May 2014

EXAMINATION ANSWER KEY ILT 13~1 NRC Written Exam (RO) 25 ID: 13*1 NRO 25 Points: 1.00 The plant is at rated power with the following DC electrical lineup:

  • Battery Charger MG A is supplying DC Bus A
  • Battery Charger MG B is supplying DC Bus B
  • Battery Charger C1 is supplying DC Bus C Subsequently, VMCC 1B2 trips.

Which of the following indications is correct several seconds following this event?

A. PAIPP #1 indicates tripped B. B BATTERY DC AMPS indicates< 0 (charge)

C. C BATTERY DC AMPS indicates> 0 (discharge)

D. A BATTERY DC AMPS indicates> 0 (discharge)

Answer: D Answer Explanation Knowledge and Ability Reference Information Importance Rating K&A RO SAO 263000 DC Electrical Distribution A3.01 -Ability to monitor automatic operations of the D.C.

3.2 3.3 ELECTRICAL DISTRIBUTION including: Meters, dials, recorders, alarms, and indicating lights Level RO I Tier I 2 I Group I 1 General ABN-51 D-3033 3013 sh. 1 References OCS OPS ILT 13-1 EXAM Page: 67 of 201 13 May 2014

EXAMINATION ANSWER KEY ILT 13~1 NRC Written Exam (RO)

D is Correct. The plant is at power when VMCC 1B2 trips. VMCC 1B2 provides power to the A & B battery MG sets, and PAIPP #2 (among other things). When VMCC trips, both the listed battery chargers trip. The A & B battery amps will indicate> 0 (discharge).

A is Incorrect but plausible. As stated above, PAIPP #2 is lost and PAIPP #1 remains energized.

Explanation B is Incorrect but plausible. Answer B shows normal indications of when the battery charger is supplying the DC Bus B. But the conditions show the charger has tripped.

C is Incorrect but plausible. The chargers for DC Bus C are powered from VMCC 1A2. Thus, with the chargers in service, then DC amps for Battery C will indicate< 0 (charge). The chargers remain unaffected by the loss of VMCC 1B2.

Lesson 2621.828.0.0012, DC Distribution Plan DCD-10445, Given a set of system indications or data, evaluate Learning and interpret them to determine limits, trends and system status.

Objective/

References ILT: None LORT: Open Provided Question Source (New, Modified, Bank) Bank Previous 2 NRC Exams (ILT Only) No Memory or Comprehension Fundamental 2:DR Cognitive or Analysis Knowledge Level NUREG 1021 Appendix B: Describing or recognizing Relationships 55.41b 7 55.43b 10CFR55 Design, components, and functions of control and safety systems, Content including instrumentation, signals, interlocks, failure modes, and automatic and manual features.

Justification for LORT questions with KIA values< 3.0 I N/A Time to Complete: 1*2 minutes Point Value: 1 System ID No.: 263000 PRA: No Safety Function(s): 6 ~ ILT Category(s) (LORT Only): N/A 0 LORT OCS OPS ILT 13-1 EXAM Page: 68 of 201 13 May 2014

EXAMINATION ANSWER KEY ILT 13~1 NRC Written Exam (RO) 26 10: 13*1 NRO 26 Points: 1.00 Given the following plant conditions:

  • The plant is operating at full power.
  • Turbine Building Closed Cooling Water (TBCCW) Pumps 1~1 and 1-2 are operating.
  • TBCCW Pump 1-3 control switch is in Normal after STOP position.
  • TBCCW Pump 1-1 trips.
  • The Control Room Operators note that TBCCW discharge pressure is 76 psig and lowering.

Based on these plant conditions, what is the TBCCW Pump 1-3 response?

TBCCW Pump 1-3 _ _ _ _ _ _ _ _ __

A. does not start.

B. immediately starts.

C. starts after a 10 second time delay.

D. starts after a 60 second time delay.

Answer: C Answer Explanation Knowledge and Ability Reference Information Importance Rating K&A RO SRO 400000 Component Cooling Water K6.04 - Knowledge of the effect that a loss or malfunction 3.0 3.1 of the following will have on the CCWS: Pumps Level RO I Tier I 2 I Group I 1 General RAP-Q1f References OCS OPS ILT 13-1 EXAM Page: 69 of 201 13 May 2014

EXAMINATION ANSWER KEY ILT 13~1 NRC Written Exam (RO)

Cis the correct answer. According to the stated plant conditions the operator must recognize that normal after stop means that pump 1*3 is in standby. Upon receipt of a low pressure signal (79

  1. for 10 seconds), this standby pump will auto start.

A is Incorrect because the pump does start after a time delay.

B is Incorrect because there is a 10 second time delay before the Explanation pump auto starts.

D is Incorrect because of the incorrect time delay which is associated with the time delay for CRD Pump start upon EDG start following a LOCA.

All distractors are plausible if the applicant does not recall the TBCCW pump auto start logic.

Lesson 2621.828.0.0048, TBCCW System Plan TBC-10445, DCD-10445, Given a set of system indications or data, Learning evaluate and interpret them to determine limits, trends and Objective/ system status.

References ILT: None LORT: Open Provided Question Source (New, Modified, Bank) Bank Previous 2 NRC Exams (ILT Only) No Memory or Comprehension Fundamental 1:I Cognitive or Analysis Knowledge Level NUREG 1021 Appendix B: Interlocks, setpoints, or system (singular) response 55.41b 7 55.43b 10CFR55 Design, components, and functions of control and safety systems, Content including instrumentation, signals, interlocks, failure modes, and automatic and manual features.

Justification for LORT questions with KIA values< 3.0 I NIA Time to Complete: 1-2 minutes Point Value: 1 System ID No.: 400000 PRA: No Safety Function(s): 8 ~ ILT Category(s) (LORT Only): N/A 0 LORT OCS OPS ILT 13-1 EXAM Page: 70 of 201 13 May 2014

EXAMINATION ANSWER KEY ILT 13~1 NRC Written Exam (RO) 27 10: 13-1 NRO 27 Points: 1.00 lAW Support Procedure 17, Termination and Prevention of Injection, instead of tripping ALL Condensate Pumps, the procedure requires that one Condensate Pump remain in operation.

Which of the following states the basis for this requirement?

One Condensate Pump will remain running to ...

A. provide cooling to the SJAE condensers.

B. prevent flooding of the main condenser tubes.

C. ensure the condensate pre~filter material is not lost.

D. provide a filtered water supply to the water quality line.

Answer: A Answer Explanation Knowledge and Ability Reference Information Importance Rating K&A RO SRO 256000 Reactor Condensate K1.22 - Knowledge of the physical connections and/or 2.8 2.8 cause- effect relationships between REACTOR CONDENSATE SYSTEM and the following: Offgas system Level RO I Tier I 2 I Group I 2 General EOP Users Guide EMG-SP17 References A is Correct. As provided in the reference, one condensate pump remains running to provide cooling to the SJAE condenser to maintain main condenser vacuum. Answer A is correct.

Explanation All distractors are incorrect but plausible if the applicant doesn't recall the correct basis.

Lesson 2621.845.0.0053 RPV Control with ATWS Plan 3055 Given a copy of the EOP, describe in detail each step or conditional statement, including technical basis, and how to Learning perform each step.

Objective/

OCS OPS ILT 13-1 EXAM Page: 71 of 201 13 May 2014

EXAMINATION ANSWER KEY ILT 13~1 NRC Written Exam (RO)

References ILT: None LORT: Open Provided Question Source (New, Modified, Bank) Bank Previous 2 NRC Exams (ILT Only) No Memory or Comprehension Fundamental 1:B Cognitive or Analysis Knowledge Level NUREG 1021 Appendix B: Bases or purpose 55.41b 10 55.43b 10CFR55 Content Administrative, normal, abnormal, and emergency operating procedures for the facility.

Justification for LORT questions with KIA values< 3.0 I NIA Time to Complete: 1*2 minutes Point Value: 1 System ID No.: 256000 PRA: No Safety Function(s): 2 181 ILT Category(s) (LORT Only): NIA 0 LORT OCS OPS ILT 13-1 EXAM Page: 72 of 201 13 May 2014

EXAMINATION ANSWER KEY ILT 13~1 NRC Written Exam (RO) 28 10: 13*1 NRO 28 Points: 1.00 Which ONE of the following provides power to the RPS~1 Scram Solenoid Valves?

A CIP-3 B. PSP-1 C. PAIP-1 D. VACP-1 Answer: B Answer Explanation Knowledge and Ability Reference Information Importance Rating K&A RO SAO 201001 CAD Hydraulic K2.02- Knowledge of electrical power supplies to the 3.6 3.7 following: Scram valve solenoids Level RO I Tier I 2 J Groupj 2 General GU 3C-733-11*010 References B is Correct. PSP-1 provides power to RPS-1 which provides power to the RPS-1 scram solenoid valves.

Explanation All distractors are Incorrect but plausible since they are all vital instrumentation power supplies. The applicant must recall the correct vital power supply to the RPS-1 scram solenoids.

Lesson 2621.828.0.0037, Reactor Protection System Plan RPS-1 0453, Explain or describe how this system is interrelated Learning with other plant systems.

Objective/

References ILT: None LORT: Open Provided Question Source (New, Modified, Bank) New Previous 2 NRC Exams (ILT Only) No OCS OPS ILT 13-1 EXAM Page: 73 of 201 13 May 2014

EXAMINATION ANSWER KEY ILT 13*1 NRC Written Exam (RO)

Memory or Comprehension Fundamental 1:F Cognitive or Analysis Knowledge Level NUREG 1021 Appendix B: Facts 55.41b 6 55.43b 10CFR55 Content Design, components, and functions of reactivity control mechanisms and instrumentation.

Justification for LORT questions with KIA values< 3.0 I NIA Time to Complete: 1*2 minutes Point Value: 1 System ID No.: 201001 PRA: No Safety Function(s): 1 181 ILT Category(s) (LORT Only): N/A 0 LORT OCSOPS ILT 13-1 EXAM Page: 74 of 201 13 May 2014

EXAMINATION ANSWER KEY ILT 13~1 NRC Written Exam (RO) 29 10: 13*1 NRO 29 Points: 1.00 The plant is at rated power with the following Reactor Building HVAC lined-up as shown on the following page.

The following annunciators then alarmed:

  • FOR TO 460V 1B2 TRIP
  • 1B2 MN BRKR TRIP With NO Operator action, Reactor Building llP will ....

OCS OPS ILT 13-1 EXAM Page: 75 of 201 13 May 2014

EXAMINATION ANSWER KEY ILT 13~1 NRC Written Exam (RO)

OFF SUPPL1r' FA H l Llllllll.ltl fir . lit II~IIIV HJ!.P\'VA-., It RHORf '; TAHIINC,

  • XHAIJ':>I ~AN 1 b Light Energized Light NOT Energized

EXAMINATION ANSWER KEY ILT 13~1 NRC Written Exam (RO)

A. NOT be impacted.

B. go MORE negative.

C. go LESS negative.

D. go from negative to positive.

Answer: B Answer Explanation Knowledge and Ability Reference Information Importance Rating K&A RO SAO 288000 Plant Ventilation K3.05

  • Knowledge of the effect that a loss or malfunction 3.1 3.3 of the PLANT VENTILATION SYSTEMS will have on following: Reactor building pressure: Plant-Specific Level RO I Tier I 2 I GroupJ 2 General 329 References As shown, 2 R8 supply fans and 1 R8 exhaust fan are in service.

In the normal configuration, R8 AP is negative.

The provided annunciator describes the loss of USS 182. The fan power supplied are as follows: SF 1-12 1A21 (remains energized);

SF 1-14 1823 (fed from USS 182 and is de-energized); EF 1-5 1A2 (remains energized). Therefore, one supply fan is de-energized.

Explanation 8 is Correct. With less supply air, the R8 AP, which is already negative, will get more negative.

All distractors are Incorrect but plausible if the applicant does not correctly interpret the status of the R8 supply and exhaust fans following the power loss to USS 182.

Lesson 2621.828.0.0042, Secondary Containment & SGTS Plan SGT-10439, Given the system logic/electrical drawings, describe Learning the system auto initiation signals, setpoints, and expected Objective/ system response including power loss or failed components.

OCS OPS ILT 13-1 EXAM Page: 77 of 201 13 May 2014

EXAMINATION ANSWER KEY ILT 13~1 NRC Written Exam (RO)

References ILT: None LORT: Open Provided Question Source (New, Modified, Bank) Bank Previous 2 NRC Exams (ILT Only) No Memory or Comprehension Fundamental 3:SPK Cognitive or Analysis Knowledge Level NUREG 1021 Appendix B: Solve a Problem using Knowledge and its meaning 55.41b 7 55.43b 10CFR55 Design, components, and functions of control and safety systems, Content including instrumentation, signals, interlocks, failure modes, and automatic and manual features.

Justification for LORT questions with KlA values < 3.0 I N/A Time to Complete: 1-2 minutes Point Value: 1 System ID No.: 288000 PRA: No Safety Function(s): 9 ~ ILT Category(s) (LORT Only): N/A 0 LORT OCS OPS ILT 13-1 EXAM Page: 78 of 201 13 May 2014

EXAMINATION ANSWER KEY ILT 13~1 NRC Written Exam (RO) 30 10: 13*1 NRO 30 Points: 1.00 The plant was at rated power when the following annunciator came into alarm:

  • THRUST BAG WEAR HI Which ONE of the following reflects the status of the Main Turbine and the cause of the alarm?

Turbine Status Cause of Alarm A. Tripped Loss of Bearing Oil Pressure B. Tripped Radial Bearing Misposition

c. NOT Tripped Loss of Bearing Oil Pressure D. NOT Tripped Radial Bearing Misposition Answer: A Answer Explanation Knowledge and Ability Referenc!Jnformation Importance Rating K&A RO SRO 245000 Main Turbine Gen. I Aux.

K4.05 *Knowledge of MAIN TURBINE GENERATOR AND 2.9 3.0 AUXILIARY SYSTEMS design feature(s) and/or interlocks which provide for the following: Turbine protection Level RO I Tier I 2 I Group I 2 General RAP-Q2b References A is Correct. The source of lube oil to the thrust bearing wear detector pressure switches is from bearing oil. A loss of bearing oil pressure will result in this alarm. This is a red TT alarm indicating that a turbine trip has occurred.

Explanation All distractors are Incorrect but plausible because they contain either the incorrect turbine status or cause. The alarm is also caused by axial bearing mis-positioning which is associated with a thrust bearing as opposed to a radial bearing.

OCS OPS ILT 13-1 EXAM Page: 79 of 201 13 May 2014

EXAMINATION ANSWER KEY ILT 13~1 NRC Written Exam (RO)

Lesson 2621.828.0.0050, Turbine and Turbine Auxiliaries Plan 245*10444, Describe the interlock signals and setpoints for the Learning affected system components and expected system response Objective/ including power loss or failed components.

References ILT: None LORT: Open Provided Question Source (New, Modified, Bank) New Previous 2 NRC Exams (ILT Only) No Memory or Comprehension Fundamental 3:PEO Cognitive or Analysis Knowledge Level NUREG 1021 Appendix 8: Predict an Event or Qutcome 55.41b 7 55.43b 10CFR55 Design, components, and functions of control and safety systems, Content including instrumentation, signals, interlocks, failure modes, and automatic and manual features.

Justification for LORT questions with KIA values< 3.0 I N/A Time to Complete: 1*2 minutes Point Value: 1 System ID No.: 245000 PRA: No Safety Function(s): 4 ~ ILT Category(s) (LORT Only): NIA 0 LORT OCS OPS ILT 13-1 EXAM Page: 80 of 201 13 May 2014

EXAMINATION ANSWER KEY ILT 13-1 NRC Written Exam (RO) 31 10: 13-1 NRO 31 Points: 1.00 A plant startup is in progress with a stable heatup rate established. LETDOWN FLOW CONTROLLER FCV-ND22 was open to 20%.

If, without operator action, air pressure to ND22 is significantly increased, which of the following states the impact on RPV water level control and the corrective action to mitigate this impact?

Impact on RPV Water Level Corrective Action A. Lowers Increase makeup to the RPV B. Lowers Close V-16-57, LETDOWN TO RADWASTE C. Rises Limit makeup to the RPV D. Rises Open V-16-60, LETDOWN TO CONDENSER Answer: A Answer Explanation Knowledge and Ability Reference Information Importance Rating K&A RO SRO 204000 RWCU K5.05 - Knowledge of the operational implications of the 2.6 2.6 following concepts as they apply to REACTOR WATER CLEANUP SYSTEM: Flow controllers Level RO I Tier I 2 I Group I 2 General BR 2013 sh 1&6 GE 148F444 201 References OCS OPS ILT 13-1 EXAM Page: 81 of 201 13 May 2014

EXAMINATION ANSWER KEY ILT 13~1 NRC Written Exam (RO)

A is Correct. During a startup with a heatup underway, RPV water level will rise due to thermal expansion. RPV water level control is through ND22 to the condenser. When air is significantly increased to this valve, it opens. As the heatup continues, RPV water level will not rise as expected for the conditions. The only course of action is to increase makeup into the RPV until air pressure is returned to normal. The applicant must have knowledge of the purpose and function of ND-22 (a major system component in the RWCU system) in order to answer this question.

Explanation C & D are Incorrect but plausible since water level lowers, not rises. If the applicant does not recall how air affects Flow control valve ND22 and believes that RPV level will rise, then limiting flow to the RPV or opening letdown valve V-16-16 would be correct answers.

B is Incorrect but plausible. Letdown to radwaste is downstream of ND22, which means that closing the radwaste letdown path will have no flow, and thus no impact on water level.

Lesson 2621.828.0.0039, Reactor Water Cleanup Plan 00819 Learning Objective/

References ILT: None LORT: Open Provided Question Source (New, Modified, Bank) Modified Previous 2 NRC Exams (ILT Only) No Memory or Comprehension Fundamental 3:PEO Cognitive or Analysis Knowledge Level NUREG 1021 Appendix B: Predict an Event or .Outcome 55.41b 5 55.43b Facility operating characteristics during steady state and transient 10CFR55 conditions, including coolant chemistry, causes and effects of Content temperature, pressure and reactivity changes, effects of load changes, and operating limitations and reasons for these operating characteristics.

Justification for LORT questions with KIA values< 3.0 I N/A Time to Complete: 1-2 minutes I Point Value: I 1 OCS OPS ILT 13-1 EXAM Page: 82 of 201 13 May 2014

EXAMINATION ANSWER KEY ILT 13~1 NRC Written Exam (RO)

System ID No.: 204000 PRA: I No Safety Function(s): 2 ~ ILT Category(s) (LORT Only): NIA 0 LORT OCS OPS ILT 13-1 EXAM Page: 83 of 201 13 May 2014

EXAMINATION ANSWER KEY ILT 13-1 NRC Written Exam (RO) 32 10: 13-1 NRO 32 Points: 1.00 The plant was at rated power when the following annunciator alarmed:

  • 1A2 MN BRKR OL TRIP lAW the applicable ABN, which of the following actions is required?

A Confirm B CR HVAC System is in service.

B. Start Reactor Building SUPPLY FAN 1-13.

C. Confirm Standby Gas Treatment System I in service.

D. Start Reactor Building STANDBY EXHAUST FAN 1-6.

Answer: A Answer Explanation Knowledge and Ability Reference Information Importance Rating K&A RO SAO 290003 Control Room HVAC K6.01 - Knowledge of the effect that a loss or malfunction 2.7 2.9 of the following will have on the CONTROL ROOM HVAC :

Electrical power Level RO I Tier I 2 I Group 1 2 General 331.1 330 ABN-45 References A is Correct. The plant was at rated power when indications of the loss of USS 1A2 are present. lAW ABN-45, Loss of USS 1A2, confirming B CR HVAC in service is required. If A CR HVAC were in service, the fan would lose power.

Answers B and D are Incorrect but plausible. When the power loss occurs, RB exhaust fan 1-5 loses power. The ABN says to Explanation secure normal RB ventilation and to start SGTS system II.

Therefore, although power is available to the alternate supply/exhaust fans in answers B and D, the procedure does not require them to be started.

Cis Incorrect but plausible if the applicant does not recognize that Standby Gas Treatment System I fan has lost power and is unavailable.

OCS OPS ILT 13-1 EXAM Page: 84 of 201 13 May 2014

EXAMINATION ANSWER KEY ILT 13*1 NRC Written Exam (RO)

Lesson 2621.828.0.0054, Turbine Building and Mise Ventilation Systems Plan TMV-10444, Describe the interlock signals and setpoints for the Learning affected system components and expected system response Objective/ including power loss or failed components.

References ILT: None LORT: Open Provided Question Source (New, Modified, Bank) Bank Previous 2 NRC Exams (ILT Only) No Memory or Comprehension Fundamental 1:P Cognitive or Analysis Knowledge Level NUREG 1021 Appendix B: Procedure steps and cautions 55.41b 7 55.43b 10CFR55 Design, components, and functions of control and safety systems, Content including instrumentation, signals, interlocks, failure modes, and automatic and manual features.

Justification for LORT questions with KIA values< 3.0 I NIA Time to Complete: 1*2 minutes Point Value: 1 System ID No.: 290003 PRA: No Safety Function(s): 9 ~ ILT Category(s) (LORT Only): N/A 0 LORT OCS OPS ILT 13-1 EXAM Page: 85 of 201 13 May 2014

EXAMINATION ANSWER KEY ILT 13~1 NRC Written Exam (RO) 33 ID: 13*1 NRO 33 Points: 1.00 The Spent Fuel Pool Cooling System was in service with one pump, filter and demineralizer. The flow controller is currently set at 70% open with flow established at 400 GPM when the following occurred:

  • The flow controller failed to 100% open Which of the following states the affect on water level in the Skimmer Surge Tanks AND in the Fuel Pool over the next several seconds?

Skimmer Surge Tanks Level Fuel Pool Level A. Lower Lower B. Higher Higher

c. Lower Higher D. Higher Lower Answer: C Answer Explanation Knowledge and Ability Reference Information Importance Rating K&A RO SRO 233000 Fuel Pool Cooling/Cleanup A1.01 -Ability to predict and/or monitor changes in parameters associated with operating the FUEL POOL 2.6 2.9 COOLING AND CLEAN-UP controls including: Surge tank level Level RO I Tier I 2 I Group I 2 General 237E756 References OCSOPS ILT 13-1 EXAM Page: 86 of 201 13 May 2014

EXAMINATION ANSWER KEY ILT 13*1 NRC Written Exam (RO)

The fuel pool overflows into the skimmer surge tanks. The fuel pool cooling pumps take a suction on the skimmer surge tanks and discharge directly back into the fuel pool. The fuel pool water level and skimmer surge tanks water level is in equilibrium and steady at a system flow rate of 400 gpm. To maintain steady levels, 400 gpm must flow from the fuel pool into the skimmer surge tanks.

Explanation Cis Correct. When the flow controller is opened further, the skimmer surge tank level will drop to accommodate more flow.

As the flow rate rises into the fuel pool, water level there will initially rise, until again equilibrium is achieved.

All distractors are Incorrect but plausible if the candidate does not understand the system and flow path.

Lesson 2621.828.0.0020, Fuel Pool Cooling System Plan FPC-10445, Describe the interlock signals and setpoints for the Learning affected system components and expected system response Objective/ including power loss or failed components.

References ILT: None LORT: Open Provided Question Source (New, Modified, Bank) Bank Previous 2 NRC Exams (ILT Only) No Memory or Comprehension Fundamental 3:PEO Cognitive or Analysis Knowledge Level NUREG 1021 Appendix B: Predict an Event or Qutcome 55.41b 7 55.43b 10CFR55 Design, components, and functions of control and safety systems, Content including instrumentation, signals, interlocks, failure modes, and automatic and manual features.

Justification for LORT questions with KIA values< 3.0 I N/A Time to Complete: 1-2 minutes Point Value: 1 System ID No.: 233000 PRA: No Safety Function(s): 9 18] ILT Category(s) (LORT Only): N/A 0 LORT OCS OPS ILT 13-1 EXAM Page: 87 of 201 13 May 2014

EXAMINATION ANSWER KEY ILT 13~1 NRC Written Exam (RO) 34 10: 13-1 NRO 34 Points: 1.00 Given the following:

  • The reactor is operating at rated power
  • The crew has entered ABN-19, RBCCW Failure Response Which one of the following describes:

(1) when a reactor scram is required by ABN-19, and (2) the most limiting reactor recirculation system component that this action is based on?

A. (1) when one CCW FLOW LO annunciator has been in alarm for> 1 minute (2) recirc pump seals B. (1) when more than one CCW FLOW LO annunciator has been in alarm for> 1 minute (2) recirc pump seals C. (1) when one CCW FLOW LO annunciator has been in alarm for > 1 minute (2) recirc pump motor D. (1) when more than one CCW FLOW LO annunciator has been in alarm for > 1 minute (2) recirc pump motor Answer: B Answer Explanation Knowledge and Ability Reference Information Importance Rating K&A RO SRO 202001 Recirculation System A2.17

  • Ability to (a) predict the impacts of the following on the RECIRCULATION SYSTEM ; and (b) based on those 3.1 3.2 predictions, use procedures to correct, control, or mitigate the consequences of those abnormal conditions or operations: Loss of seal cooling water Level I RO I Tier I 2 I Group I 2 OCS OPS ILT 13-1 EXAM Page: 88 of 201 13 May 2014

EXAMINATION ANSWER KEY ILT 13~1 NRC Written Exam (RO)

General ABN~19 RAP~E7d, E7b References B is Correct. When RPV temperature is> 212F, the mode switch is in STARTUP or RUN, and there are two or more CCW FLOW LO alarms for greater than one minute, ABN-19 directs a reactor scram and trip of all operating recirc pumps. Recirc pumps seals are the limiting component since (1) the seal temperature limits specified in ABN-19 are lower for the seals than for the pump Explanation motors and (2) high seal temperatures can cause seal failure, which is of higher consequence than high motor bearing and/or winding temperatures.

All distractors are Incorrect but plausible if the applicant does not recall the correct procedural action for this event or does not know which component is most limiting for the event.

Lesson 2621.828.0.0035, RBCCW System Plan RBC-1 0450, Describe and interpret procedure sections and steps Learning for plant emergency or off-normal conditions that involve this Objective/ system including personnel allocation and equipment operation in accordance with applicable ABN, EOP and EOP support procedures, and EP Procedures.

References ILT: None LORT: Open Provided Question Source (New, Modified, Bank) Bank Previous 2 NRC Exams (ILT Only) No Memory or Comprehension Fundamental 1:P Cognitive or Analysis Knowledge Level NUREG 1021 Appendix B: Procedure steps and cautions 55.41b 10 55.43b 10CFR55 Content Administrative, normal, abnormal, and emergency operating procedures for the facility.

Justification for LORT questions with KIA values< 3.0 N/A Time to Complete: 1*2 minutes Point Value: 1 System ID No.: 202001 PRA: No Safety Function(s): 1 1ZJ ILT Category(s) (LORT Only): N/A 0 LORT OCS OPS ILT 13-1 EXAM Page: 89 of 201 13 May 2014

EXAMINATION ANSWER KEY ILT 13~1 NRC Written Exam (RO) 35 10: 13-1 NRO 35 Points: 1.00 The plant is at rated power. An event then occurred and the crew observed the following annunciators in the alarm condition:

  • OFF GAS PRESS HI
  • OFF GAS DISCH VLV OPEN
  • OFF GAS FLTR llP HI
  • OFF GAS ISOL ACT I
  • OFF GAS ISOL ACT II Which of the annunciator alarms listed below is the NEXT to alarm? (assume NO Operator actions)

A. STACK EFFLUENT HI B. SJAE STM PRESS HI/LO C. COND VAC LO 25 INCHES D. SCRAM CONTACTOR OPEN Answer: C Answer Explanation Knowledge and Ability Reference Information Importance Rating K&A RO SAO 271 000 Off-gas A3.05 - Ability to monitor automatic operations of the 2.9 2.9 OFFGAS SYSTEM including: System indicating lights and alarms Level RO I Tier I 2 I Group I 2 General RAP-10F1b RAP-10F1a RAP-10F3a References RAP-10F3b RAP-10F2a RAP-10F1a OCSOPS ILT 13-1 EXAM Page: 90 of 201 13 May 2014

EXAMINATION ANSWER KEY ILT 13~1 NRC Written Exam (RO)

C is Correct. The plant is at rated power when an over pressure condition is sensed in the offgas system. This will result in closure of the 6 offgas outlet isolation valves from the main condenser to the offgas system. As a result, condenser vacuum will degrade and the first of the listed annunciators to alarm is the COND VAC LO 251NCHES annunciator.

A is Incorrect but plausible. Since the offgas discharge valve, V-7-31, has opened, it could be assumed that stack effluent will Explanation rise. But since all the condenser offgas flow is already isolated, stack gas effluent will not rise.

B is Incorrect but plausible. Although offgas has isolated, steam to the SJAEs remains unaffected.

Dis Incorrect but plausible. When the condenser vacuum lowers to the turbine trip setpoint, the reactor will scram and the SCRAM CONTACTOR OPEN annunciator will alarm. But this is not the first of the listed annunciators to alarm.

Lesson 2621.828.0.0002, Air Extraction and Off Gas Plan AEG-00666, Given auto TRIP signals and setpoints, analyze Learning cause(s) and describe system response, to include Air Extraction Objective/ and Off-Gas system components.

References ILT: None LORT: Open Provided Question Source (New, Modified, Bank) Bank Previous 2 NRC Exams (ILT Only) No Memory or Comprehension Fundamental 3:PEO Cognitive or Analysis Knowledge Level NUREG 1021 Appendix B: Predict an Event or Outcome 55.41b 7 55.43b 10CFR55 Design, components, and functions of control and safety systems, Content including instrumentation, signals, interlocks, failure modes, and automatic and manual features.

Justification for LORT questions with KIA values< 3.0 I NIA Time to Complete: 1-2 minutes Point Value: 1 System ID No.: 271000 PRA: No Safety Function(s): 9 181 ILT OCS OPS ILT 13-1 EXAM Page: 91 of 201 13 May 2014

EXAMINATION ANSWER KEY ILT 13~1 NRC Written Exam (RO)

I Category(s) (LORT Only): I N/A ID LORT OCS OPS ILT 13-1 EXAM Page: 92 of 201 13 May 2014

EXAMINATION ANSWER KEY ILT 13~1 NRC Written Exam (RO) 36 10: 13-1 NRO 36 Points: 1.00 The plant is at 95% power with reactivity manipulations in-progress. There are no impediments to control rod maneuvers. The Operator places and holds the ROD CONTROL switch to the ROD OUT NOTCH position for a control rod at position 30.

Which of the following states the correct plant response?

A The green INSERT light will be ON for 5 seconds.

B. The red WITHDRAW light will be ON for 1.5 seconds.

C. The amber SETTLE light will be ON for 5 seconds ONLY after the switch is released.

D. The withdraw stabilizing valve OPENS to pass 2 gpm during the withdraw of the control rod.

Answer: B Answer Explanation Knowledge and Ability Reference Information Importance Rating K&A RO SRO 201002 RMCS A4.01 - Ability to manually operate and/or monitor in the 3.5 3.4 control room: Rod movement control switch Level RO I Tier I 2 J Group I 2 General ABN-6 237E478 302.2 References B is Correct and A is Incorrect. When a single notch withdraw is made, the green insert light is on for 1 second, and the red withdraw light will be on for 1.5 seconds.

Cis Incorrect but plausible. The amber light will come on even if Explanation the rod control switch is held in the rod out notch position.

D is Incorrect but plausible. With no rod motion, 2 stabilizer valves are open to pass 2 gpm (withdraw) and 4 gpm (insert).

When a withdraw occurs, the withdraw valve closes, stopping its normal 2 gpm flow.

OCS OPS ILT 13-1 EXAM Page: 93 of 201 13 May 2014

EXAMINATION ANSWER KEY ILT 13~1 NRC Written Exam (RO)

Lesson 2621.828.0.0050, Turbine and Turbine Auxiliaries Plan 245-10447, Given normal operating procedures and documents Learning for the system, describe or interpret the procedural steps.

Obiective/

References ILT: None LORT: Open Provided Question Source (New, Modified, Bank) Bank Previous 2 NRC Exams (ILT Only) No Memory or Comprehension Fundamental 3:PEO Cognitive or Analysis Knowledge Level NUREG 1021 Appendix B: Predict an Event or .Outcome 55.41b 10 55.43b 10CFR55 Content Administrative, normal, abnormal, and emergency operating procedures for the facility.

Justification for LORT questions with KIA values< 3.0 I N/A Time to Complete: 1-2 minutes Point Value: 1 System ID No.: 201002 PRA: No Safety Function(s): 1 IZI ILT Category(s) (LORT Only): N/A 0 LORT OCS OPS ILT 13-1 EXAM Page: 94 of 201 13 May 2014

EXAMINATION ANSWER KEY ILT 13~1 NRC Written Exam (RO) 37 10: 13*1 NRO 37 Points: 1.00 The plant is shutdown for 1R25 with fuel shuffling in progress. If RPV water level were to lower, which of the following would be the FIRST RPV water level listed to violate a Tech Spec Safety Lim it?

A. 60 in TAF B. 54 in TAF C. 48 in TAF D. 36 in TAF Answer: B Answer Explanation Knowledge and Ability Reference Information Importance Rating K&A RO SRO 290002 Reactor Vessel Internals 2.2.22

  • Equipment Control: Knowledge of limiting 4.0 4.7 conditions for operations and safety limits Level RO I Tier I 2 I Group I 2 General TS 2.1 References OCS OPS ILT 13-1 EXAM Page: 95 of 201 13 May 2014

EXAMINATION ANSWER KEY ILT 13*1 NRC Written Exam (RO)

B is Correct. lAW TS 2.1.0, During all modes of operation with irradiated fuel in the reactor vessel, the water level shall not be less than 56 11 TAF. The question states that the plant is in 1R23 implying that irradiated fuel is in the vessel and RPV level is 54 11 TAF {which is< 56 11 {4'8") and corresponds to the TS value for Lo-Lo-Lo RPV water level). The question states that fuel shuffling is in progress indicating the RPV is flooded up giving the student an initial RPV water level before level starts to lower. 54" TAF will be the FIRST RPV level in the question where this Safety Limit has been violated.

A is Incorrect. The value of 60" TAF corresponds to a value Explanation below the instrument setpoint of Lo-Lo-Lo RPV water level {61" TAF is actual setpoint value). This distractor is plausible if the student believes that the Safety Limit is the instrument setpoint and not the TS value of Lo-Lo-Lo RPV water level.

C and Dare Incorrect. These water levels, 48" and 36" do violate the Safety Limit however they would not be the FIRST water level reached. TS 2.1.0 lists the safety limit at< 4'8" TAF making the 48" value plausible. If the student believes that RPV level must be< 48 11 to violate the safety limit, the 36 11 value is close to 48 11 and would be the only value< 48 11 making this choice plausible.

NOTE: Question meets safety limit portion of the KIA.

Lesson 2621.850.0.0090, Overview/Highlights of Technical Specifications Plan TSX-1658, State the requirements associated with given areas of technical specifications {safety) limits, LSSS, etc.

Learning Objective/

References ILT: None LORT: Open Provided Question Source {New, Modified, Bank) Bank Previous 2 NRC Exams {ILT Only) No Memory or Comprehension Fundamental 1:F Cognitive Knowledge or Analysis Level ~------~----------------------------------------~

NUREG 1021 Appendix B: Facts OCS OPS ILT 13-1 EXAM Page: 96 of 201 13 May 2014

EXAMINATION ANSWER KEY ILT 13~1 NRC Written Exam (RO) 10CFR55 55.41b I 3 I 55.43b I

Content Mechanical components and design features of the reactor primary system.

Justification for LORT questions with KIA values< 3.0 I NIA Time to Complete: 1*2 minutes Point Value: 1 System ID No.: 290002 PRA: No Safety Function(s): 5 ~ ILT Category(s) (LORT Only): NIA 0 LORT OCS OPS ILT 13-1 EXAM Page: 97 of 201 13 May 2014

EXAMINATION ANSWER KEY ILT 13-1 NRC Written Exam (RO) 38 ID: 13-1 NRO 38 Points: 1.00 The plant was at rated power when the following event occurred:

  • Recirculation Pump D AIRFAIL light is ON One minute later, an event occurred which required the Operator to reduce recirculation flow to minimum.

Which of the following shows the indications for MG-SET SPEED CONTROLLER D AFTER the flow reduction as compared to BEFORE the flow reduction? (Assume the D MG Set Speed Controller remained in AUTO)

V DEMAND SPEED Y DEVIATION A. No change No change B. Less Greater C. Less Less D. Less No change Answer: B Answer Explanation Knowledge and Ability Reference Information Importance Rating K&A RO SRO 202002 Recirculation Flow Control A3.03 - Ability to monitor automatic operations of the 3.1 3.0 RECIRCULATION FLOW CONTROL SYSTEM including:

Scoop tube operation: BWR-2,3,4 Level RO I Tier 1 2 J Group I 2 General 301.2 3E*627-41-001 sh. 2 References OCS OPS ILT 13-1 EXAM Page: 98 of 201 13 May 2014

EXAMINATION ANSWER KEY ILT 13~1 NRC Written Exam (RO)

B is Correct. A loss of air to recirculation pump D has occurred in the question stem. When this occurs, the scoop tube will lockup in its current position. With the D speed controller in auto and the master flow controller reduced, the demand signal will go down, but the actual speed will remain the same, and the Explanation deviation between the demand/actual will rise. Answer B is correct.

All distractors are Incorrect but plausible if the applicant does not interpret conditions in the stem correctly or does not recall what happens when a loss of air occurs.

Lesson 2621.828.0.0040, Recirc Flow Control Plan RFC-00214, Explain what causes a scoop tube lockup and Learning mechanically what occurs, including how to reset.

Objective/

References ILT: None LORT: Open Provided Question Source (New, Modified, Bank) New Previous 2 NRC Exams (ILT Only) No Memory or Comprehension Fundamental 3:SPK Cognitive or Analysis Knowledge Level NUREG 1021 Appendix 8: Solve a Problem using Knowledge and its meaning 55.41b 7 55.43b 10CFR55 Design, components, and functions of control and safety systems, Content including instrumentation, signals, interlocks, failure modes, and automatic and manual features.

Justification for LORT questions with KIA values< 3.0 I N/A Time to Complete: 1-2 minutes Point Value: 1 System ID No.: 202002 PRA: No Safety Function(s): 1 ~ ILT Category(s) (LORT Only): N/A 0 LORT OCS OPS ILT 13-1 EXAM Page: 99 of 201 13 May 2014

EXAMINATION ANSWER KEY ILT 13~1 NRC Written Exam (RO) 39 ID: 13*1 NRO 39 Points: 1.00 The plant is at rated power with the following conditions:

  • Main Generator volts: 24KV
  • Main Generator MW: 650 MW
  • Main Generator VARS: 100 MVARS
  • Hydrogen Pressure: 45 psig A grid disturbance results in steadily LOWERING grid voltage. The Main Generator voltage regulator responds as designed by attempting to raise Main Generator terminal voltage.

With NO operator action, this transient could result in _ _ _ _ _ __

A overheating the Main Generator rotor windings B. overheating the Main Generator stator windings C. exceeding the Generator Under Excitation Limit D. Generator Lockout due to reverse power relay trip Answer: B Answer Explanation Knowledge and Ability Reference Information Importance Rating K&A RO SRO 700000 Generator Voltage and Electric Grid Disturbances AK1.02

  • Knowledge of the operational implications of the following concepts as they apply to GENERATOR 3.3 3.4 VOLTAGE AND ELECTRIC GRID DISTURBANCES and the following: Over-excitation.

Level RO I Tier I 1 I Group I 1 General 336.1 References OCS OPS ILT 13-1 EXAM Page: 100 of 201 13 May 2014

EXAMINATION ANSWER KEY ILT 13~1 NRC Written Exam (RO)

B is Correct. The given conditions (lowering grid voltage) will cause the generator automatic voltage regulator to attempt to raise grid voltage, causing generator over-excitation which causes the generator to pick up additional VARS (i.e., move up on the Generator Capability Curve). Without operator action, this would result in exceeding the Generator Capability Curve (B-C) for 45 psig hydrogen pressure. Per procedure 336.1 Attachment 336.1-1, curve B-C is limited by armature (stator) heating.

A is Incorrect but plausible since this would be true if curve A-B was the limiting factor. Also plausible if the applicant was Explanation confused between field, armature, rotor and stator.

C is Incorrect but plausible since this would be true if grid voltage was rising, resulting in lowering VARS on the main generator (i.e., move down on the Generator Capability Curve).

D is Incorrect. A reverse power trip occurs when real load (MW) is reduced to the point where the grid supplies the generator.

The given conditions would not result in lowering MW, especially to the point of reverse power. Plausible if the applicant was confused on real vs. reactive load sharing between generators.

Lesson 2621.828.0.0025, Main Generator Plan GEN-10445, Given a set of system indications or data, evaluate Learning and interpret them to determine limits, trends and system status.

Objective/

References ILT: Att. 336.1*1 LORT: Open Provided Question Source (New, Modified, Bank) Bank Previous 2 NRC Exams (ILT Only) Yes Memory or Comprehension Fundamental 3:SPK Cognitive or Analysis Knowledge Level NUREG 1021 Appendix B: .S.olve a Problem using Knowledge and its meaning 55.41b 7 55.43b 10CFR55 Design, components, and functions of control and safety systems, Content including instrumentation, signals, iraterlocks, failure modes, and automatic and manual features.

Justification for LORT questions with KIA values< 3.0 I N/A Time to Complete: 1-2 minutes I Point Value: I 1 OCS OPS ILT 13-1 EXAM Page: 101 of 201 13 May 2014

EXAMINATION ANSWER KEY ILT 13~1 NRC Written Exam (RO)

System ID No.: 700000 PRA: I No Safety Function(s): 6 ~ ILT Category(s) (LORT Only): N/A 0 LORT OCS OPS ILT 13-1 EXAM Page: 102 of 201 13 May 2014

EXAMINATION ANSWER KEY ILT 13~1 NRC Written Exam (RO) 40 ID: 13*1 NRO 40 Points: 1.00 The plant was at rated power when an ATWS occurred.

The Unit Supervisor has directed that RPV injection be terminated and prevented, lAW Support Procedure 17, Termination and Prevention of Injection, and that RPV water level be lowered to 30".

Which of the following states the Core Spray Main Pump and Feedwater/Condensate Pump configuration AFTER implementation of Support Procedure 17, and the basis for lowering RPV water level?

Pump Configuration Basis for 30" RPV Water Level A. All Core Spray Main Pumps in The lowered water level will PTL and ALL lower reactor power from Feedwater/Condensate increased voids Pumps OFF B. All Core Spray Main Pumps in The lowered water level will PTL and ALL reduce subcooling to minimize Feedwater/Condensate power oscillations Pumps OFF EXCEPT one Condensate Pump ON C. ALL Core Spray Main Pumps The lowered water level will in PTL; All lower reactor power from Feedwater/Condensate increased voids Pumps ON with MFRVs CLOSED D. All Core Spray Main Pumps The lowered water level will ON with the Parallel Isolation reduce subcooling to minimize Valves CLOSED; All power oscillations Feedwater/Condensate Pumps OFF Answer: B Answer Explanation Knowledge and Ability Reference Information OCS OPS ILT 13-1 EXAM Page: 103 of 201 13 May 2014

EXAMINATION ANSWER KEY ILT 13*1 NRC Written Exam (RO)

Importance Rating K&A RO SAO 295031 Reactor Low Water Level/ 2 EK1.03 - Knowledge of the operational implications of the 3.7 4.1 following concepts as they apply to REACTOR LOW WATER LEVEL: Water level effects on reactor power Level RO I Tier I 1 I Group I 1 General EMG-SP17 EOP Users Guide References B is Correct. The question stem describes a high power ATWS (power> 2%) and the requirement to terminate/prevent injection and to lower RPV water level to 30" lAW the ATWS EOP. lAW SP17, all core spray main pumps are placed in pull-to-lock (PTL),

and all feedwater/condensate pumps except one are secured.

This one pump is required to supply the SJAE condensers to Explanation maintain main condenser vacuum. lAW the EOP Users Guide, RPV water level is lowered to minimize feedwater subcooling to prevent large power oscillations that could cause fuel damage.

The other answers are plausible but list the incorrect pump configuration or incorrect basis.

Lesson 2621.845.0.001 B, RPV Control with ATWS Plan EWA-03055 Given a copy of RPV Control, describe in detail each Learning step or conditional statement, including technical basis, and how Objective/ to perform each step as required.

References ILT: None LORT: Open Provided Question Source (New, Modified, Bank) Bank Previous 2 NRC Exams (ILT Only) No Memory or Comprehension Fundamental 1:P&B Cognitive or Analysis Knowledge Level NUREG 1021 Appendix B: Procedure steps and cautions & Bases or purpose 55.41b 10 55.43b 10CFR55 Content Administrative, normal, abnormal, and emergency operating procedures for the facility.

Justification for LORT questions with KIA values< 3.0 N/A Time to Complete: 1-2 minutes I Point Value: I 1 OCS OPS ILT 13-1 EXAM Page: 104 of 201 13 May 2014

EXAMINATION ANSWER KEY ILT 13~1 NRC Written Exam (RO)

System ID No.: 295031 PRA: I No Safety Function(s): 2 ~ ILT Category(s) (LORT Only): N/A 0 LORT OCS OPS ILT 13-1 EXAM Page: 105 of 201 13 May 2014

EXAMINATION ANSWER KEY ILT 13~1 NRC Written Exam (RO) 41 10: 13*1 NRO 41 Points: 1.00 A fire was detected in the 4160V 1C Room. Portable extiguishers did not extiguish the fire. The fire suppression system was manually activated.

lAW Support Procedure TB3A (FSP~TB3A), Fire Support Procedure for TB, 4160 SWGR AM 1C, caution must be exercised prior to entry into the 4160V SWGR Room due to the effects of ....

A. C02 in the atmosphere.

B. halon in the atmosphere.

C. water spray on energized circuits.

D. water spray and flooding of other areas.

Answer: A Answer Explanation Knowledge and Ability Reference Information Importance Rating K&A RO SRO 600000 Plant Fire On-site I 8 AK1.02 - Knowledge of the operational implications of the 2.9 3.1 following concepts as they apply to Plant Fire On Site:

Fire Fighting Level RO I Tier I 1 I Group I 1 General FSP-TB3A ABN-29 References A is Correct. If a fire is detected in either 4160 vault CorD, the ABN directs that if portable extinguishers cannot extinguish the fire, then the low pressure C0 2 system shall be activated. FSP-TB3A directs that if the system was activated, then a SCBA is required prior to entry.

Explanation All distractors are Incorrect but plausible as it lists hazards related to other suppression methods at the station. The applicant may also not recall which fire suppression system is activated.

OCS OPS ILT 13-1 EXAM Page: 106 of 201 13 May 2014

EXAMINATION ANSWER KEY ILT 13~1 NRC Written Exam (RO)

Lesson 2621.828.0.0019, Fire Protection System Plan FSP-1 0450, Describe and interpret procedure sections and steps Learning for plant emergency or off-normal conditions that involve this Objective/ system including personnel allocation and equipment operation lAW applicable ABN, EOP & EOP support procedures and EP procedures.

References ILT: None LORT: Open Provided Question Source (New, Modified, Bank) Bank Previous 2 NRC Exams (ILT Only) No Memory or Comprehension Fundamental 1:P Cognitive or Analysis Knowledge Level NUREG 1021 Appendix B: Procedure steps and cautions 55.41b 10 55.43b 10CFR55 Content Administrative, normal, abnormal, and emergency operating procedures for the facility.

Justification for LORT questions with KIA values< 3.0 I NIA Time to Complete: 1*2 minutes Point Value: 1 System ID No.: 600000 PRA: No Safety Function(s): 8 ~ ILT Category(s) (LORT Only): N/A 0 LORT OCS OPS ILT 13-1 EXAM Page: 107 of 201 13 May 2014

EXAMINATION ANSWER KEY ILT 13-1 NRC Written Exam (RO) 42 10: 13*1 NRO 42 Points: 1.00 Which of the following indications are available on the Remote Shutdown Panel?

A. RPV water level (YARWAY) and reactor power B. RPV water level (GEMAC) and RPV pressure C. RPV water level (FUEL ZONE) and RPV pressure D. RPV water level (FUEL ZONE) and Isolation Condenser A shell water level Answer: C Answer Explanation Knowledge and Ability Reference Information Importance Rating K&A RO SRO 295016 Control Room Abandonment /7 AK2.01 - Knowledge of the interrelations between 4.4 4.5 CONTROL ROOM ABANDONMENT and the following:

Remote shutdown panel: Plant-Specific Level RO I Tier I 1 I Group I 1 General 346 References C is Correct. The Remote Shutdown Panel has the following primary coolant indications: RPV water level (fuel zone), RPV pressure, and Isolation Condenser B shell water level.

Explanation Other distractors are Incorrect but plausible if the applicant confuses what indications are available on the panel with indications on other Local Shutdown Panels.

Lesson 2621.828.0.0064, Alternate Shutdown Facility Plan ASF-1 0446, Identify and explain system operating Learning controls/indications under all plant operating conditions.

Objective/

References ILT: None LORT: Open Provided Question Source (New, Modified, Bank) Bank Previous 2 NRC Exams (ILT Only) No OCS OPS ILT 13-1 EXAM Page: 108 of 201 13 May 2014

EXAMINATION ANSWER KEY ILT 13~1 NRC Written Exam (RO)

Memory or Comprehension Fundamental 1:F Cognitive or Analysis Knowledge Level NUREG 1021 Appendix 8: Facts 55.41b 7 55.43b 10CFR55 Design, components, and functions of control and safety systems, Content including instrumentation, signals, interlocks, failure modes, and automatic and manual features.

Justification for LORT questions with KIA values< 3.0 I NIA Time to Complete: 1*2 minutes Point Value: 1 System I D No.: 295106 PRA: No Safety Function(s): 7 IZJ ILT Category(s) (LORT Only): N/A 0 LORT OCS OPS ILT 13-1 EXAM Page: 109 of 201 13 May 2014

EXAMINATION ANSWER KEY ILT 13~1 NRC Written Exam (RO) 43 10: 13-1 NRO 43 Points: 1.00 The plant was at 83% power, with the following conditions:

  • Recirculation Pump E is OFF, with its control switch in PTL An event occurred which required the Operator to complete a rapid power reduction to 65% power with recirculation flow.

When power was stable, the following annunciators alarmed:

  • MN BRKR 1A TRIP
  • MN BRKR 1A LKOUT TRIP
  • BUS 1A UV Which ONE of the following actions are required?

A. Manually insert CRAM rods due to reduced core flow.

B. Manually scram the reactor due to reduced core flow.

C. Manually scram the reactor due to reduced feedwater flow.

D. Manually reduce recirculation flow due to reduced feedwater flow.

Answer: B Answer Explanation Knowledge and Ability Reference Information Importance Rating K&A RO SRO 295001 Partial or Complete Loss of Forced Core Flow Circulation /1 & 4 AK2.01 -Knowledge of the interrelations between 3.6 3.7 PARTIAL OR COMPLETE LOSS OF FORCED CORE FLOW CIRCULATION and the following: Recirculation system Level RO I Tier I 1 I Group I 1 General ABN-2 ABN-17 References OCS OPS ILT 13-1 EXAM Page: 110 of 201 13 May 2014

EXAMINATION ANSWER KEY ILT 13~1 NRC Written Exam (RO)

B is Correct. The question stem describes a loss of 4160 VAC Bus 1A. Recirculation pumps A, C, and E (presently OFF), and feedwater/condensate pumps A are powered from this Bus. When power is lost to Bus 1A, one feedwater pump (A), one condensate pump (A), and two recirculation pumps (A and C) are lost. ABN-2, Recirculation System Failures, requires a manual scram if <3 recirculation pumps are running OR if multiple recirculation pumps trip. Answer B is correct.

Explanation ABN-17 requires a scram if multiple feedwater or multiple condensate pumps trip. Also, since power is within the capacity of just two FW pumps, no power reduction would be required for the loss of one FW pump. Two FW pumps can supply flow up to 70% power.

All distractors are Incorrect but plausible if the applicant does not recognize either what pumps were lost or does not recall the actions required by ABN-17.

Lesson 2621.828.0.0038, Reactor Recirculation System Plan RRS-1 0450, Describe and interpret procedure sections and steps for plant emergency or off-normal conditions that involve this Learning system including personnel allocation and equipment operation Objective/ lAW applicable ABNs, SDRPs, EOP & EOP Support Procedures and EP Procedures.

References ILT: None LORT: Open Provided Question Source (New, Modified, Bank) Bank Previous 2 NRC Exams (ILT Only) No Memory or Comprehension Fundamental 2:RI Cognitive or Analysis Knowledge Level NUREG 1021 Appendix B: Recognizing interaction between systems (plural), including consequences and implications 55.41b 7 55.43b 10CFR55 Design, components, and functions of control and safety systems, Content including instrumentation, signals, interlocks, failure modes, and automatic and manual features.

Justification for LORT questions with KIA values< 3.0 I N/A Time to Complete: 1*2 minutes Point Value: 1 System ID No.: 295001 PRA: No Safety Function(s): 1&4 IZj ILT OCS OPS ILT 13-1 EXAM Page: 111 of 201 13 May 2014

EXAMINATION ANSWER KEY ILT 13-1 NRC Written Exam (RO)

I Category(s) (LORT Only): I N/A ID LORT OCS OPS ILT 13-1 EXAM Page:112of201 13May2014

EXAMINATION ANSWER KEY ILT 13~1 NRC Written Exam (RO) 44 10: 13*1 NRO 44 Points: 1.00 The plant was at rated power with the following 125 VDC battery chargers in-service:

  • Battery Charger MG Set A
  • AlB static Charger
  • Static Charger C 1 The following annunciator then alarmed:
  • 1A2 MN BRKR TRIP Which of the following states the impact on the DC Distribution System?

A. B Battery has lost its charger and shall be declared inoperable immediately.

B. C Battery has lost its charger and shall be declared inoperable immediately.

C. A Battery has lost its charger but it can be aligned to the Battery Charger MG B for charging.

D. C Battery has lost its charger but it can be aligned to the standby Static Charger C2 for charging.

Answer: B Answer Explanation Knowledge and Ability Reference Information Importance Rating K&A RO SRO 295004 Partial or Total Loss of DC Pwr /6 AK2.01 *Knowledge of the interrelations between 3.1 3.1 PARTIAL OR COMPLETE LOSS OF D.C. POWER and the following: Battery charger Level RO I Tier I 1 I Group I 1 General 3033 ABN-45 338 References 3028 OCS OPS ILT 13-1 EXAM Page: 113 of 201 13 May 2014

EXAMINATION ANSWER KEY ILT 13~1 NRC Written Exam (RO)

The plant is at rated power with the following lineup: DC A is being charged by the Battery Charger MG Set A, DC B is being charged by the AlB static Charger, and DCC is being charged by Static Charger C1.

The provided alarm shows a loss of power to USS 1A2, which feeds VMCC 1A2. VMCC 1A2 supplies the Static Chargers C1 &

C2. Battery Charger MG Set A is supplied from VMCC 1B2 (through USS 1B2) and the AlB Static Charger is supplied by MCC 1B21 (from USS 1B2) and both are unaffected by the event.

Explanation B is Correct. lAW ABN-45, C Battery is to be considered inoperable immediately with the loss of the charger.

Answer A is Incorrect but plausible if the applicant does not recognize that B Battery still has its charger.

Answer C is Incorrect but plausible if the applicant does not recognize that A Battery is not impacted.

Answer D is Incorrect but plausible if the applicant does not recognize that both the C1 and C2 static chargers have lost power.

Lesson 2621.828.0.0016, Electrical Distribution System Plan EDS-10453, Explain or describe how this system is interrelated Learning with other plant systems.

Objective/

References ILT: None LORT: Open Provided Question Source (New, Modified, Bank) Bank Previous 2 NRC Exams (ILT Only) No Memory or -

Comprehension Fundamental 3:SPK Cognitive or Analysis Knowledge Level NUREG 1021 Appendix B: .Solve a Problem using .Knowledge and its meaning 55.41b 7 55.43b 10CFR55 Design, components, and functions of control and safety systems, Content including instrumentation, signals, interlocks, failure modes, and automatic and manual features.

Justification for LORT questions with KIA values< 3.0 I N/A OCS OPS ILT 13-1 EXAM Page: 114 of 201 13 May 2014

EXAMINATION ANSWER KEY ILT 13~1 NRC Written Exam (RO)

Time to Complete: 1-2 minutes Point Value: 1 System ID No.: 295004 PRA: No Safety Function(s): 6 ~ ILT Category(s) (LORT Only): N/A 0 LORT OCS OPS ILT 13-1 EXAM Page: 115 of 201 13 May 2014

EXAMINATION ANSWER KEY ILT 13-1 NRC Written Exam (RO) 45 10: 13*1 NRO 45 Points: 1.00 The plant was shutdown with fuel shuffling in progress on the refuel floor.

The fuel hoist was loaded with a new fuel bundle in the spent fuel pool and was to be inserted into the second core quadrant. The following annunciator then alarmed while the bridge was maneuvering in the Spent Fuel Pool:

  • ROD DRIFT Which of the following states the effect on the refuel bridge and the basis?

Refuel Bridge Effect Basis A. The refuel bridge will be Prevents a large reactivity prevented from being moved addition into the core anywhere over the core B. The refuel bridge will Prevents a large reactivity experience a bridge fault and addition into the core ALL refuel bridge motion will be halted

c. The refuel bridge will be Prevents damage to the fuel prevented from being moved bundle and the drifting control anywhere over the core rod if both were inserted at the same time in the same fuel cell D. The refuel bridge will Prevents damage to the fuel experience a bridge fault and bundle and the drifting control ALL refuel bridge motion will rod if both were inserted at the be halted same time in the same fuel cell Answer: A Answer Explanation Knowledge and Ability Reference Information Importance Rating K&A RO SAO 295023 Refueling Ace Cooling Mode I 8 AK3.02 - Knowledge of the reasons for the following 3.4 3.8 responses as they apply to REFUELING ACCIDENTS :

Interlocks associated with fuel handling equipment Level I RO I Tier I 1 I Group I 1 OCS OPS ILT 13-1 EXAM Page: 116 of 201 13 May 2014

EXAMINATION ANSWER KEY ILT 13~1 NRC Written Exam (RO)

General UFSAR Table 7. 7-1 UFSAR 9.1.4.3 References A is Correct. The question describes a fuel-loaded refuel platform, currently in the fuel pool, to be moved to the second core quadrant, while a Rod Drift alarm annunciates. The annunciator shows that a control rod is no longer at position 00.

When the bridge is moved toward the core area, the refuel interlock will auto stop the bridge before the core area is reached. This is to prevent a large reactivity addition to the core (largest reactivity addition would be in the cell with the control rod is drifting).

Explanation B is Incorrect but plausible. As stated, bridge motion towards the core is halted, but other bridge motions are not impacted and there is no bridge fault.

C & D are Incorrect but plausible. It is possible that inserting a fuel bundle into the core location with the drifting control rod, were it selected and being manually driven in by the operator (which is the expected control room action), that damage to the bundle or control rod blade could occur. None the less, it is not the basis for the refuel interlock.

Lesson 2621.812.0.0003, Refueling Plan RFL-02391, Demonstrate understanding of the interlocks and rod Learning blocks associated with the following refueling platform Objective/ components, including their purpose:

1. Bridge & Trolley
2. Main Hoist
3. Auxiliary Hoist References ILT: None LORT: Open Provided Question Source (New, Modified, Bank) Bank Previous 2 NRC Exams (ILT Only) No Memory or Comprehension Fundamental 1:B Cognitive or Analysis Knowledge Level NUREG 1021 Appendix B: Bases or purpose OCS OPS ILT 13-1 EXAM Page: 117 of 201 13 May 2014

EXAMINATION ANSWER KEY ILT 13~1 NRC Written Exam (RO) 10CFR55 55.41b I 7 I 55.43b I

Design, components, and functions of control and safety systems, Content including instrumentation, signals, interlocks, failure modes, and automatic and manual features.

Justification for LORT questions with KIA values< 3.0 I N/A Time to Complete: 1*2 minutes Point Value: 1 System ID No.: 295023 PRA: No Safety Function(s): 8 [81 ILT Category(s) (LORT Only): NIA 0 LORT OCS OPS ILT 13-1 EXAM Page: 118 of 201 13 May 2014

EXAMINATION ANSWER KEY ILT 13~1 NRC Written Exam (RO) 46 10: 13*1 NRO 46 Points: 1.00 The plant is at 25% power when an event required entry into ABN~ 10, Turbine Generator Trip.

With the above conditions, complete the statements below.

The IMMEDIATE OPERATOR ACTIONS required by ABN-10 are to confirm the Main_

(1L is tripped, then confirm the Main (2) is tripped.

The reason the Main (2) is then tripped immediately following a Main (1) trip is to prevent (3) .

ill (2) (3)

A. Generator Turbine overspeeding the Turbine/Generator B. Turbine Generator overspeeding the Turbine/Generator

c. Generator Turbine motoring the Main Generator D. Turbine Generator motoring the Main Generator Answer: D Answer Explanation Knowledge and Ability Reference Information Importance Rating K&A RO SRO 295005 Main Turbine Generator Trip /3 AK3.04- Knowledge of the reasons for the following 3.2 3.2 responses as they apply to MAIN TURBINE GENERATOR TRIP: Main generator trip Level RO I Tier I 1 I Group I 1 General GEK-5522 ABN-10 References OCSOPS ILT 13-1 EXAM Page: 119 of 201 13 May 2014

EXAMINATION ANSWER KEY ILT 13~1 NRC Written Exam (RO)

Dis Correct. lAW GEK-5522 section GEK-46517, Sequential Tripping and Prevention of Motoring, the reason the Main Generator is tripped immediately following a Main Turbine trip is if the Main Generator didn't immediately (or relatively soon after) the Main Turbine Trip, it will result in motoring the Main Generator and cause rapid heating of the L.P. turbine exhaust hoods and L.P. turbine last stage buckets. The immediate Explanation actions of ABN-1 0 (below 30% power) are to Confirm the Main Turbine is tripped, then Confirm the Main Generator is tripped.

All distractors are Incorrect but plausible if the applicant does not recall the order of Immediate Actions of ABN-10 or the reason.

Overspeeding is plausible if the applicant believes that the Main Generator motoring would result in maintaining or raising the speed of the Main Turbine/Generator.

Lesson 2621.828.0.0050, Turbine and Turbine Auxiliaries Plan MTA-10444, Describe the interlock signals and setpoints for the affected system components and expected system response Learning including power loss or failed components.

Objective/

References ILT: None LORT: Open Provided Question Source (New, Modified, Bank) Bank Previous 2 NRC Exams (ILT Only) Yes Memory or Comprehension Fundamental 1:B Cognitive or Analysis Knowledge Level NUREG 1021 Appendix B: Bases or purpose 55.41b 5 55.43b Facility operating characteristics during steady state and transient 10CFR55 conditions, including coolant chemistry, causes and effects of Content temperature, pressure and reactivity changes, effects of load changes, and operating limitations and reasons for these operating characteristics.

Justification for LORT questions with KIA values< 3.0 I NIA Time to Complete: 1-2 minutes Point Value: 1 System 10 No.: 295005 PRA: No Safety Function(s): 3 ~ ILT Category(s) (LORT Only): NIA 0 LORT OCS OPS ILT 13-1 EXAM Page: 120 of 201 13 May 2014

EXAMINATION ANSWER KEY ILT 13~1 NRC Written Exam (RO) 47 ID: 13-1 NRO 47 Points: 1.00 The plant had just been shutdown when a complete loss of offsite power occurred due to a fault on both 34.5 KV power lines.

Which of the following is correct regarding AC power supplies? (Assume NO operator actions unless stated)

A. Combustion Turbine #1 can be manually aligned directly to Bus 1A to provide power to Feedwater Pump 1A and Condensate Pump 1A.

B. Emergency Diesel Generator #1 will automatically start and load onto Bus 1C to provide power to Bus 1A2 to provide power to CRD Pump 1A.

C. When the fault has been cleared on Bank 5, breaker 81 B will automatically close to provide power to Feedwater Pump 1B and Condensate Pump 1B.

D. Emergency Diesel Generator #2 will automatically start and load onto Bus 1D to provide power to Bus 1B2 to provide power to Condensate Transfer Pump 1*2.

Answer: B Answer Explanation Knowledge and Ability Reference Information Importance Rating K&A RO SRO 295003 Partial or Complete Loss of A.C. Power/6 AK3.01 - Knowledge of the reasons for the following responses 3.3 3.5 as they apply to PARTIAL OR COMPLETE LOSS OF A. C.

POWER: Manual and auto bus transfer Level RO I Tier I 1 I Group I 1 General 341 UFSAR 8.3.1.1.1 References OCS OPS ILT 13-1 EXAM Page: 121 of 201 13 May 2014

EXAMINATION ANSWER KEY ILT 13~1 NRC Written Exam (RO)

The plant is shutdown with power supplied from the 34.5 KV startup transformers.

The question stem then describes a loss of offsite power. 4160 Busses 1A, 1B, 1C and 1D will become de-energized. The 34.5 KV lines supply power to the station through startup transformers SA and SB. 34.5 KV Bank 5 supplies SA, and Bank 6 supplies SB.

EDGs 1 and 2 will automatically start and load onto their respective busses due to the loss of voltage (EDG1 loads onto Bus 1C and EDG2 loads onto Bus 1D). EDG1 will power Bus 1C and Bus 1A2 will then receive power. CRD Pump 1A will then be powered. CRD can be used as an RPV high pressure injection source and it can also be used to insert control rods during ATWS conditions while at power. Answer B is Correct. (condition:

loss of AC power; reason: to automatically transfer to EDG1 to supply CRD Pump)

Even with the EDGs operating, the combustion turbines (CT)

Explanation remain available and can be procedurally started and aligned to supply power to the station through 4160 Bus 1B only. Feedwater and condensate pumps 1A are powered from Bus 1A. Thus, the CT cannot directly power Bus 1A. Answer A is Incorrect.

It is expected that when the fault is cleared on the startup transformer, that it can automatically close onto its respective bus. As stated earlier, Bank 5 supplies startup transformer SA (through breaker S1 A)- not startup transformer SB (through breaker S1 B). Feedwater/condensate Pumps are powered from Bus 1B. Answer C is Incorrect.

As stated earlier, EDG2 will automatically start and load onto Bus 1D which will supply power to Bus 1B2. The condensate transfer pumps, which are used to provide makeup capability to the isolation condenser shells, are powered from 1B32 - not 1B2.

Bus 1B32 remains de-energized under the conditions presented in the question. Answer Dis Incorrect.

Lesson 2621.828.0.0013, Emergency Diesel Generators Plan EDG-1 0444, Describe the interlock signals and setpoints for the affected system components and expected system response Learning including power loss or failed components.

Objective/

References ILT: None LORT: Open Provided Question Source (New, Modified, Bank) Bank OCS OPS ILT 13-1 EXAM Page: 122 of 201 13 May 2014

EXAMINATION ANSWER KEY ILT 13~1 NRC Written Exam (RO)

Previous 2 NRC Exams (ILT Only) J No Memory or Comprehension Fundamental 1:8 Cognitive or Analysis Knowledge Level NUREG 1021 Appendix 8: Bases 55.41b 7 55.43b 10CRF55 Design, components, and functions of control and safety systems, Content including instrumentation, signals, interlocks, failure modes, and automatic and manual features.

Justification for LORT questions with KIA values< 3.0 I NIA Time to Complete: 1*2 minutes Point Value: 1 System ID No.: 295003 PRA: No Safety Function(s): 6 181 ILT Category(s) (LORT Only): N/A 0 LORT OCS OPS ILT 13-1 EXAM Page: 123 of 201 13 May 2014

EXAMINATION ANSWER KEY ILT 13~1 NRC Written Exam (RO) 48 10: 13*1 NRO 48 Points: 1.00 The plant was in a normal rated power condition. A security event led the crew to manually insert a reactor SCRAM. Plant conditions now include the following:

  • All immediate scram actions are complete
  • RPV water level lowered to 120 inches and is now stable at 138 inches
  • RPV Pressure is 850 psig and rising slowly without any operator action
  • The security event NO longer exists; no plant systems were impacted Assuming NO operator action is taken to control RPV Pressure, over the NEXT 30 minutes, RPV Pressure will continue to RISE ...

A then stabilize at approximately 942 psig B. to approximately 1051 psig then start to lower C. then stabilize at approximately 1065 psig D. then stabilize at approximately 1085 psig Answer: A Answer Explanation Knowledge and Ability Reference Information Importance Rating K&A RO SRO 295006 SCRAM /1 AA 1.03 -Ability to operate and/or monitor the following as 3.7 3.7 they apply to SCRAM: Reactor/turbine pressure regulating system Level RO I Tier I 1 I Group I 1 General ABN-1 RPVC-no ATWS References OCS OPS ILT 13*1 EXAM Page: 124 of 201 13 May 2014

EXAMINATION ANSWER KEY ILT 13~1 NRC Written Exam (RO)

A is Correct. At rated power, the EPR relay setpoint is approximately 942 psig which is turbine throttle pressure.

Following a reactor scram, with no headloss the piping between the reactor and turbine, RPV Pressure will rise and stabilize at approximately 942 psig. This question examines the operators ability to monitor the Turbine Control System following a reactor scram and predict the plant response. The reason for the scram, a security threat, was used to place the plant in a condition where the scram was 'uncomplicated' and manually initiated. Without stating a reason could lead the applicant to believe there was a plant parameter trending toward a scram setpoint possibly Explanation making one of the distractors correct.

B is Incorrect but plausible since this is the setpoint where the Isolation Condensers would automatically initiate. With the ICs in service, RPV Pressure would lower. This is also what would occur if the MSIVs were closed, however the question stem states they*re open.

C and D are Incorrect but plausible since 1065# and 1085# are both the setpoints for the EMRVs. In addition, since the EMRVs open on spring pressure, if the RPV were isolated with ICs unavailable, CorD could be correct.

Lesson 2621.828.0.0051, Turbine Controls Plan TCS-10445, Given a set of system indications or data, evaluate Learning and interpret them to determine limits, trends and system status.

Objective/

References ILT: None LORT: Open Provided Question Source (New, Modified, Bank) New Previous 2 NRC Exams (ILT Only) No Memory or Comprehension Fundamental 3:PEO Cognitive Knowledge or Analysis Level NUREG 1021 Appendix 8: Predict an Event or Outcome OCS OPS ILT 13-1 EXAM Page: 125 of 201 13 May 2014

EXAMINATION ANSWER KEY ILT 13~1 NRC Written Exam (RO) 55.41b I 5 I 55.43b I

Facility operating characteristics during steady state and transient 10CFR55 conditions, including coolant chemistry, causes and effects of Content temperature, pressure and reactivity changes, effects of load changes, and operating limitations and reasons for these operating characteristics.

Justification for LORT questions with KIA values< 3.0 I NIA Time to Complete: 1-2 minutes Point Value: 1 System ID No.: 295006 PRA: No Safety Function(s): 1 IZI ILT Category(s) (LORT Only): N/A 0 LORT OCS OPS ILT 13-1 EXAM Page: 126 of 201 13 May 2014

EXAMINATION ANSWER KEY ILT 13~1 NRC Written Exam (RO) 49 ID: 13*1 NRO 49 Points: 1.00 The plant was at rated power when the following annunciators alarmed:

  • DW PRESS HI-HI I
  • DW PRESS HI-HI II After taking the required manual actions at Panel 4F, the Operator makes the following report:
  • All red scram indicating lights are out on Panel 4F
  • All amber LPRM lights are out on Panel 4F
  • The turbine is still on-line
  • The Operator is manually inserting control rods lAW Support Procedure 21, Alternate Insertion of Control Rods, which of the following states an alternate method to achieve a shutdown reactor given the conditions above?

A. Place the 100 Amp Main RPS breakers to OFF.

B. Reset the scram and manually scram the reactor.

C. Place the RPS Sub Channel Keylock switches to TRIP.

D. Close the Scram Air Header Drain valve and open the Scram Air Header Isolation valve.

Answer: C Answer Explanation Knowledge and Ability Reference Information Importance Rating K&A RO SRO 295037 SCRAM Conditions Present and Reactor Power Above APRM Downscale or Unknown /1 EA1.01 - Ability to operate and/or monitor the following as 4.6 4.6 they apply to SCRAM CONDITION PRESENT AND REACTOR POWER ABOVE APRM DOWNSCALE OR UNKNOWN: Reactor Protection System Level RO I Tier I 1 J Group I 1 General EMG-SP21 References OCS OPS ILT 13-1 EXAM Page: 127 of 201 13 May 2014

EXAMINATION ANSWER KEY ILT 13~1 NRC Written Exam (RO)

C is Correct. The indications show a scram condition with the reactor still at power with an electric ATWS. lAW the reference, under the given conditions and with the MSIVs open, the correct action to insert all control rods is to place the RPS Sub Channel Keylock switches to TRIP. Answer Cis correct.

A is Incorrect but plausible since it would be correct if the MSIVs Explanation were closed.

B is Incorrect but plausible if the applicant does not recognize there is an Electric ATWS, not a Hydraulic ATWS, since this is an action for a hydraulic ATWS.

Dis Incorrect but plausible if the applicant does not recognize these valve positions for the valves provided are reversed.

Lesson 2621.828.0.001 B, RPV Control with ATWS Plan EWA-03055: Given a copy of RPV Control, describe in detail each Learning step or conditional statement, including technical basis, and Objective/ how to perform each step as required.

References ILT: None LORT: Open Provided Question Source (New, Modified, Bank) Bank Previous 2 NRC Exams (ILT Only) No Memory or Comprehension Fundamental 3:SPK Cognitive or Analysis Knowledge Level NUREG 1021 Appendix B: Solve a Problem using Knowledge and its meaning 55.41b 10 55.43b 10CFR55 Content Administrative, normal, abnormal, and emergency operating procedures for the facility.

Justification for LORT questions with KIA values< 3.0 I NIA Time to Complete: 1-2 minutes Point Value: 1 System ID No.: 295037 PRA: No Safety Function(s): 1 ~ ILT Category(s) (LORT Only): NIA 0 LORT OCS OPS ILT 13-1 EXAM Page: 128 of 201 13 May 2014

EXAMINATION ANSWER KEY ILT 13~1 NRC Written Exam (RO) 50 10: 13*1 NRO 50 Points: 1.00 The plant was at rated power when an event occurred. Present plant conditions include the following:

  • The Drywell atmosphere is at saturated conditions with a Drywell temperature of 245 °F and rising slowly
  • Torus temperature indicates 89 oF and steady

- B 550 c 500

- 1 (7,sso}

- J 450 DRYWELL TFMPFRATURE 350 400 1/

('F) - J 30{)

250

- I 200

- I 150

- I 100

- /A 0 18 5 10 15 20 25 30 '<I';

........l 40 DRYWELL PRESSURE (PSIG) lAW the EOPs, which ONE of the following is correct regarding the Containment Spray System?

A. A loop of Torus Cooling is required to be initiated.

B. Drywell Sprays may NOT be initiated since Drywell Pressure is less than 12 psig.

C. Drywell Sprays may be initiated since conditions are on the good side of the Containment Spray Initiation Limit Curve.

D. Drywell Sprays may NOT be initiated since conditions are on the bad side of the Containment Spray Initiation Limit Curve.

Answer: C OCS OPS ILT 13-1 EXAM Page: 129 of 201 13 May 2014

EXAMINATION ANSWER KEY ILT 13-1 NRC Written Exam (RO)

Answer Explanation Knowledge and Ability Reference Information Importance Rating K&A RO SRO 295028 High Drywell Temperature I 5 EA1.01 *Ability to operate and/or monitor the following as 3.8 3.9 they apply to HIGH DRYWELL TEMPERATURE : Drywell spray: Mark-1&11 Level RO J Tier I 1 I Group J 1 General PCC EOP EOP Users Guide References Cis Correct. The Conditions in the stem place the operator monitoring Drywell Temperature which is high at 245F. Using the Steam Table in Attachment ABN-30*4, the applicant must determine this corresponds to a Drywell Pressure of 13psig to continue answering the question. Of the choices listed, the only one that is correct is that they may place the Drywell Spray system in service since it is less than the Containment Spray Initiation Limit.

Explanation A is Incorrect but plausible since Torus Temperature is elevated, however has not yet reached the point, 95F, where Torus Cooling is required to be initiated.

B & D are Incorrect but plausible if the applicant does not correctly determine what Drywell Pressure is based on the Drywell Temperature (at saturation).

Lesson 2621.845.0.0002, Primary Containment Control Plan Learning PCC-03000: Using Procedure EMG-3200.02, evaluate the Objective/ technical basis for each step in the procedure, and apply this evaluation to determine correct courses of action under emergency conditions.

References I LT: Steam Table LORT: Open Provided (Attachment ABN-30*4)

Question Source (New, Modified, Bank) Bank Previous 2 NRC Exams (ILT Only) No OCS OPS ILT 13-1 EXAM Page: 130 of 201 13 May 2014

EXAMINATION ANSWER KEY ILT 13~1 NRC Written Exam (RO)

Memory or Comprehension Fundamental 3:SPR Cognitive or Analysis Knowledge Level NUREG 1021 Appendix 8: Solve a Problem using References 55.41b 10 55.43b 10CFR55 Content Administrative, normal, abnormal, and emergency operating procedures for the facility.

Justification for LORT questions with KIA values< 3.0 I N/A Time to Complete: 1-2 minutes Point Value: 1 System I D No.: 295028 PRA: No Safety Function(s): 5 181 ILT Category(s) (LORT Only): N/A 0 LORT OCS OPS ILT 13-1 EXAM Page: 131 of 201 13 May 2014

EXAMINATION ANSWER KEY ILT 13-1 NRC Written Exam (RO) 51 10: 13*1 NRO 51 Points: 1.00 The plant was at 28% power when a Feedwater Level Control malfunction resulted in RPV water level rising to a maximum value of 177 inches (Yarway).

FIVE MINUTES after the event the following indications were noted:

  • Several EMRVs indicated in the VALVE OPEN REGION for 4 seconds and now indicate in the VALVE CLOSED REGION
  • RPV Pressure indicates 800 psig and lowering Which of the following statements is correct (assume NO operator actions were taken)?

A. ALL MSIVs are OPEN B. BOTH EDGs are idling C. ALL Reactor Feed Pumps are tripped D. BOTH Isolation Condensers are in service Answer: D Answer Explanation Knowledge and Ability Reference Information Importance Rating K&A RO SRO 295025 High Reactor Pressure /3 EA2.06 - Ability to determine and/or interpret the following 3.7 3.8 as they apply to HIGH REACTOR PRESSURE: Reactor water level Level RO I Tier 1 I Group J 1 General RAP*C1a RAP-J1b References OCS OPS ILT 13-1 EXAM Page: 132 of 201 13 May 2014

EXAMINATION ANSWER KEY ILT 13*1 NRC Written Exam (RO)

Dis Correct. RPV level> 175" (Yarway) will result in a Turbine trip. Five minutes following the Turbine trip the indications provided state that several EMRVs had opened then closed. This indicates RPV Pressure will rise above the reactor scram setpoint (1 045#) and the initiation set point for the Isolation Condensers (>

1051#) and both ICs will initiate. The question stem states that several EMRVs indicated in the VALVE OPEN REGION for 4 seconds and now indicate in the VALVE CLOSED REGION which indicates that RPV pressure rose to at least 1065# which is above the IC initiation setpoint of 1051#.

Explanatio A is Incorrect. Due to no operator action taken the mode switch is n still in RUN. Following the scram and IC initiation, RPV Pressure will drop below the MSIV closure setpoint of 845#. This will have occurred and the MSIVs will be closed within the 5 minutes stated in the question stem.

B is Incorrect. The conditions for an EDG Idle start have not been satisfied (Hi OW Pressure or Lo-Lo RPV water level).

C is Incorrect. The Reactor Feed Pump trip setpoint (ROPS) is 181". The question stem states that the highest RPV level attained was 177", therefore (with no operator actions) all feed pumps are still operating.

Lesson 2621.828.0.0023, Isolation Condensers Plan 2030, Describe the Isolation Condenser design feature(s) and/or interlocks (including signals and setpoints) which provide for the following: a) Automatic System Initiation and b) Automatic Learning System Isolation Objective/

References ILT: None LORT: Open Provided Question Source (New, Modified, Bank) Bank Previous 2 NRC Exams (ILT Only) N/A Memory or Comprehension Cognitive Fundamental X or Analysis Level Knowledge 10CFR55 55.41b 10 55.43b Content Justification for LORT questions with KIA values N/A

< 3.0 Time to Complete: 1*2 minutes I Point Value: 1 OCS OPS ILT 13-1 EXAM Page: 133 of 201 13 May 2014

EXAMINATION ANSWER KEY ILT 13~1 NRC Written Exam (RO)

System ID No.: 295025 PRA: I No Safety Function(s): 3 181 IL T Category(s) (LORT Only):

10 181 LORT OCS OPS ILT 13-1 EXAM Page: 134 of 201 13 May 2014

EXAMINATION ANSWER KEY ILT 13~1 NRC Written Exam (RO) 52 10: 13*1 NRO 52 Points: 1.00 The plant was at rated power.

A leak in the Service Air system in the Condensate Demineralizer Regen Room, caused a drop in Service Air pressure and the Operator reported the following observations:

  • The backup air compressors are running
  • INSTR AIR SUPPLY PRESS was rising When INSTR AIR SUPPLY PRESS reached 100 psig, the Service Air leak worsened.

It resulted in a lowering of Service Air pressure at the rate of 2 psig/minute.

lAW ABN-35, Loss of Instrument Air, which of the following is correct under the conditions provided? (Assume plant components respond as designed)

A. NO manual scram will be required due to lowering air pressure.

B. A manual scram will be required in 19 minutes from lowering air pressure.

C. A manual scram will be required in 23 minutes from lowering air pressure.

D. A manual scram will be required in 27 minutes from lowering air pressure.

Answer: A Answer Explanation Knowledge and Ability Reference Information Importance Rating K&A RO SRO 295019 Partial or Total Loss of lnst. Air /8 AA2.01

  • Ability to determine and/or interpret the following 3.5 3.6 as they apply to PARTIAL OR COMPLETE LOSS OF INSTRUMENT AIR : Instrument air system pressure Level RO I Tier I 1 J Group J 1 General ABN-35 RAP-M2b BR 2013, sh. 1 References OCS OPS ILT 13-1 EXAM Page: 135 of 201 13 May 2014

EXAMINATION ANSWER KEY ILT 13~1 NRC Written Exam (RO)

A is Correct. The plant is at power when an event caused the standby air compressors to start on low air pressure. After the compressors start, air pressure begins to recover. At 100 psig, the malfunction worsens and begins to lower air pressure at a rate of 2 psig/minute. The leak is downstream of V-6S-2, Service Air Isolation Valve. When air pressure lowers to 75 psig, the service air isolation valve, V-6S-2, will automatically close. This will isolate air going to service air from the air compressors and Explanation instrument air. Once isolated, the running air compressors will restore instrument air pressure to normal and no scram will be required due to lowering air pressure.

lAW ABN-35, when instrument air pressure drops to 55 psig, then a manual scram is required. All other answers are Incorrect but plausible if the applicants forget about the service isolation valve automatic action and the setpoint on which to perform the manual scram.

Lesson 2624.828.0.0043, Service, Instrument, & Breathing Air Plan CAS-1 0445, Given a set of system indications or data, evaluate Learning and interpret them to determine limits, trends and system status.

Objective/

References ILT: None LORT: Open Provided Question Source (New, Modified, Bank) Bank Previous 2 NRC Exams (ILT Only) No Memory or Comprehension Fundamental 3:PEO Cognitive or Analysis Knowledge Level NUREG 1021 Appendix B: Predict an Event or Outcome 55.41b 7 55.43b 10CFR55 Design, components, and functions of control and safety systems, Content including instrumentation, signals, interlocks, failure modes, and automatic and manual features.

Justification for LORT questions with KIA values< 3.0 I N/A Time to Complete: 1-2 minutes Point Value: 1 System 10 No.: 295019 PRA: No Safety Function(s): 8 ~ ILT Category(s) (LORT Only): NIA 0 LORT OCS OPS ILT 13-1 EXAM Page: 136 of 201 13 May 2014

EXAMINATION ANSWER KEY ILT 13~1 NRC Written Exam (RO) 53 10: 13*1 NRO 53 Points: 1.00 The plant is at rated power.

Under the current plant conditions, which of the following will cause a system isolation (change in valve position), in order to minimize an off-site release of radioactivity?

(assume NO operator actions and assume any time delays are satisfied)

A. Service Water Radiation Monitor indicates upscale B. Both Off Gas Radiation Monitors indicate upscale C. Both Containment Hi Range Radiation Monitors indicate upscale D. Both Plant Stack Radioactive Gaseous Effluent Radiation Monitor channels indicate upscale Answer: B Answer Explanation Knowledge and Ability Reference Information Importance Rating K&A RO SRO 295038 High Off-site Release Rate I 9 EA2.01 - Ability to determine and/or interpret the following 3.3 4.3 as they apply to HIGH OFF-SITE RELEASE RATE: tOff*

site Level RO J Tier I 1 I Group I 1 General 1OF-1-c References 8 is Correct. All distractors are Incorrect but plausible since they are all other significant radiation alarms. The stack RAGEMS do not perform an isolation function when tripped high. CHRRMS will close OW/Torus vent/purge valves when tripped, but none of Explanation these valves would be open under the present plant conditions, thus no change in valve status takes place. High-high trip of both offgas monitors isolates offgas from the plant stack (with a time delay). Service water rad monitor provides for no protective functions.

OCS OPS ILT 13-1 EXAM Page: 137 of 201 13 May 2014

EXAMINATION ANSWER KEY ILT 13~1 NRC Written Exam (RO)

Lesson 2621.828.0.033A, Plant Radiation Monitoring Systems Plan 272-10449, State the function and interpretation of system alarms, Learning alone and in combination, as applicable in accordance with the Objective/ system RAPS.

References ILT: None LORT: Open Provided Question Source (New, Modified, Bank) Bank Previous 2 NRC Exams (ILT Only) No Memory or Comprehension Fundamental 1:I Cognitive or Analysis Knowledge Level NUREG 1021 Appendix B: lnterlocks, setpoints, or system (singular) response 55.41b 7 55.43b 10CFR55 Design, components, and functions of control and safety systems, Content including instrumentation, signals, interlocks, failure modes, and automatic and manual features.

Justification for LORT questions with KIA values< 3.0 I NIA Time to Complete: 1-2 minutes Point Value: 1 System 10 No.: 295038 PRA: No Safety Function(s): 9 ~ ILT Category(s) (LORT Only): N/A 0 LORT OCS OPS ILT 13-1 EXAM Page: 138 of 201 13 May 2014

EXAMINATION ANSWER KEY ILT 13~1 NRC Written Exam (RO) 54 10: 13*1 NRO 54 Points: 1.00 The plant was at rated power when the following annunciators alarmed:

  • TBCCW- DISCH PRESS LO
  • TBCCW- SURGE TANK LVL HIILO
  • TURB BLDG SUMP LVLS- LUBE OIL BAY 1-1 LEVEL HI
  • TURB BLDG SUMP LVLS- CONDENSATE BAY 1-2 LEVEL HI The Operator reports TBCCW HX OUTLET PRESS indicates 15 psig and lowering and that the TBCCW Surge Tank indicates low and CANNOT be raised.

Which of the following states the required Subsequent Operator Actions lAW ABN-20, TBCCW Failure Response?

First Action Second Action A. Manually scram the reactor Stop all recirculation pumps B. Manually scram the reactor Manually trip the turbine C. Secure TBCCW Pumps Manually scram the reactor D. Perform a rapid power Manually trip the turbine reduction Answer: A Answer Explanation Knowledge and Ability Reference Information Importance Rating K&A RO SRO 295018 Partial or Total Loss of CCW /8 2.1.23

  • Conduct of Operations: Ability to perform specific 4.3 4.4 system and integrated plant procedures during all modes of plant operation Level RO I Tier I 1 I Group I 1 General ABN-20 RAP-M4a References OCS OPS ILT 13-1 EXAM Page: 139 of 201 13 May 2014

EXAMINATION ANSWER KEY ILT 13~1 NRC Written Exam (RO)

A is Correct. The question stem depicts a major leak in the TBCCW System: low system pressure resulting in auto start of the standby pump, an alarm of the TBCCW surge tank (hi or low),

indications of flooding where TBCCW would be present, and a control room indication of lowering TBCCW pressure. Together, these describe a major leak in the TBCCW System. lAW ABN-20, TBCCW Failure Response, a 'major unisolable TBCCW leak' is defined by: 1) a leak exceeding the makeup capacity and 2) a leak which cannot be isolated quickly and 3) a leak that results in imminent loss of the TBCCW system due to loss of NPSH to the TBCCW pumps". Two of the three requirements are clear from what is given {1 and 3). The second can be inferred since the question provides no clues/indications that the leak can be isolated quickly. Also, since the system pressure is lowering, the problem has not been corrected. Therefore, there is a major unisolable TBCCW leak. lAW ABN-20, the first 2 subsequent operator actions while at power, are to scram the reactor and stop all recirculation pumps. Answer A is correct.

Explanation B is Incorrect but plausible. There are no indications in the question stem that suggest that the turbine will not trip when the reactor is scrammed. Therefore, the turbine will trip when the reactor is scrammed. ABN-1, Reactor Scram, does not require a manual turbine trip following the scram. Answer B is plausible but incorrect.

C is Incorrect but plausible. Because there are indications of flooding in the question stem, from apparently the TBCCW System, it is plausible to secure the pumps responsible for the flooding. Answer Cis plausible but incorrect.

D is Incorrect but plausible. Because the TBCCW system discharge pressure is low, cooling of TBCCW components has been reduced. To minimize the impact, performing a rapid power reduction would help to minimize the TBCCW cooling requirements. Tripping the turbine results in the greatest load reduction in TBCCW and is a plausible distractor. Answer Dis incorrect.

Lesson 2621.828.0.0048, TBCCW Plan 274*10450, Describe and interpret procedure sections and steps Learning for plant emergency and off-normal conditions that involve this Objective/ system including personnel allocation and equipment operation lAW plant procedures.

OCS OPS ILT 13-1 EXAM Page: 140 of 201 13 May 2014

EXAMINATION ANSWER KEY ILT 13~1 NRC Written Exam (RO)

References ILT: None LORT: Open Provided Question Source (New, Modified, Bank) Bank Previous 2 NRC Exams (ILT Only) No Memory or Comprehension Fundamental 3:SPK Cognitive or Analysis Knowledge Level NUREG 1021 Appendix B: Solve a Problem using Knowledge and its meaning 55.41b 10 55.43b 10CFR55 Content Administrative, normal, abnormal, and emergency operating procedures for the facility.

Justification for LORT questions with KIA values< 3.0 I N/A Time to Complete: 1*2 minutes Point Value: 1 System ID No.: 295018 PRA: No Safety Function(s): 8 181 ILT Category(s) (LORT Only): NIA 0 LORT OCS OPS ILT 13-1 EXAM Page: 141 of 201 13 May 2014

EXAMINATION ANSWER KEY ILT 13~1 NRC Written Exam (RO) 55 10: 13-1 NRO 55 Points: 1.00 The plant is at 3% power on a startup after a refuel outage. A pre-job brief is being conducted in preparation for performing Procedure 602.4.003, Electromatic Relief Valve Operability Test. Torus water temperature is currently 88 °F and Torus Cooling is not in service.

An open EMRV raises Torus water temperature by 2 °F/minute. Which of the following states how long the EMRVs can remain open during the surveillance test UNTIL a Technical Specification reactor scram requirement is FIRST met? (assume a constant heatup rate)

A. 3 Yz minutes B. 8 Yz minutes C. 11 minutes D. 13 minutes Answer: C Answer Explanation Knowledge and Ability Reference Information Importance Rating K&A RO SAO 295026 Suppression Pool High Water Temp./5 2.2.22- Equipment Control: Knowledge of limiting 4.0 4.7 conditions for operations and safety limits.

Level RO l Tier I 1 I Group I 1 General TS3.5 References OCS OPS ILT 13-1 EXAM Page: 142 of 201 13 May 2014

EXAMINATION ANSWER KEY ILT 13~1 NRC Written Exam (RO)

C is Correct. TS 3.5.A.1.c(1) states that during normal power operation, the pool temperature limit is 95 °F.

lAW TS 3.5.A.1.c(2): During testing which adds heat to the suppression pool, the water temperature shall not exceed 10 oF above the normal POWER OPERATION limit specified in (1) above, which is 95 °F. Thus the maximum allowed Torus water temperature during this test is 105 °F.

TS 3.5.A.1.c(3) states that the reactor shall be scrammed from any power condition if the pool temperature reaches 110 oF. At 2 Explanation oF/minute for 11 minutes (= 22 °F), the pool temperature will reach 88 + 22 = 110 oF. Thus, after 11 minutes the scram requirement will be first met. Answer Cis correct.

Answer A places the torus water temperature at the normal power operation TS temperature limit of 95 °F. Answer B places the torus water temperature at the TS testing temperature limit of 105 °F. Answer D places the torus water temperature above the TS scram temperature limit of 110 oF but is past the time when the scram is first required as in answer B. All distractors are Incorrect but plausible since the values are TS values in some fashion.

Lesson 2621.828.0.0032, Primary Containment System Plan Referencing plant Technical Specifications (* from memory for Learning Initial Candidates) and given a set of plant conditions, determine, Objective/ as applicable, the:

a) Definitions*

b) Safety Limits and Bases*

c) Limiting Safety System Settings and Bases*

d) Limiting Conditions for Operation and Applicability e) LCO Action Requirements (SAO ONLY) f) Surveillance Requirements (SAO ONLY) g) Design Features, Containment, Auxiliary Equipment, Administrative Controls, and Appendix B Environmental Technical Specifications (SAO ONLY) h) Bases for Surveillance Requirements, Design Features, Containment, Auxiliary Equipment, Administrative Controls, and Appendix B Environmental Technical Specifications. (SAO ONLY)*

References ILT: None LORT: Open Provided OCS OPS ILT 13-1 EXAM Page: 143 of 201 13 May 2014

EXAMINATION ANSWER KEY ILT 13~1 NRC Written Exam (RO)

Question Source (New, Modified, Bank) Bank Previous 2 NRC Exams (ILT Only) No Memory or Comprehension Fundamental 3:SPK Cognitive or Analysis Knowledge Level NUREG 1021 Appendix B: .Solve a Problem using Knowledge and its meaning 55.41b 10 55.43b 10CFR55 Content Administrative, normal, abnormal, and emergency operating procedures for the facility.

Justification for LORT questions with KIA values< 3.0 I NIA Time to Complete: 1-2 minutes Point Value: 1 System 10 No.: 295026 PRA: No Safety Function(s): 5 ~ ILT Category(s) (LORT Only): NIA D LORT OCS OPS ILT 13-1 EXAM Page: 144 of 201 13 May 2014

EXAMINATION ANSWER KEY ILT 13~1 NRC Written Exam (RO) 56 ID: 13*1 NRO 56 Points: 1.00 The plant was at rated power when the Control Room was notified that Drywell pressure switches PS RV46A and PS RV46B, which input into the starting circuit for the Core Spray System, have failed in their current state such that they will not detect a high Drywell pressure condition.

Which of the following states the ability of the Core Spray System to function during a high Drywell pressure condition?

A Core Spray Pumps A AND B will auto start as designed, with no manual Operator actions required.

B. Core Spray Pump A will NOT auto start, but MAY be manually started.

Core Spray Pump B will auto start as designed.

C. Core Spray Pump A will NOT start and CANNOT be manually started.

Core Spray Pumps B AND C auto start as designed.

D. NEITHER Core Spray Pump A NOR B will auto start, but can be manually started. All other Core Spray components operate as designed.

Answer: A Answer Explanation Knowledge and Ability Reference Information Importance Rating K&A RO SAO 295024 High Drywell Pressure I 5 2.2.37 *Equipment Control: Ability to determine operability 3.6 4.6 and/or availability of safety related equipment Level RO I Tier I 1 I Group I 1 General NU 5060E6003, sh.

2621.828.0.001 0 RAP-C2f References 1-4 OCS OPS ILT 13-1 EXAM Page: 145 of 201 13 May 2014

EXAMINATION ANSWER KEY ILT 13~1 NRC Written Exarn (RO)

A is Correct. With no failures, a single high Drywell pressure signal will start the Core Spray System normally. This includes the Core Spray System A and B. There are 4 Drywell high pressure switches. If any two fails, there are still 2 others to start the Core Spray System in its normal start mode.

Explanation Two instrument failures in RPS could render that RPS channel inoperable, but the Core Spray start logic is inter-mixed among systems.

The other answers are plausible but Incorrect since no manual operator actions are required.

Lesson 2621.828.0.0010, Core Spray System Plan CSS-10439, Given the system logic/electrical drawings, describe Learning the system auto initiation signals, setpoints and expected system Objective/ response including power loss or failed components.

References ILT: None LORT: Open Provided Question Source (New, Modified, Bank) Bank Previous 2 NRC Exams (ILT Only) No Memory or Comprehension Fundamental 1:1 Cognitive or Analysis Knowledge Level NUREG 1021 Appendix B: Interlocks, setpoints, or system (singular) response 55.41b 7 55.43b 10CFR55 Design, components, and functions of control and safety systems, Content including instrumentation, signals, interlocks, failure modes, and automatic and manual features.

Justification for LORT questions with KlA values < 3.0 I N/A Time to Complete: 1-2 minutes Point Value: 1 System ID No.: 295024 PRA: No Safety Function(s): 5 ~ ILT Category(s) (LORT Only): N/A 0 LORT OCS OPS ILT 13-1 EXAM Page: 146 of 201 13 May 2014

EXAMINATION ANSWER KEY ILT 13~1 NRC Written Exam (RO) 57 10: 13*1 NRO 57 Points: 1.00 Which ONE of the following contains indications that are an EOP entry condition?

A.

DVV TORUS PRESS PRESS Pf..IPOS I PT-IP12 OCS OPS ILT 13-1 EXAM Page: 147 of 201 13 May 2014

EXAMINATION ANSWER KEY ILT 13~1 NRC Written Exam (RO)

B. REACTOR LEVEL NARROW RANGE YARWAY GEMAC C.

OCS OPS ILT 13*1 EXAM Page: 148 of 201 13 May 2014

EXAMINATION ANSWER KEY ILT 13~1 NRC Written Exam (RO)

D.

  • r**rnR REACTOR R*E

' }-)>,_. v '

FEEDVV.A.TER F,r*:.E*::::e; INLET TEMP

_r_E-*4-7........

1.1.' 111£1 *.1*.-***

Answer: C Answer Explanation Knowledge and Ability Reference Information Importance Rating K&A RO SRO 295030 Low Suppression Pool Water Level/ 5 2.4.1 - Emergency Procedures I Plan: Knowledge of EOP 4.6 4.8 entry conditions and immediate action steps.

Level RO I Tier I 1 I Group I 1 General PCC EOP EOP Users Guide References OCS OPS ILT 13-1 EXAM Page: 149 of 201 13 May 2014

EXAMINATION ANSWER KEY ILT 13~1 NRC Written Exam (RO)

Cis Correct. Torus Water Level less than 143 in requires entry into the Primary Containment Control EOP. Choice C shows indications that Torus Water Level is low at 142 in. This examines the KIA fQr kngwledge of EOP entry ~Qnditigns; there are DQ immediate a~tiQn steps a~sociated with LQw Syggre~~iQn PQQI Water Lewl.

A is Incorrect but plausible. The EOP entry for high Drywell pressure is above 3 psig. The instrument setpoint for OW Hi-Hi LOCA initiations is 2.9 psig. This distractor provides an Explanation indication of 2.8 psig which is close to both of these.

B is Incorrect but plausible. The EOP entry for RPV water level is less than 138 in TAF. The indications shown are 139 in TAF. The scram setpoint for low RPV water level is 141 in (instrument setpoint) so the applicant might confuse this with the EOP entry setpoint.

D is Incorrect but plausible. The EOP entry for High RPV pressure is 1045 psi g. This distractor provides RPV pressure indications of 1040 psig, which is close to the EOP entry_.

Lesson 2621.845.0.0002, Primary Containment Control Plan PCC-03025, Given key plant parameters, determine if entry Learning conditions for the EOPs have been met.

Objective/

References ILT: None LORT: Open Provided Question Source (New, Modified, Bank) Bank Previous 2 NRC Exams (ILT Only) No Memory or Comprehension Fundamental 3:SPK Cognitive or Analysis Knowledge Level NUREG 1021 Appendix B: Solve a Problem using Knowledge and its meaning 55.41b 10 55.43b 10CFR55 Content Administrative, normal, abnormal, and emergency operating procedures for the facility.

Justification for LORT questions with KIA values< 3.0 I N/A Time to Complete: 1-2 minutes I Point Value: I 1 OCS OPS ILT 13-1 EXAM Page: 150 of 201 13 May 2014

EXAMINATION ANSWER KEY ILT 13~1 NRC Written Exam (RO)

System ID No.: 295030 PRA: I No Safety Function(s): 5 1:81 ILT Category(s) (LORT Only): N/A 0 LORT OCS OPS ILT 13-1 EXAM Page: 151 of 201 13 May 2014

EXAMINATION ANSWER KEY ILT 13"1 NRC Written Exam (RO) 58 ID: 13-1 NRO 58 Points: 1.00 The plant was shutdown and was cooling down with the Shutdown Cooling System (SOC). Present plant conditions include the following:

  • RPV water level indicates 160" and steady
  • RECIRC PUMP SUCTION TEMPS indicates 340 °F
  • Shutdown Cooling Pumps A and Care in service Which ONE of the following annunciators/trends warns the crew of the GREATEST impact on the cooldown rate?

A. Annunciator 1A2 MN BRKR TRIP alarms.

B. Annunciator 182 MN BRKR TRIP alarms.

C. RECIRC PUMP SUCTION TEMPS rises to 351 °F D. SOC Pumps A and B suction pressure lowers to 10 psig.

Answer: C Answer Explanation Knowledge and Ability Reference Information Importance Rating K&A RO SRO 295021 Loss of Shutdown Cooling I 4 AA1.05- Ability to operate and/or monitor the following as 3.0 3.0 they apply to LOSS OF SHUTDOWN COOLING : Reactor recirculation Level RO I Tier I 1 I Group I 1 General ABN-3 305 References OCS OPS ILT 13-1 EXAM Page: 152 of 201 13 May 2014

EXAMINATION ANSWER KEY ILT 13-1 NRC Written Exam (RO)

The plant is shutdown and cooling down with SOC pumps A & B.

If recirculation loop temperatures exceed 350 oF, SOC isolation valves V-17-19 & V-17-54 auto close. When V-17-19 closes, then all SOC pumps trip. This results in a total loss of SOC and the greatest impact on the cooldown rate. Answer C is correct.

Explanation SOC Pump A is powered from USS 1A2, and SOC pumps B & C are powered from USS 1B2. Thus a loss of either bus results in either the A or B & C pumps available, and also impacts the cooldown rate. Answers A & B are incorrect.

The SOC pumps will trip on a low suction pressure of 4 psig, not 10 as stated in answer D. Answer D is incorrect.

Lesson 2621.828.0.0045, Shutdown Cooling System Plan SDC-10450, Describe and interpret procedure sections and steps Learning for plant emergency or off-normal conditions that involve this Objective/ system including personnel allocation and equipment operation lAW applicable ABN, EOP & EOP support procedures and EP procedures.

References ILT: None LORT: Open Provided Question Source (New, Modified, Bank) Bank Previous 2 NRC Exams (ILT Only) No Memory or Comprehension Fundamental 1:I Cognitive or Analysis Knowledge Level NUREG 1021 Appendix B: Interlocks, setpoints, or system (singular) response 55.41b 10 55.43b 10CFR55 Content Administrative, normal, abnormal, and emergency operating procedures for the facility.

Justification for LORT questions with KIA values< 3.0 I NIA Time to Complete: 1-2 minutes Point Value: 1 System I D No.: 295021 PRA: No Safety Function(s): 4 ~ ILT Category(s) (LORT Only): NIA 0 LORT OCS OPS ILT 13-1 EXAM Page: 153 of 201 13 May 2014

EXAMINATION ANSWER KEY ILT 13~1 NRC Written Exam (RO) 59 10: 13-1 NRO 59 Points: 1.00 The plant was at rated power with elevated offgas radiation readings, when an event occurred.

The Operator reported the following alarms and indications:

  • Annunciator AREA MON HI has alarmed and the ISOLATION COND AREA ARM indicated 80 mr/hr and steady
  • Annunciator COND A FLOW HI POSSIBLE RUPTURE has alarmed
  • See the indications below ISOL CONDEr*JSER ISCtl CONDEf\l:SER A B OCS OPS ILT 13-1 EXAM Page: 154 of 201 13 May 2014

EXAMINATION ANSWER KEY ILT 13~1 NRC Written Exam (RO)

Five minutes later, the Operator observes the following indications:

ISOL r:ONDENSE~: ISOL COr;.JDEf\J:3ER A B Which of the following is correct?

A. A radiological release WAS in progress and Isolation Condenser A HAS isolated.

B. A radiological release IS in progress and Isolation Condenser A HAS FAILED to automatically isolate and should be manually isolated.

C. An Isolation Condenser A steam leak into the reactor Building occurred and Isolation Condenser A HAS isolated.

D. An Isolation Condenser A steam leak into the Reactor Building occurred and Isolation Condenser A HAS FAILED to automatically isolate.

OCS OPS ILT 13-1 EXAM Page: 155 of 201 13 May 2014

EXAMINATION ANSWER KEY ILT 13*1 NRC Written Exam (RO)

Answer: 8 Answer Explanation Knowledge and Ability Reference Information Importance Rating K&A RO SRO 295017 High Off-site Release Rate I 9 AK1.02 - Knowledge of the operational implications of the following concepts as they apply to HIGH OFF-SITE 3.8 4.3 RELEASE RATE: Protection of the general public Level RO I Tier I 1 I Group I 2 General RAP-C3a References The plant was at power when indications are provided which show the following: Isolation Condenser A shell water level is high and pressure is low. The Isolation Condenser area radiation monitor is alarming, and the Possible Rupture alarm is in alarm.

Two minutes later, indications are shown again. This indication shows that Isolation Condenser A shell water level is even higher.

B is Correct.

When the Possible Rupture comes in, after a 27 second time delay, the Isolation Condenser A isolation valves will go closed.

Thus, about 1.5 minutes after this alarm, the valves must still be open since the shell water level is still rising. Therefore, there is a tube leak on Isolation Condenser A resulting in an offsite release Explanation (the shell directly vents outside of the reactor Building), the isolation has failed, and the Operator should manually perform the actions to isolate the Isolation Condenser to protect the general public from a High Off-Site release.

A is Incorrect but plausible if the candidate does not realize that shell water level is still rising, then it will appear that the condenser automatically isolated.

The possible Rupture annunciator could also be indicative of a steam line break in the Reactor Building, especially with the Isolation Condenser ARM in alarm. But since a steam line break would not impact shell water level as in the question, answers C

& D are Incorrect but plausible.

OCS OPS ILT 13-1 EXAM Page: 156 of 201 13 May 2014

EXAMINATION ANSWER KEY ILT 13~1 NRC Written Exam (RO)

Lesson 2621.828.0.0023 Isolation Condensers Plan Learning 2338 Given plant conditions, evaluate the impact on the Isolation Objective/ Condenser System and the plant.

References ILT: None LORT: Open Provided Question Source (New, Modified, Bank) Bank Previous 2 NRC Exams (ILT Only) No Memory or Comprehension Fundamental 3:SPK Cognitive or Analysis Knowledge Level NUREG 1021 Appendix B: .Solve a Problem using Knowledge and its meaning 55.41b 5 55.43b Facility operating characteristics during steady state and transient 10CFR55 conditions, including coolant chemistry, causes and effects of Content temperature, pressure and reactivity changes, effects of load changes, and operating limitations and reasons for these operating characteristics.

Justification for LORT questions with KIA values< 3.0 I NIA Time to Complete: 1M2 minutes Point Value: 1 System 10 No.: 295017 PRA: No Safety Function(s): 9 ~ ILT Category(s) (LORT Only): N/A 0 LORT OCS OPS ILT 13-1 EXAM Page: 157 of 201 13 May 2014

EXAMINATION ANSWER KEY ILT 13~1 NRC Written Exam (RO) 60 ID: 13*1 NRO 60 Points: 1.00 The plant is at rated power when a radiological event occurred.

lAW ABN-26, High Main Steam I Off-Gas I Stack Effluent Activity, which ONE of the following require the crew to insert a manual Scram AND close the MSIVs?

A. CHARMS indicates 50 Rlhr B. Off Gas Hi-Hi annunciator in alarm for 10 min C. Main Steam Line radiation indicates 850 mRihr D. Stack Effluent Hi-Hi annunciator in alarm for 10 min Answer: C Answer Explanation Knowledge and Ability Reference Information Importance Rating K&A RO SRO 295034 Secondary Containment Ventilation High Radiation

/9 EK2.06- Knowledge of the interrelations between 3.9 4.2 SECONDARY CONTAINMENT VENTILATION HIGH RADIATION and the following: PCIS/NSSSS: Plant-Specific Level RO I Tier I 1 I Group I 2 General ABN-26 References OCS OPS ILT 13-1 EXAM Page: 158 of 201 13 May 2014

EXAMINATION ANSWER KEY ILT 13-1 NRC Written Exam (RO)

C is Correct. lAW ABN-26, a Main Steam Line Radiation level of >

800 mr/hr requires the crew to manually scram the reactor, then close the MSIVs following a successful scram. One result of closing the MSIVs (which are part of the PCIS/NSSS), is that it prevents/minimizes the further release of activity into the Secondary Containment ventilation system.

A is Incorrect but plausible. A significantly elevated CHARMS (Containment High Range Radiation Monitoring System) indication of 50Rihr may lead the applicant to believe a manual scram is warranted (including MSIV closure), however this is not a requirement or condition contained within ABN-26, of which the question asks.

Explanation B is Incorrect but plausible. An Off-Gas Hi-Hi annunciator in alarm does require the crew to manually scram the reactor and close the MSIVs, but only after 15 min. It is true the crew can *tail conservative* and take action before the 15 min has elapsed, however the question asks what is procedurally *required*. After only 10 min a scram and MSIV closure is not yet required by procedure.

D is Incorrect but plausible. Stack Effluent Hi-Hi (>2000 cps on Stack RAGEMS) is significant however ABN-26 addresses procedural actions based on a Main Steam Line radiation level or Off-Gas Hi Hi condition.

Lesson 2621.828.0.033A, Plant Radiation Monitoring Systems Plan 273*10446, Identify and explain system operating controls I Learning indications under all plant operating conditions.

Objective/

References ILT: None LORT: Open Provided Question Source (New, Modified, Bank) New Previous 2 NRC Exams (ILT Only) No Memory or Comprehension Fundamental 1:P Cognitive Knowledge or Analysis Level NUREG 1021 Appendix B: Procedure steps and cautions OCS OPS ILT 13-1 EXAM Page: 159 of 201 13 May 2014

EXAMINATION ANSWER KEY ILT 13-1 NRC Written Exam (RO) 10CFR55 55.41b I 10 I 55.43b I

Content Administrative, normal, abnormal, and emergency operating procedures for the facility.

Justification for LORT questions with KIA values< 3.0 I N/A Time to Complete: 1-2 minutes Point Value: 1 System 10 No.: 295034 PRA: No Safety Function(s): 9 ~ ILT Category(s) (LORT Only): N/A 0 LORT OCS OPS ILT 13-1 EXAM Page: 160 of 201 13 May 2014

EXAMINATION ANSWER KEY ILT 13~1 NRC Written Exam (RO}

61 10: 13*1 NRO 61 Points: 1.00 The following is a partial summary of steps contained in the temperature leg of the Secondary Containment Control EOP:

IF A PRIMARY SYSTEM IS DISCHARGING INTO SECONDARY CONTAINMENT

1. BEFORE TEMPERATURE REACHES A MAXSAFEVALUETHENENTERRPV CONTROL- NO ATWS
2. IF TEMPERATURE EXCEEDS A MAX SAFE VALUE IN 2 AREAS, THEN EMERGENCY DEPRESSURIZE lAW the EOP Users Guide, which of the following states the bases for Emergency Depressurization above?
1. It places the RPV in the lowest energy state
2. It reduces the driving head on primary systems discharging into the Secondary Containment
3. It allows RPV injection from low pressure systems to makeup for the primary system leak A. 1 ONLY B. 1 AND 2 C. 2 AND 3 D. 1, 2 AND 3 Answer: B Answer Explanation Knowledge and Ability Reference Information Importance Rating K&A RO I SRO OCS OPS ILT 13*1 EXAM Page: 161 of 201 13 May 2014

EXAMINATION ANSWER KEY ILT 13-1 NRC Written Exam (RO) 295032 High Secondary Containment Area Temperature I 5 EK3.01 - Knowledge of the reasons for the following responses as they apply to HIGH SECONDARY 3.5 3.8 CONTAINMENT AREA TEMPERATURE :

Emergency/normal depressurization Level RO I Tier J 1 I Group I 2 General EOP Users Guide References lAW the EOP Users Guide, the temperature increases is so wide spread that is poses a direct threat to secondary containment integrity, equipment located in the secondary containment or continued safe operation.

ED will place the plant in its lowest energy state and will reduce the driving head and flow from primary systems that are discharging into the secondary containment. Answer B (1 and 2)

Explanation is Correct and answer A (1 only) is Incorrect.

It is true that lowering RPV pressure will make alternate, low pressure systems available for RPV injection, but it is not the bases for the ED. Answer C is Incorrect (2 and 3).

Answer D (1, 2, and 3) is Incorrect but plausible for the reasons stated above.

Lesson 2621.845.0.0011, Secondary Containment Control Plan SCC-03082, Using the Secondary Containment Control EOP, Learning evaluate the technical basis for each step and apply this Objective/ evaluation to determine the correct course of action under emergency conditions.

References ILT: None LORT: Open Provided Question Source (New, Modified, Bank) Modified Previous 2 NRC Exams (ILT Only) No Memory or Comprehension Fundamental 1:B Cognitive or Analysis Knowledge Level NUREG 1021 Appendix B: Bases or purpose OCS OPS ILT 13-1 EXAM Page: 162 of 201 13 May 2014

EXAMINATION ANSWER KEY ILT 13~1 NRC Written Exam (RO) 55.41b I 5 I 55.43b I Facility operating characteristics during steady state and transient 10CFR55 conditions, including coolant chemistry, causes and effects of Content temperature, pressure and reactivity changes, effects of load changes, and operating limitations and reasons for these operating characteristics.

Justification for LORT questions with KIA values< 3.0 I N/A Time to Complete: 1-2 minutes Point Value: 1 System 10 No.: 295032 PRA: No Safety Function(s): 5 ~ ILT Category(s) (LORT Only): NIA 0 LORT OCS OPS ILT 13-1 EXAM Page: 163 of 201 13 May 2014

EXAMINATION ANSWER KEY ILT 13~1 NRC Written Exam (RO) 62 10: 13-1 NRO 62 Points: 1.00 The plant was at rated power making preparations to shutdown due to a loss of USS-1B2. An event then occurred resulting in a small leak in the Drywell. The following plant parameters were observed:

  • Drywell Pressure is 1.7 psig

The US has ordered the BOP to perform Support Procedure 27 (SP-27), Maximizing Drywell Cooling.

Which of the following Drywell Recirc Fans will be running following the completion of SP-27?

1. DWRECIRCFAN1-1
2. OW RECIRC FAN 1-2
3. OW RECIRC FAN 1-3
4. OW RECIRC FAN 1-4
5. OW RECIRC FAN 1-5 A 1 and 2 ONLY B. 4 and 5 ONLY C. 1, 2, and 3 ONLY D. 1, 2, 3, 4, and 5 Answer: C Answer Explanation Knowledge and Ability Reference Information Importance Rating K&A RO SAO 29501 0 High Drywell Pressure I 5 AA 1.01 - Ability to operate and/or monitor the following as 3.4 3.5 they apply to HIGH DRYWELL PRESSURE : Drywell ventilation/cooling Level RO I Tier I 1 I Group I 2 General EMG-SP27 PCC EOP References OCS OPS ILT 13-1 EXAM Page: 164 of 201 13 May 2014

EXAMINATION ANSWER KEY ILT 13~1 NRC Written Exam (RO)

Cis Correct. The normal system lineup for Drywell Recirc fans are the 1*1, 1-2, 1*4, & 1*5 fan in operation with the 1-3 fan off.

The question states there is a loss of USS-1 82. The power supply to Drywell recirc fans 1-4, & 1-5 is MCC-1 823 via USS-1 82, therefore Drywell Recirc fans 1-4, & 1-5 do not have power available to operate. Only Drywell Recirc fans 1-1, 1-2, & 1-3 are available for Drywell cooling.

Cis Correct. The normal system lineup for Drywell Recirc fans are the 1-1, 1*2, 1*4, & 1*5 fan in operation with the 1-3 fan off.

The question states there is a loss of USS-1A2. The power supply to Drywell recirc fans 1-1, 1-2, & 1-3 is MCC-1A23 via uss-Explanation 1A2, therefore Drywell Recirc fans 1*1, 1-2, & 1-3 do not have power available to operate. Only Drywell Recirc fans 1*4 & 1-5 are available for Drywell cooling.

A, 8, & D are Incorrect but plausible if the applicant does not recall the actions required by SP-27 or recall the power supply to the drywell cooling pumps. Drywell Recirc fans 1-4, & 1-5 are powered from MCC-1 823 via USS-1 82. Since USS-1 82 has been lost, those fans are not operable. Answers A, B & D include one or more of Drywell Recirc fans 1-4, & 1-5 as a choice, therefore which is incorrect. The only fans that have power available are Drywell Recirc fans 1-1, 1-2, & 1-3.

Lesson 2621.845.0.0056, Primary Containment Control Plan PCC-10445, Given a set of system indications or data, evaluate and interpret them to determine limits, trends and system status.

Learning Objective/

References ILT: None LORT: Open Provided Question Source (New, Modified, Bank) Modified Previous 2 NRC Exams (ILT Only) No Memory or Comprehension Fundamental 2:RI Cognitive Knowledge or Analysis Level NUREG 1021 Appendix 8: Recognizing interaction between systems (plural), including consequences and implications 55.41b 7 55.43b 10CFR55 Design, components, and functions of control and safety systems, Content including instrumentation, signals, interlocks, failure modes, and automatic and manual features.

OCS OPS ILT 13-1 EXAM Page: 165 of 201 13 May 2014

EXAMINATION ANSWER KEY ILT 13*1 NRC Written Exam (RO)

Justification for LORT questions with KlA values < 3.0 I N/A Time to Complete: 1*2 minutes Point Value: 1 System 10 No.: 295010 PRA: No Safety Function(s): 5 [81 ILT Category(s) (LORT Only): N/A 0 LORT OCS OPS ILT 13-1 EXAM Page: 166 of 201 13 May 2014

EXAMINATION ANSWER KEY ILT 13~1 NRC Written Exam (RO) 63 ID: 13*1 NRO 63 Points: 1.00 The plant was at rated power when the MSIVs inadvertently and SIMULTANEOUSLY closed. The Operator makes the following report:

  • EMRVs are cycling open (for 12 seconds) and closed (for 15 seconds)
  • ISOL CONDENSER SHELL A AND B indicate 205 °F and rising
  • The EDGs are in standby Based on the above ... (Assume NO Operator actions)

A. the plant responded normally.

B. an ATWS has occurred with power < 2%.

C. an ATWS has occurred with power> 2%.

D. Reactor power can be reduced with Recirculation flow AND control rods.

Answer: C Answer Explanation Knowledge and Ability Reference Information Importance Rating K&A RO SRO 295007 High Reactor Pressure I 3 AA2.02

  • Ability to determine and/or interpret the following 4.1 4.1 as they apply to HIGH REACTOR PRESSURE : Reactor power Level RO I Tier I 1 I Group I 2 General 420 USAR Table 5.2*1 References OCSOPS ILT 13-1 EXAM Page: 167 of 201 13 May 2014

EXAMINATION ANSWER KEY ILT 13-1 NRC Written Exam (RO)

The reactor was at rated power when all MSIVs closed. This should have resulted in a reactor scram. But the indications show otherwise: EMRVs are cycling on high RPV pressure, and both isolation condensers are in service (as evidenced by the high shell water temperature). The isolation condensers automatically initiate on high RPV pressure (1 060 psi g) or RPV water level low-low (86"). The ICs must have initiated on RPV high pressure since the EDGs are in standby (EDGs will idle start on a high OW pressure or RPV water level low-low; since there are no indications of any primary containment problems, then there must be NO RPV water level low-low condition).

Explanation C is Correct and A & B are Incorrect but plausible. Each IC can take about 3% power each of steam. With both in service, and with EMRVs cycling, then power is at least 6%. Therefore, an ATWS has occurred with power still above 2%.

Dis Incorrect but plausible. It is true that terminating recirculation flow and inserting control rods during an ATWS can reduce power. But 3 recirculation pumps trip immediately on 1060 psig RPV pressure and the other 2 pumps trip if the high pressure is sustained for > 1.05 seconds. Therefore, all recirculation pumps have already tripped since EMRVs are open under high pressure conditions for 12 seconds (the lowest EMRVs open at > 1060 psi g).

Lesson 2621.845.0.001 B, RPV Control with ATWS Plan EWA-3055A, Given a copy of RPV Control With ATWS, describe in Learning detail each step or conditional statement, including technical Objective/ basis, and how to perform each step as required.

References ILT: None LORT: Open Provided Question Source (New, Modified, Bank) Bank Previous 2 NRC Exams (ILT Only) No Memory or Comprehension Fundamental 3:SPK Cognitive or Analysis Knowledge Level NUREG 1021 Appendix B: Solve a Problem using Knowledge and its meaning OCS OPS ILT 13-1 EXAM Page: 168 of 201 13 May 2014

EXAMINATION ANSWER KEY ILT 13~1 NRC Written Exam (RO) 10CFR55 55.41b l 7 I 55.43b I

Design, components, and functions of control and safety systems, Content including instrumentation, signals, interlocks, failure modes, and automatic and manual features.

Justification for LORT questions with KIA values< 3.0 I N/A Time to Complete: 1*2 minutes Point Value: 1 System I D No.: 295007 PRA: No Safety Function(s): 3 ~ ILT Category(s) (LORT Only): NIA 0 LORT OCSOPS ILT 13-1 EXAM Page: 169 of 201 13 May 2014

EXAMINATION ANSWER KEY ILT 13~1 NRC Written Exam (RO) 64 10: 13*1 NRO 64 Points: 1.00 Refueling was in progress when an accident occurred on the refuel floor. The Standby Gas Treatment System initiated following a 2-min time delay when sensed radiation levels were above the setpoint.

Which of the following states the logic necessary for the SGTS automatic start described above and is also a Secondary Containment Control EOP entry condition?

A. Radiation levels~ 9 mr/hr as sensed by BOTH RB Vent Radiation Monitor.

B. Radiation levels~ 50 mr/hr as sensed by Refuel Floor Radiation Monitor C-9.

C. Radiation levels~ 9 mr/hr as sensed by EITHER RB Vent Radiation Monitor.

D. Radiation levels~ 50 mr/hr as sensed by Fuel Pool Filter Area Radiation Monitor C-2.

Answer: B Answer Explanation Knowledge and Ability Reference Information Importance Rating K&A RO SRO 295033 Secondary Containment Area Radiation Levels I 9 2.4.2- Emergency Procedures/Plan: Knowledge of system 4.5 4.6 set points, interlocks and automatic actions associated with EOP entry conditions.

Level RO I Tier I 1 I Group I 2 General References sec EOP RAP*10F3m RAP-10F1f OCS OPS ILT 13-1 EXAM Page: 170 of 201 13 May 2014

EXAMINATION ANSWER KEY ILT 13~1 NRC Written Exam (RO)

SGTS will auto start from ~9 Mr/hr on EITHER RB vent radiation monitor (with no time delay), OR high radiation (50 Mr/hr) on the refuel floor rad monitors (with a 2-minute time delay) on 8*9 or C*

9. Since there was a SGTS start following a 2-min time delay, the refuel floor rad monitor B-9 must have sensed rad levels above its setpoint. Therefore, only one RB vent radiation monitor had to reach its setpoint of 9 mr/hr to automatically auto start SGTS and also a Secondary Containment Control EOP entry condision.

Explanation Answer B is Correct.

A and Care Incorrect but plausible if the applicant does not recall SGTS logic.

D is Incorrect but plausible since it is a setpoint most radiation monitors alarm at, however it does not start the SGTS and it is not an EOP entry condition.

Lesson 2621.828.0.0042, Secondary Containment and SGTS Plan 261*10445, Given a set of indications or date, evaluate and Learning interpret them to determine limits, trends and system status.

Objective/

References ILT: None LORT: Open Provided Question Source (New, Modified, Bank) New Previous 2 NRC Exams (ILT Only) No Memory or Comprehension Fundamental 2:DR Cognitive or Analysis Knowledge Level NUREG 1021 Appendix B: Describing or recognizing Relationships 55.41b 7 55.43b 10CFR55 Design, components, and functions of control and safety systems, Content including instrumentation, signals, interlocks, failure modes, and automatic and manual features.

Justification for LORT questions with KIA values< 3.0 I N/A Time to Complete: 1*2 minutes Point Value: 1 System ID No.: 295033 PRA: No Safety Function(s): 9 ~ ILT Category(s) (LORT Only): N/A 0 LORT OCS OPS ILT 13*1 EXAM Page: 171 of 201 13 May 2014

EXAMINATION ANSWER KEY ILT 13*1 NRC Written Exam (RO) 65 10: 13-1 NRO 65 Points: 1.00 The plant was at rated power when condenser vacuum began to degrade uncontrollably.

Which of the following is correct as vacuum continues to degrade?

When condenser vacuum degrades to (1) inches Hg, the (2) will close to prevent over*pressurizing the condenser.

ill A. 10 MSIVs B. 10 Turbine Bypass Valves C. 20 Turbine Bypass Valves D. 20 MSIVs Answer: B Answer Explanation Knowledge and Ability Reference Information Importance Rating K&A RO SRO 295002 Loss of Main Condenser Vacuum /3 AK3.04 - Knowledge of the reasons for the following 3.4 3.6 responses as they apply to LOSS OF MAIN CONDENSER VACUUM : Bvpass valve closure.

Level RO I Tier I 1 I Group J 2 General USAR 10.2.2.2.4 References OCS OPS ILT 13-1 EXAM Page: 172 of 201 13 May 2014

EXAMINATION ANSWER KEY ILT 13-1 NRC Written Exam (RO)

As condenser vacuum lowers, its ability to function as the ultimate heat sink also drops. At 22", a turbine trip and a scram signal are generated. At 10: vacuum, the turbine bypass valves are auto closed to prevent a main condenser over-pressure condition. When the condenser is over-pressurized, the Explanation condenser will relieve to the Turbine Building (atmospheric reliefs function at 5 psig). Answer B is Correct.

All distractors are plausible but Incorrect if the applicant does not recall the correct setpoint or bases.

Lesson 2621.828.0.0051, Turbine Controls Plan TCS-10441: Given the system logic/electrical drawings, describe the system trip signals, set-points and expected system response Learning including power loss or failed components.

Objective/ TCS-10446: Identify and explain system operating controls/indications under all llant operating conditions.

References ILT: None LORT: Open Provided Question Source (New, Modified, Bank) Bank Previous 2 NRC Exams (ILT Only) No Memory or Comprehension Fundamental 1:1 & B Cognitive or Analysis Knowledge Level NUREG 1021 Appendix B: Interlocks, setpoints, or system (singular) response & .B_ases 55.41b 7 55.43b 10CRF55 Design, components, and functions of control and safety systems, Content including instrumentation, signals, interlocks, failure modes, and automatic and manual features.

Justification for LORT questions with KIA values< 3.0 I N/A Time to Complete: 1-2 minutes Point Value: 1 System ID No.: 295002 PRA: No Safety Function(s): 3 181 ILT Category(s) (LORT Only): N/A 181 LORT OCS OPS ILT 13-1 EXAM Page: 173 of 201 13 May 2014

EXAMINATION ANSWER KEY ILT 13~1 NRC Written Exam (RO) 66 10: 13*1 NRO 66 Points: 1.00 The plant was at rated power when an Electric ATWS occurred.

lAW OP-OC-101-111-1001, Strategies for Successful Transient Mitigation, which ONE of the following tasks can be performed by the URO without being directed by the US and without immediate procedure reference?

A. Place the ADS Timers in BYPASS B. Place the ROPS switch in BYPASS C. Trip ALL Reactor Recirculation Pumps D. Manually commence driving rods that failed to scram Answer: B Answer Explanation Knowledge and Ability Reference Information Importance Rating K&A RO SRO Conduct of Operations 3.8 4.2 2.1.1 - Knowledge of conduct of operations requirements.

Level RO I Tier I 3 I Group I N/A General OP-OC-101-111*

References 1001 8 is Correct. lAW OP-OC-101-111-1001, in the ATWS Scram Choreography section, one of the actions the URO shall immediately take is to place the ROPS (Reactor Overfill Protection System) switch from NORMAL to BYPASS.

Explanation All distractors are Incorrect but plausible since they are all actions that will be preformed shortly after the ATWS situation commences, however all the distractors are actions the US will direct.

Lesson 2621.845.0.0018, RPV Control with ATWS (Electric)

Plan EWA-03055, Given a copy of RPV Control, describe in detail each Learning step or conditional statement, including technical basis, and how Objective/ to perform each step as required OCS OPS ILT 13-1 EXAM Page: 174 of 201 13 May 2014

EXAMINATION ANSWER KEY ILT 13~1 NRC Written Exam (RO)

References ILT: None LORT: Open Provided Question Source (New, Modified, Bank) New Previous 2 NRC Exams (ILT Only) No Memory or Comprehension Fundamental 1:P Cognitive or Analysis Knowledge Level NUREG 1021 Appendix B: Procedure steps and cautions 55.41b 10 55.43b 10CFR55 Content Administrative, normal, abnormal, and emergency operating procedures for the facility.

Justification for LORT questions with KIA values< 3.0 I NIA Time to Complete: 1*2 minutes Point Value: 1 System ID No.: N/A PRA: No Safety Function(s): NIA ~ ILT Category(s) (LORT Only): NIA 0 LORT OCS OPS ILT 13-1 EXAM Page: 175 of 201 13 May 2014

EXAMINATION ANSWER KEY ILT 13-1 NRC Written Exam (RO) 67 10: 13*1 NRO 67 Points: 1.00 The plant was at 80% power with recirculation pump C in local manual control. An event occurred which resulted in the automatic trip of Recirculation Pump C. The following plant conditions exist:

  • Reactor power indicates 1350 MWth
  • The Operator was required to close Recirculation Pump C suction valve lAW ABN-2
  • All Drywell parameters indicate normal Which of the following states who is REQUIRED to be notified (lAW procedure) and the action that would be requested of those notified?

Who Notified Requested Action A. Chemistry Perform an offgas sample B. Field Supervisor Equalize flows in operating loops C. Electrical Maintenance To close Recirculation Pump C discharge valve D. Field Supervisor Place Recirculation MG Set C back in Remote-Manual Answer: C Answer Explanation Knowledge and Ability Reference Information Importance Rating K&A RO SRO Conduct of Operations 2.1.8 - Ability to coordinate personnel activities outside 3.4 4.1 the control room.

Level RO I Tier I 3 J Group J NIA General ABN-2 References OCS OPS ILT 13-1 EXAM Page: 176 of 201 13 May 2014

EXAMINATION ANSWER KEY ILT 13~1 NRC Written Exam (RO)

A is Incorrect but plausible. The indication show that reactor power was reduced 194 MWth by the tripped recirc pump. lAW 202.1, Power Operation, chemistry must be notified to perform a reactor coolant sample when power changes by 289.5 MWth (15%

power), not an offgas sample. Since power only changes by 194 MWth, no notification is procedurally required.

B is Incorrect but plausible since the Field Supervisor is licensed at Oyster Creek and could equalize flow, however the action is incorrect; the applicant may not diagnose the recirc pump tripped.

C is Correct. The question states that the ABN has been entered, Explanation which in this case, is ABN-2, Recirculation System Failures. For this case, the recirc. pump discharge should be closed. If it can't, then shut the suction valve and request EM to aid in closing the discharge valve. Since all OW parameters are normal, there is no leakage type problems which would have caused a recirc pump isolation (which includes closing the suction valve). Therefore, the only reason to close the pump suction valve is because the pump discharge valve won't close. Because the pump is tripped, there is no necessity to place it back in auto.

D is Incorrect but plausible. The action is correct if the crew is attempting to prepare the Recirc pump for a normal start, however is not required as an action at per the ABN.

Lesson 2621.828.0.0038, Reactor Recirculation System Plan Learning Objective/ RRS-10450, Describe and interpret procedure sections and steps for plant emergency or off-normal conditions that involve this system including personnel allocation and equipment operation lAW applicable ABN, EOP & EOP support procedures and EP procedures.

References ILT: None LORT: Open Provided Question Source (New, Modified, Bank) New Previous 2 NRC Exams (ILT Only) No OCS OPS ILT 13-1 EXAM Page: 177 of 201 13 May 2014

EXAMINATION ANSWER KEY ILT 13~1 NRC Written Exam (RO)

Memory or Comprehension Fundamental 3:SPK Cognitive or Analysis Knowledge Level NUREG 1021 Appendix 8: Solve a Problem using Knowledge and its meaning 55.41b 10 55.43b 10CFR55 Design, components, and functions of control and safety systems, Content including instrumentation, signals, interlocks, failure modes, and automatic and manual features.

Justification for LORT questions with KlA values < 3.0 I N/A Time to Complete: 1-2 minutes Point Value: 1 System 10 No.: NIA PRA: No Safety Function(s): N/A 181 ILT Category(s) (LORT Only): NIA 0 LORT OCS OPS ILT 13-1 EXAM Page: 178 of 201 13 May 2014

EXAMINATION ANSWER KEY ILT 13*1 NRC Written Exam (RO) 68 10: 13*1 NRO 68 Points: 1.00 The plant is starting up after a refuel outage.

Which of the following states who can manipulate Reactor Controls?

A An Equipment Operator candidate who is being directly supervised by an active licensed operator.

B. An active licensed operator with a corrective lenses license restriction who does not have his glasses.

C. An inactive licensed operator who is reactivating and who is being directly supervised by an active licensed operator.

D. A Reactor engineer who has been selected for the next initial license training class who is being directly supervised by an active licensed operator.

Answer: C Answer Explanation Knowledge and Ability Reference Information Importance Rating K&A RO SAO Equipment Control 2.2.38

  • Knowledge of conditions and limitations in the 3.6 4.5 facility license.

Level RO I Tier I 3 J Group J NIA General OP*AA-1 03-1 03 TS 6.2.2 OP-AA-1 05*1 02 References OP-AA-1 03-1 02 10CFR55.13 OP-AA-105-101 OCS OPS ILT 13-1 EXAM Page: 179 of 201 13 May 2014

EXAMINATION ANSWER KEY ILT 13~1 NRC Written Exam (RO)

The process for maintaining the configuration and status of reactor controls is related to who can change or manipulate the reactor controls. Procedures allow only certain individuals to manipulate the reactor controls (apparatus and mechanisms that the manipulation of would directly affect the reactivity or power level of the reactor).

Cis Correct. lAW the OP-AA-103-103, an inactive licensed operator must be enrolled in a license reactivation program to perform main control room manipulations. lAW OP-AA-105-102, the hours spent shift functions will be performed in the presence and under the direct supervision of an active RO or SRO.

Explanation Therefore, the inactive operator must be reactivating and under the direct supervision of an active operator.

B is Incorrect. It is the operators' responsibility to meet his license restrictions. The operator is only allowed to perform license functions when license restrictions are met.

A & D are Incorrect. Only a trainee enrolled in a license training program can manipulate the controls, while under the direct supervision of a licensed operator. In answer A, the candidate is an EO candidate not a license candidate. In answer D, the system engineer is not yet currently enrolled in the initial license training program.

Lesson 2621.830.0.0018, Equipment Control - Admin Plan 2.2.14, Knowledge of the process for controlling equipment configuration or status.

Learning Objective/

References ILT: None LORT: Open Provided Question Source (New, Modified, Bank) Bank Previous 2 NRC Exams (ILT Only) No Memory or Comprehension Fundamental 1:P Cognitive or Analysis Knowledge Level NUREG 1021 Appendix B: Procedure steps and cautions 55.41b 10 55.43b 10CFR55 Content Administrative, normal, abnormal, and emergency operating procedures for the facility.

Justification for LORT questions with KIA values< 3.0 I NIA Time to Complete: 1-2 minutes I Point Value: I 1 OCS OPS ILT 13*1 EXAM Page: 180 of 201 13 May 2014

EXAMINATION ANSWER KEY ILT 13-1 NRC Written Exam (RO)

System ID No.: NIA PRA: I No Safety Function(s): NIA 181 ILT Category(s) (LORT Only): N/A 0 LORT OCSOPS ILT 13-1 EXAM Page: 181 of 201 13 May 2014

EXAMINATION ANSWER KEY ILT 13~1 NRC Written Exam (RO) 69 10: 13*1 NRO 69 Points: 1.00 While operating at rated power, ALL Turbine Control Valves and Turbine Bypass Valves fail OPEN.

Which ONE of the following scram functions prevents exceeding the Fuel Cladding Integrity Safety Limit? (Assume NO operator actions)

A. MSIV closure B. RPV water level C. Flow~biased APRM D. Reactor high pressure Answer: A Answer Explanation Knowledge and Ability Reference Information Importance Rating K&A RO SRO Equipment Control 2.2.22 - Knowledge of limiting conditions for operations 4.0 4.7 and safety limits.

Level RO I Tier I 3 I Group I N/A General Turbine Tech UFSAR 15.1.3 TS 2.3 References Manual Tab 10 OCS OPS ILT 13*1 EXAM Page: 182 of 201 13 May 2014

EXAMINATION ANSWER KEY ILT 13*1 NRC Written Exam (RO)

A is Correct. If the EPR relay strokes to 100%, the turbine control and bypass valves will open to their mechanical stops, resulting in a decrease in reactor pressure. At "'850 psig, with the mode switch in RUN (no operator actions) the MSIV's will close, resulting in a reactor scram on MSIV position. These two actions (MSIV closure and reactor scram) prevent exceeding the low-pressure portion of the Fuel Cladding Integrity Safety Limit.

B is Incorrect but plausible. Initially, reactor water level will increase due to swell. As additional water inventory is lost due to Explanation higher steam flow, water level will return to the normal band as the Feedwater Control System responds to the transient. The MSIV closure scram will occur before RPV water can lower to the low-level scram setpoint.

C is Incorrect but plausible. Reactor power will lower as a result of this transient, increasing the margin between actual power and the APRM flow-biased scram setpoint.

D is Incorrect but plausible if the applicant does not recognize that reactor pressure will lower as a result of this transient.

Lesson 2621.850.0.0090, Overview/Highlights of Technical Specifications Plan TSX-01658, State requirements associated with given areas of Learning technical specifications (safety) limits, LSSS, etc)

Objective/

References ILT: None LORT: Open Provided Question Source (New, Modified, Bank) Bank Previous 2 NRC Exams (ILT Only) No Memory or Comprehension Fundamental 1:F Cognitive or Analysis Knowledge Level NUREG 1021 Appendix B: Facts 55.41b 7 55.43b 10CFR55 Design, components, and functions of control and safety systems, Content including instrumentation, signals, interlocks, failure modes, and automatic and manual features.

Justification for LORT questions with KIA values< 3.0 I N/A Time to Complete: 1-2 minutes I Point Value: I 1 OCS OPS ILT 13-1 EXAM Page: 183 of 201 13 May 2014

EXAMINATION ANSWER KEY ILT 13~1 NRC Written Exam (RO)

System I D No.: NIA PRA: I No Safety Function(s): N/A ~ ILT Category(s) (LORT Only): N/A 0 LORT OCS OPS ILT 13-1 EXAM Page: 184 of 201 13 May 2014

EXAMINATION ANSWER KEY ILT 13-1 NRC Written Exam (RO) 70 10: 13*1 NRO 70 Points: 1.00 Which of the following conditions REQUIRES that Radiation Protection be notified to assess any radioactive releases including the release of tritium?

A. Initiating Torus Cooling during EOPs.

B. Initiating Drywell Sprays during EOPs.

C. Venting the Primary Containment lAW the Hardcard.

D. Initiating the Isolation Condensers during a cooldown.

Answer: D Answer Explanation Knowledge and Ability Reference Information Importance Rating K&A RO SRO Radiation Control 2.3.13 - Knowledge of Radiological Safety Procedures pertaining to licensed operator duties, such as response 3.4 3.8 to radiation monitor alarms, containment entry requirements, fuel handling responsibilities, access to locked high radiation areas, aligning filters, etc.

Level RO I Tier I 3 I Group I N/A General 307 P&L 4.2.7 References OCS OPS ILT 13-1 EXAM Page: 185 of 201 13 May 2014

EXAMINATION ANSWER KEY ILT 13~1 NRC Written Exam (RO)

Dis Correct. lAW procedure 307, any time that the Isolation Condensers are placed in service by either manual or automatic initiation, Rad Pro shall be notified to assess, minimize and contain any radioactive releases, including the release of tritium.

Using Isolation Condensers during a cooldown from a plant scram has occurred in the past. Operation of the Isolation Condenser System is a Licensed Operator duty.

A is Incorrect but plausible. Initiating Torus Cooling during EOPs is performed when Torus Temperature rises >95F, and could indicate an EMRV lifting or LOCA.

Explanation B is Incorrect but plausible. Initiating Drywell Sprays during EOPs is performed when Drywell pressure or temperature is elevated, usually from a LOCA. Initiating Drywell Sprays requires the use of the ESW pumps. A tube leak in the Containment Spray heat exchanger could release radioactivity to the ESW System and then out to the environment. There is no procedural requirement to notify RP during Drywell Sprays.

C is Incorrect but plausible. Venting the Drywell using the hardcard will allow primary containment atmosphere to be release directly to the outside atmosphere through the normal RB Vent System, but there is no requirement to notify RP to assess radioactivity.

Lesson 2621.828.0.0023, Isolation Condensers Plan ICS-02338, Given plant conditions, evaluate the impact on the Learning Isolation Condenser System and the plant.

Objective/

References ILT: None LORT: Open Provided Question Source (New, Modified, Bank) Bank Previous 2 NRC Exams (ILT Only) No Memory or Comprehension Fundamental 1:P Cognitive Knowledge or Analysis Level NUREG 1021 Appendix B: Procedure steps and cautions OCS OPS ILT 13-1 EXAM Page: 186 of 201 13 May 2014

EXAMINATION ANSWER KEY ILT 13-1 NRC Written Exam (RO) 10CFR55 55.41b I 12 I 55.43b 1 Content Radiological safety principles and procedures.

Justification for LORT questions with KIA values< 3.0 I N/A Time to Complete: 1*2 minutes Point Value: 1 System ID No.: NIA PRA: No Safety Function(s): NIA ~ ILT Category(s) (LORT Only): NIA 0 LORT OCS OPS ILT 13-1 EXAM Page: 187 of 201 13 May 2014

EXAMINATION ANSWER KEY ILT 13-1 NRC Written Exam (RO) 71 10: 13*1 NRO 71 Points: 1.00 An Operator is needed to perform a filter alignment in an area where the general area radiation level is 60 mR/hr.

  • The job will take 30 minutes for one operator, but can be completed in 20 minutes with two operators.
  • Area radiation levels could be reduced to 30 mR/hr with shielding.
  • It will take 20 minutes for one individual to install shielding.

To ensure the job dose is maintained "As Low As Reasonably Achievable",

(1) operator(s) should perform the lineup (2) shielding.

Condider TOTAL personnel dose only. Assume NO dose is recieved in transit to and from the job site.

A. (1) one (2) with B. (1) one (2) without C. (1) two (2) with D. (1) two (2) without Answer: 8 Answer Explanation Knowledge and Ability Reference Information Importance Rating K&A RO SRO Radiation Control 2.3.12 - Knowledge of Radiological Safety Principles pertaining to licensed operator duties, such as 3.2 3.7 containment entry requirements, fuel handling responsibilities, access to locked high-radiation areas, aligning filters, etc.

Level RO I Tier I 3 I Group I NIA General RP-AA-400 References OCS OPS ILT 13-1 EXAM Page: 188 of 201 13 May 2014

EXAMINATION ANSWER KEY ILT 13~1 NRC Written Exam (RO)

B is Correct. 1 individual to perform lineup without shielding is a total exposure of 30 mR. (30 minutes in a 60 mR/hr field).

All distractors are Incorrect but plausible if the applicant does not correctly calculate the stay time with ALARA.

A is Incorrect. 1 individual to install shielding is 20 mR (20 minutes in a 60 mR/hr field), plus one individual to perform the lineup is 15 mR (30 minutes in a 30 mR/hr field), for a total Explanation exposure of 35 mR/hr.

C is Incorrect. 1 individual to install shielding is 20 mR/hr (20 minutes in a 60 mR/hr field), plus two individuals to perform lineup is 20 mR (20 minutes in a 30 mR/hr field, times 2), for a total exposure of 40 mR.

D is Incorrect. 2 individuals to perform alineup without sheilding is a total exposure of 40 mR (20 minutes in a 60 mR/hr field, times 2).

Lesson 2621.830.0.0003, Radiation Control - Admin Plan 2.3.12, Knowledge of Radiological Safety Principles pertaining to licensed operator duties, such as containment entry Learning requirements, fuel handling responsibilities, access to locked Objective/ high-radiation areas, aligning filters, etc.

References ILT: None LORT: Open Provided Question Source (New, Modified, Bank) Bank Previous 2 NRC Exams (ILT Only) No Memory or Comprehension Fundamental 3:SPK Cognitive or Analysis Knowledge Level NUREG 1021 Appendix B: Solve a Problem using Knowledge and its meaning 55.41b 12 55.43b 10CRF55 Content Radiological safety principles and procedures.

Justification for LORT questions with KIA values< 3.0 I NIA Time to Complete: 1-2 minutes Point Value: 1 System ID No.: NIA PRA: No Safety Function(s): NIA ~ ILT OCSOPS ILT 13-1 EXAM Page: 189 of 201 13 May 2014

EXAMINATION ANSWER KEY ILT 13-1 NRC Written Exam (RO)

I Category(s) (LORT Only): I NIA lo LORT OCS OPS ILT 13-1 EXAM Page: 190 of 201 13 May 2014

EXAMINATION ANSWER KEY ILT 13~1 NRC Written Exam (RO) 72 10: 13*1 NRO 72 Points: 1.00 The plant was at rated power with the following conditions and indications:

  • EMRV NR108A keylock switch is in DISABLE
  • EMRV NR108B keylock switch is in DISABLE
  • Torus temperature is 82 °F and steady
  • Generator MWe indicates 655 MWe An event then occurs which requires an Emergency Depressurization -with ATWS.

Accordingly, ALL EMRV switches on Panel 1F/2F were placed in MAN. (Assume EMRVs in DISABLE remain in DISABLE) lAW the EOP Users Guide, which of the following is the LOWEST RPV pressure to satisfy the Minimum Steam Cooling Pressure (MSCP) for the given conditions to ensure adequate core cooling?

Table 7 NUMBER RPV OF OPEN PRESSURE EMRVS (PSIG) 5 230 4 290 3 390 2 590 A. 240 psig B. 300 psig

c. 400 psig D. 600 psig Answer: c Answer Explanation Knowledge and Ability Reference Information Importance Rating K&A RO SAO Emergency Procedures I Plan 3.9 4.3 2.4.17 - Knowledge of EOP terms and definitions.

Level I RO I Tier I 3 I Group I N/A OCS OPS ILT 13-1 EXAM Page: 191 of 201 13 May 2014

EXAMINATION ANSWER KEY ILT 13~1 NRC Written Exam (RO)

General RPVC- with ATWS EOP User's Guide References C is Correct. The question stem shows that 2 EMRVs have been disabled, which means that they will not open from the control room panel 1F/2F. Indications are provided which show that the disabled EMRVs are closed {torus temperature not rising and generator load not lower than expected for rated power). Thus, the disabled EMRVs are closed and cannot be opened from their control switches on panel1 F/2F. Therefore, all other EMRVs were opened {when placed in MAN), for a total of 3 EMRVs opened. The MSCP associated with 3 open EMRVs is 390 psig. As long as RPV Explanation pressure is at or above the MSCP, then adequate core cooling is assured. Answer C is correct since 400 psig is greater than the minimum steam cooling pressure of 390 psig. The applicant must have knowledge of the MSCP definition to answer the question.

All distractors are Incorrect but plausible if the applicant does not understand the basis or does not understand the impact of the DISABLE switches.

Lesson 2621.845.0.0005, Steam Cooling Plan ESC-03004, Given a copy of Steam Cooling, describe in detail Learning each step or conditional statement including the technical basis Objective/ and how to verify or perform each step as required.

References ILT: None LORT: Open Provided Question Source {New, Modified, Bank) Bank Previous 2 NRC Exams {ILT Only) No Memory or Comprehension Fundamental 1:P Cognitive or Analysis Knowledge Level NUREG 1021 Appendix B: Procedure steps and cautions 55.41b 10 55.43b 10CFR55 Content Radiological safety principles and procedures.

Justification for LORT questions with KIA values< 3.0 N/A Time to Complete: 1-2 minutes Point Value: 1 System I D No.: NIA PRA: No OCS OPS ILT 13-1 EXAM Page: 192 of 201 13 May 2014

EXAMINATION ANSWER KEY ILT 13~1 NRC Written Exam (RO)

Safety Function(s): NIA 181 ILT Category(s) (LORT Only): N/A 0 LORT OCS OPS ILT 13-1 EXAM Page: 193 of 201 13May2014

EXAMINATION ANSWER KEY ILT 13~1 NRC Written Exam (RO) 73 10: 13*1 NRO 73 Points: 1.00 Given the following plant conditions:

  • The reactor is operating at rated power.
  • Ow3~c, "DEMIN STRNR DP HI" is received and acknowledged.
  • D~2~c, "DEMIN CONDUCT HI" is received and also acknowledged a short time later.
  • CU DEMIN SYS POST STRAINER LlP (DPIS-IJ22B) indication is rising on RK05.

Which ONE of the following conditions would cause these alarms and indications?

A Cleanup filter resin exhaustion.

B. Cleanup filter resin breakthrough.

C. Cleanup demineralizer resin exhaustion.

D. Cleanup demineralizer resin breakthrough.

Answer: D Answer Explanation Knowledge and Ability Reference Information Importance Rating K&A RO SRO Emergency Procedures I Plan 2.4.45 - Ability to prioritize and interpret the significance of 4.1 4.3 each annunciator or alarm.

Level RO I Tier I 3 I Group I NIA General RAP-D2c, D3c ABN-4 References OCS OPS ILT 13-1 EXAM Page: 194 of 201 13 May 2014

EXAMINATION ANSWER KEY ILT 13~1 NRC Written Exam (RO)

D is Correct. ABN-4. There are two D/P alarms associated with the Demineralizer. Demin D/P Hi which alarms at 15 psid to warn the operators of filter problems such as filter breakthrough (inlet to de min). The Demin Strainer D/P alarm comes in at 10 psid to warn the operator of demin breakthrough (outlet from demin) A breakthrough of the demin will result in rising conductivity sensed immediately downstream of the demin. Additionally lAW ABN-4 a rising CU DEMIN SYS POST STRAINER AP (DPIS-IJ22B) on RK05 along with annunciators D-2-c and D-3-c is a positive indication that there is DEMIN BREAKTHROUGH and NOT FILTER BREAKTHROUGH. Resin exhaustion alone in either the Cleanup Explanation Filter or Demin would not give you the indications in the question stem. This question meets the *interpret* portion of the KIA.

A is Incorrect but plausible. Although conductivity would be affected by filter breakthrough it would not result in a DIP alarm.

B is Incorrect but plausible. Filter breakthrough would cause a Demin D/P Hi alarm versus demin strainer D/P Hi.

C is Incorrect but plausible. Although conductivity would be affected by demineralizer breakthrough it would not result in a D/P alarm.

Lesson 2621.828.0.0039, Reactor Water Cleanup Plan RCU-1 0444, Describe the interlock signals and setpoints for the Learning affected system components and expected system response Objective/ including power loss or failed components.

References ILT: None LORT: Open Provided Question Source (New, Modified, Bank) Bank Previous 2 NRC Exams (ILT Only) No Memory or Comprehension Fundamental 1:I Cognitive or Analysis Knowledge Level NUREG 1021 Appendix B: Interlocks, setpoints, or system (singular) response 55.41b 7 55.43b 10CFR55 Design, components, and functions of control and safety systems, Content including instrumentation, signals, interlocks, failure modes, and automatic and manual features.

Justification for LORT questions with KIA values< 3.0 J NIA OCS OPS ILT 13-1 EXAM Page: 195 of 201 13 May 2014

EXAMINATION ANSWER KEY ILT 13~1 NRC Written Exam (RO)

Time to Complete: 1*2 minutes Point Value: 1 System I D No.: NIA PRA: No Safety Function(s): N/A !ZI ILT Category(s) (LORT Only): NIA 0 LORT OCS OPS ILT 13-1 EXAM Page: 196 of 201 13 May 2014

EXAMINATION ANSWER KEY ILT 13~1 NRC Written Exam (RO) 74 10: 13*1 NRO 74 Points: 1.00 Plant is operating at rated power when the following occurs;

  • A steam leak occurs in the Main Steam Line Tunnel with area temperatures between 160 F and 170 F
  • All automatic isolations have occurred as designed
  • Due to the leak location and isolation actions, NO LOCA signal occurred from high drywall pressure or RPV low level Which one of the following will reduce the UNMONITORED release rate?

A. Start the Turbine Building Ventilation System if NOT operating.

B. Shutdown the Turbine Building Ventilation System if operating.

C. Shutdown the Radwaste Building Ventilation System if operating.

D. Start the Trunnion Room recirculating fans if NOT operating.

Answer: A Answer Explanation Knowledge and Ability Reference Information Importance Rating K&A RO SRO Radiation Control 3.8 4.3 2.3.11 - Abilit" to control radiation releases.

Level RO I Tier I 3 I Group I NIA General 328 References A is Correct. The stem describes a condition where a leak in the Main Steam Line Tunnel will occur, which will vent to the Turbine Building Ventilation System (TB HVAC). The TB HVAC system maintains the Turbine Building at a negative pressure, venting through the stack, which is monitored. lAW 328, TB HVAC, the Explanation TB HVAC is monitored when running.

All distractors are Incorrect but plausible if the applicant does not interpret where the release point (location) is and/or is confused on where the release is monitored from.

OCS OPS ILT 13-1 EXAM Page: 197 of 201 13 May 2014

EXAMINATION ANSWER KEY ILT 13~1 NRC Written Exam (RO)

Lesson 2621.828.0.0054, Turbine Building and Mise Ventilation Plan TMV-10437, Without the aid of references, draw and label a Learning sketch of the system flowpaths, including major equipment Objective/ (valves, pumps, instrumentation, etc.) and showing interconnections with other s~stems.

References ILT: None LORT: Open Provided Question Source (New, Modified, Bank) Bank Previous 2 NRC Exams (ILT Only) No Memory or Comprehension Fundamental 3:SPK Cognitive or Analysis Knowledge Level NUREG 1021 Appendix B: .S.olve a Problem using Knowledge and its meaning 55.41b 12 55.43b 10CFR55 Content Radiological safety principles and procedures.

Justification for LORT questions with KIA values< 3.0 J N/A Time to Complete: 1-2 minutes Point Value: 1 System ID No.: N/A PRA: No Safety Function(s): N/A IZ! ILT Category(s) (LORT Only): N/A 0 LORT OCSOPS ILT 13-1 EXAM Page: 198 of 201 13 May 2014

EXAMINATION ANSWER KEY ILT 13 ~ 1 NRC Written Exam (RO) 75 10: 13-1 NRO 75 Points: 1.00 Given the following:

  • Reactor power is at rated power
  • "C" Reactor Recirc Loop is ISOLATED
  • Reactor Core State Parameters are shown below:

OCS OPS ILT 13-1 EXAM Page: 199 of 201 13 May 2014

EXAMINATION ANSWER KEY ILT 13~1 NRC Written Exam (RO)

Based on current plant parameters, which ONE of the following parameters requires immediate action to restore it back to within allowable limits?

A. FLLLP B. MFLPD C. MAPRAT D. MFLCPR Answer: D Answer Explanation Knowledge and Ability Reference Information Importance Rating K&A RO SRO Conduct of Operations 2.1.19

  • Ability to use plant computers to evaluate system 3.9 3.8 or component status.

Level RO I Tier I 3 I Group I N/A General Att 202.1*5 References Dis Correct. With current plant condition, rated pwr, 4 loops operating, a MFLCPR value of >0.99 requires the crew to take immediate action to restore MFLCPR to within limits.

A & Bare Incorrect but plausible if the applicant does not interpret the indications from the Reactor Core State Parameters Explanation correctly.

Cis Incorrect but plausible if the applicant does not interpret the indications from the Reactor Core State Parameters correctly since one loop is isolated and one Core Spray L\P instrument is inoperable.

Lesson 2621.863.0.0007, Plant Computers Plan PCS-02239, Given examples of various displays from the 30 Learning Monicore evaluate them in terms of the data available and the Objective/ importance of this data to monitoring safe plant operation.

OCS OPS ILT 13*1 EXAM Page: 200 of 201 13 May 2014

EXAMINATION ANSWER KEY ILT 13~1 NRC Written Exam (RO)

References ILT: Att 202.1-5 LORT: Open Provided Question Source (New, Modified, Bank) New Previous 2 NRC Exams (ILT Only) No Memory or Comprehension Fundamental 3:SPR Cognitive or Analysis Knowledge Level NUREG 1021 Appendix 8: .Solve a Problem using References 55.41b 10 55.43b 10CFR55 Content Administrative, normal, abnormal, and emergency operating procedures for the facility.

Justification for LORT questions with KIA values< 3.0 I NIA Time to Complete: 1-2 minutes Point Value: 1 System I D No.: N/A PRA: No Safety Function(s): N/A ~ ILT Category(s) (LORT Only): NIA 0 LORT OCS OPS ILT 13-1 EXAM Page: 201 of 201 13 May 2014

EXAMINATION ANSWER KEY ILT 13-1 NRC Written Exam (SAO) 1 ID: 13-1 NSRO 01 Points: 1.00 The plant is shutting down and cooling down for a refueling outage with the Shutdown Cooling System. Current plant conditions include the following:

  • A and B SOC loops are in service
  • SO CLG CCW OUTLET V-5-106 indicates 80% open
  • RECIRC PUMP SUCTION TEMPS all indicate 205°F
  • SHUTDOWN COOLING CCW OUTLET NU01 B indicates 121 oF An event then occurs 1 minute later. With NO operator action, the plant conditions 10 minutes after the event include the following:
  • A and B SOC loops are in service
  • SO CLG CCW OUTLET V-5-1 06 indicates 0% open
  • RECIRC PUMP SUCTION TEMPS all indicate 20]CF and rising
  • SHUTDOWN COOLING CCW OUTLET NU01 B indicates 162°F and rising Which of the following actions is required FIRST for this event?

A. As RWCU temperatures rise, reduce RWCU flow lAW ABN-19, RBCCW Failure Response.

B. Initiate Isolation Condensers NOW as an alternate cooling method lAW ABN-3, Loss of Shutdown Cooling.

C. Verify EMRV operability PRIOR to coming out of the COLD SHUTDOWN CONDITION as required by TS 3.4, Emergency Cooling.

D. Verify Primary and Secondary Containment integrity PRIOR to coming out of the COLD SHUTDOWN CONDITION as required by TS 3.5, Containment.

Answer: D Answer Explanation Knowledge and Ability Reference Information Importance Rating K&A RO I SRO OCS OPS ILT 13-1 EXAM Page: 1 of 75 13 May 2014

EXAMINATION ANSWER KEY ILT 13~1 NRC Written Exam (SRO) 295018 Partial or Total Loss of CCW I 8 AA2.02- Ability to determine and/or interpret the following as they apply to PARTIAL OR COMPLETE LOSS OF 3.1 3.2 COMPONENT COOLING WATER: Cooling water temperature Level SRO I Tier I 1 I Group I 1 General TS 3.5 ABN-19 References TS 3.4 ABN-3 D is Correct. The question describes a plant shutdown and cooldown in progress using A & B SOC loops for a refueling outage (implying a significant amount of decay heat). An event occurs resulting in V-5-106, Combined RBCCW outlet from the SOC heat exchangers, going closed. With a loss of RBCCW through the SOC heat exchangers, RBCCW temperatures at the outlet of the heat exchangers rise. Based on the indications provided, reactor coolant temperature (given via the recirc pump suction temperatures), is rising toward the Hot Shutdown temperature of 212F. TS 3.5 requires that Primary and Secondary Containment is verified prior to reaching 212F. This would be the next required action.

A is Incorrect. Due to RBCCW flow through the SOC heat exchangers essentially being secured, RBCCW heat load on the RBCCW system has been significantly reduced. This will result Explanation in RBCCW temperatures lowering, and subsequently RWCU temperatures will also lower. Distractor A is a correct statement in ABN-19, however it is incorrect for this event since RWCU temperatures will lower, not rise, and therefore plausible if the applicant confuses system interellations or response.

B is Incorrect. Initiation of Isolation Condensers (ICs) is an alternate cooling method in Hot Shutdown lAW ABN-3, however the plant is not currently in Hot Shutdown. The distractor states to initiate ICs now, which is incorrect.

Cis Incorrect. EMRVs must be verified operable lAW TS 3.4 when > 212F and before reaching 11 0# RPV Pressure. This distractor is incorrect since it's not required to verify EMRV operability prior to reaching Hot Shutdown and also the question asks which action would be required first.

OCS OPS ILT 13-1 EXAM Page: 2 of 75 13 May 2014

EXAMINATION ANSWER KEY ILT 13-1 NRC Written Exam (SRO)

Lesson 2621.845.0.0035, Reactor Building Closed Cooling Water System Plan (RBCCW) 57, State how Service Water, Shutdown Cooling, Reactor Learning Cleanup, Primary Containment, AC Electrical Distribution and Objective/ chemical treatment systems interrelate with the RBCCW System.

References ILT: None LORT: Open Provided Question Source (New, Modified, Bank) Bank Previous 2 NRC Exams (ILT Only) No Memory or Comprehension Fundamental 3:SPK Cognitive or Analysis Knowledge Level NUREG 1021 Appendix B: Solve a Problem using Knowledge and its meaning 55.41b 55.43b 2 10CFR55 Content Facility operating limitations in the technical specifications and their bases.

Justification for LORT questions with KIA values< 3.0 I N/A Time to Complete: 1*2 minutes Point Value: 1 System ID No.: 295018 PRA: No Safety Function(s): 8 iZ! ILT Category(s) (LORT Only): N/A iZ! LORT OCS OPS ILT 13-1 EXAM Page: 3 of 75 13 May 2014

EXAMINATION ANSWER KEY ILT 13~1 NRC Written Exam (SRO) 2 10: 13*1 NSRO 02 Points: 1.00 The plant was at rated power when a transient resulted in an automatic SCRAM on low RPV water level. The following conditions exist:

  • RPV water level lowered to 115" and is rising slowly
  • RPV pressure is 920 psig and rising slowly
  • ALL APRM/LPRM DNSCL lights are LIT
  • ALL SCRAM SOLENOID GROUP lights are OFF
  • ARI has been manually initiated
  • ROPS has been placed in BYPASS
  • ADS Timers have been bypassed
  • MSIV and RBCCW Lo-Lo interlocks have been bypassed Which ONE of the following is required NEXT?

A. Trip all Reactor Recirculation Pumps, lAW RPV Control- With ATWS.

B. Control RPV water level 138"- 175", lAW RPV Control- With ATWS.

C. Vent the scram air header, lAW Support Procedure 21, Alternate Insertion of Control Rods.

D. Terminate and prevent injection into the RPV lAW Support Procedure 17, Termination and Prevention of Injection.

Answer: B Answer Explanation Knowledge and Ability Reference Information Importance Rating K&A RO SRO 295031 Reactor low water level EA2.02 Ability to determine and/or interpret the following 4.0 4.2 as they apply to REACTOR LOW WATER LEVEL: Reactor power Level SRO I Tier I 1 I Group I 1 General RPV Control - With References ATWS OCSOPSILT13-1 EXAM Page: 4 of 75 13 May2014

EXAMINATION ANSWER KEY ILT 13w1 NRC Written Exam (SAO)

B is Correct. The question stem states that the reactor has scrammed due to low RPV water level. Conditions in the stem state that numerous control rods indicate position 06 or greater, APRM/LPRM downscale lights are lit, and all scram solenoid group lights are off. The applicant must diagnose this as a hydraulic ATWS with reactor power below 2%. The applicant must also have knowledge of procedure steps within the RPV Control- with ATWS EOP. Based on ADS timers being bypassed, MSIV and RBCCW Lo-Lo interlocks being bypassed, RPV water level >30", and reactor power below 2%, the only action listed that is correct is to maintain RPV water level between 138-175" using Support Procedure 191AW per the RPV Control- with ATWS EOP.

Explanation A is Incorrect but plausible if the applicant does not recognize reactor power is <2% from the conditions in the stem and therefore does not recognize procedural requirements. This would also be a correct choice if reactor power was ~ 2%.

C is Incorrect buy plausible. Venting the Scram Air Header is a correct choice per Support Procedure 21, but for an Electric ATWS, not a Hydraulic ATWS.

D is Incorrect by plausible. This choice would be correct if reactor power was ~2%. If the applicant may not recognize reactor power is <2% from the conditions in the stem.

Lesson 2621.845.0.0053 RPV Control- With ATWS Plan Learning Objective/ 3055A Given a copy of RPV Control, describe in detail each step or conditional statement, including technical basis, and how to perform each step as required.

References ILT: None LORT: Open Provided Question Source (New, Modified, Bank) New Previous 2 NRC Exams (ILT Only) No Memory or Comprehension Fundamental 3:SPK Cognitive Knowledge or Analysis Level NUREG 1021 Appendix B: Solve a Problem using Knowledge and its meaning OCS OPS ILT 13-1 EXAM Page: 5 of 75 13 May 2014

EXAMINATION ANSWER KEY ILT 13~1 NRC Written Exam (SRO) 10CFR55 55.41b I I 55.43b I 5 Content Assessment of facility conditions and selection of appropriate procedures during normal, abnormal, and emergency situations.

Justification for LORT questions with KIA values< 3.0 l NIA Time to Complete: 1-2 minutes Point Value: 1 System I D No.: 295031 PRA: No Safety Function(s): 2 ~ ILT Category(s) (LORT Only): NIA ~ LORT OCS OPS ILT 13-1 EXAM Page: 6 of 75 13 May 2014

EXAMINATION ANSWER KEY ILT 13-1 NRC Written Exam (SRO) 3 10: 13-1 NSRO 03 Points: 1.00 The plant was at rated power when a seismic event lasting several minutes occurred.

The following conditions exist:

  • ALL LPRM amber lights on Panel 4F are lit
  • RPV Pressure is 700 psig and rising
  • RPV water level is 50" and lowering
  • Drywell Pressure is 26 psig and rising
  • Drywell Temperature is 240°F and rising
  • Torus water temperature is 148°F and steady
  • Torus water level is 141" and rising
  • Annunciator TORUS/DW2 VAC BRKR OPN is in alarm Which of the following actions is required NEXT?

A Emergency Depressurize the RPV due to PSP.

B. Emergency Depressurize the RPV due to HCTL.

C. Initiate the Liquid Poison System lAW Support Procedure 22.

D. Emergency Depressurize the RPV when Drywell Temp reaches the Drywell design temperature.

Answer: A Answer Explanation Knowledge and Ability Reference Information Importance Rating K&A RO SRO 295030 Low Suppression Pool Water Level/ 5 EA2.04 -Ability to determine and/or interpret the following as they apply to LOW SUPPRESSION POOL WATER 3.5 3.7 LEVEL : Drywell/ suppression chamber differential pressure: Mark-1&11 Level SRO I Tier I 1 I Group I 1 General RPV Control-with PCC EOP EOP User's Guide References ATWS EOP OCS OPS ILT 13-1 EXAM Page: 7 of 75 13 May 2014

EXAMINATION ANSWER KEY ILT 13-1 NRC Written Exam (SRO)

A is Correct. The question stem provides information that a seismic event resulted in a LOCA combined with a Torus leak.

There is also a Hydraulic ATWS with 4 control rods stuck at position 48, however reactor power can be determined to be less than 2% due to all LPRM amber lights lit on Panel 4F. The question states that there are several Drywell to Torus vacuum breakers open and it can be assumed that Drywell to Torus differential pressure will be zero (ie. Drywell and Torus Pressure will now be equal). Now that Torus Pressure and Torus Level are known, the student will be able to determine that ED is required due to the PSP being violated. This meets the knowledge and interpret portion requirements of the KIA.

8 is Incorrect. With RPV pressure at 700#, Torus level at 141",

and Torus temperature at 148F, the HCTL curve has not been exceeded. This distractor is plausible if the student does not Explanation interpret the HCTL curve correctly or believes based on provided indications this would be the NEXT action that would be directed.

C is Incorrect. The question stem states that Torus Temperature is 148F. With reactor power< 2% (LPRM amber lights lit) the BIIT temperature is 150F. SLC injection is only required when Torus Temperature is expected to exceed this value. This distractor is plausible if the student does not recognize they are able to determine reactor power by all LPRM amber lights lit and believes that reactor power is higher due to the ATWS.

D is Incorrect. The temperature leg in PCC says to wait until it is determined that bulk Drywell Temperature cannot be restored and maintained below 281 F (Drywell design temperature) before performing an ED. Drywell Temperature is currently only 240F and rising. The PSP is already exceeded therefore the NEXT action would be to ED due to exceeding the PSP.

Lesson 2621.845.0.0056, Primary Containment Control Plan 200-10445, Given a set of system indications or data, evaluate and interpret them to determine limits, trends and system status.

Learning Objective/

References ILT: BIIT & PSP Graphs LORT: Open Provided Question Source (New, Modified, Bank) Modified Previous 2 NRC Exams (ILT Only) No OCS OPS ILT 13-1 EXAM Page: 8 of 75 13 May 2014

EXAMINATION ANSWER KEY ILT 13-1 NRC Written Exam (SAO)

Memory or Comprehension Fundamental 3:SPK Cognitive or Analysis Knowledge Level NUREG 1021 Appendix B: Solve a Problem using Knowledge and its meaning 55.41b 55.43b 5 10CFR55 Content Assessment of facility conditions and selection of appropriate procedures during normal, abnormal, and emergency situations.

Justification for LORT questions with KIA values< 3.0 I NIA Time to Complete: 1*2 minutes Point Value: 1 System ID No.: 295030 PRA: No Safety Function(s): 5 ~ ILT Category(s) (LORT Only): NIA ~ LORT OCS OPS ILT 13-1 EXAM Page: 9 of 75 13 May 2014

EXAMINATION ANSWER KEY ILT 13~1 NRC Written Exam (SRO) 4 10: 13~1 NSRO 04 Points: 1.00 The plant was at rated power. An event then occurred resulting in an Electric ATWS.

At Time = 0, plant conditions include the following:

  • Reactor power is 80% and lowering slowly
  • Reactor pressure is 1000 psig and steady
  • RPV water level is being maintained lAW EOPs
  • Drywell pressure is 2.2 psig and steady
  • Torus level is 164" and steady
  • Torus temperature is 146°F and rising 2°F/min With NO further operator action, a General Emergency EAL threshold will FIRST be met at Time= min.

A. 7 B. 5

c. 4 D. 0 Answer: A Answer Explanation Knowledge and Ability Reference Information Importance Rating K&A RO SRO 295026 Suppression Pool High Water Temp. I 5 2.4.41- Emergency Procedures I Plan: Knowledge of the 2.9 4.6 emergency action level thresholds and classifications.

Level SRO I Tier I 1 I Group I 1 General EP-AA-1010 RPVC- with ATWS References OCS OPS ILT 13-1 EXAM Page: 10 of 75 13 May 2014

EXAMINATION ANSWER KEY ILT 13~1 NRC Written Exam (SRO)

A is Correct. Plant conditions state there is an Electrical ATWS.

Containment parameters indicate Torus water temperature rising at 2F/min. lAW EP-AA-1010, OC Emergency Plan Annex, a threshold for a General Emergency (MG2) has been exceeded with a failure of an automatic scram, manual scram, combined with the Heat Capacity Temperature Limit (HCTL) being exceeded. With Torus level at 164", RPV pressure at 1000 psig, Explanation and Torus temp at 146F rising 2F/min, the FIRST time listed where the HCTL has been exceeded and therefore a GE threshold exceeded is 7 min, when Torus temp will be 160F.

All distractors are Incorrect but plausible if the applicant either does not correctly interpret the HCTL graph and/or correctly interpret the EAL threshold guidance within EP-AA-1 010.

Lesson 2621.845.0.0056, Primary Containment Control Plan 200-10445, Given a set of system indications or data, evaluate Learning and interpret them to determine limits, trends and system status.

Objective/

References ILT: HCTL graph and EAL LORT: Open Provided Hot Matrix Question Source (New, Modified, Bank) New Previous 2 NRC Exams (ILT Only) No Memory or Comprehension Fundamental 3:SPR Cognitive or Analysis Knowledge Level NUREG 1021 Appendix 8: .Solve a Problem using References 55.41b 55.43b 5 10CFR55 Content Assessment of facility conditions and selection of appropriate procedures during normal, abnormal, and emergency situations.

Justification for LORT questions with KIA values< 3.0 I NIA Time to Complete: 1-2 minutes Point Value: 1 System 10 No.: 295026 PRA: No Safety Function(s): 5 ~ ILT Category(s) (LORT Only): NIA ~ LORT OCS OPS ILT 13-1 EXAM Page: 11 of 75 13 May 2014

EXAMINATION ANSWER KEY ILT 13*1 NRC Written Exam (SRO) 5 10: 13-1 NSRO 05 Points: 1.00 The plant was at rated power when a small LOCA occurred. The following plant conditions currently exist:

  • RPV pressure is 780 psig and lowering slowly.
  • RPV water level is 178" and lowering slowly.
  • Torus temperature is 112° F and rising.
  • OW pressure is 13 psig and rising.
  • Drywell temperature is 285° F and rising.
  • A loss of offsite power has occurred.
  • EDG 2 BREAKER indicates green light ON.
  • The SYSTEM I MODE SELECT switch is in the OW SPRAY position with the TORUS CLG light ON and the OW SPRAY light OFF.

Which of the following is the NEXT required action? (Assume the Reactor Building has been evacuated)

A. Emergency depressurize the RPV, lAW Primary Containment Control.

B. Initiate one loop of Isolation Condensers to augment the cooldown, lAW RPV Control - No ATWS.

C. Initiate one loop of Containment Spray in the OW SPRAY mode, lAW Primary Containment Control.

D. Lower reactor pressure to allow low pressure systems to inject into the RPV, lAW RPV Control- No ATWS.

Answer: A Answer Explanation Knowledge and Ability Reference Information Importance Rating K&A RO SRO 295028 High Drywell Temperature I 5 2.4.9

  • Emergency Procedures I Plan: Knowledge of low power I shutdown implications in accident (e.g., loss of 3.8 4.2 coolant accident or loss of residual heat removal) mitigation strate_g_ies.

Level SRO I Tier I 1 I Group I 1 General PCC EOP References OCS OPS ILT 13*1 EXAM Page: 12 of 75 13 May 2014

EXAMINATION ANSWER KEY ILT 13-1 NRC Written Exam (SRO)

A is Correct. The stem provides that OW temperature is 285° and rising, and no OW sprays are available. Also, since RPV pressure is lowering and OW pressure is rising, there are no indications that the LOCA has been terminated. Therefore, OW temperature will continue to rise. lAW Primary Containment Control EOP, ED should be directed when it has been determined that OW temperature cannot be maintained/restored below 281 oF.

B is Incorrect but plausible. Because RPV water level is above 160" currently, placing ICs into service is not allowed lAW RPV Control- No ATWS (the limit is 160" for initiating).

Explanation C is Incorrect but plausible. The question stem says that a loss of offsite power has occurred, and that EDG2 is NOT powering Bus 1D. Therefore, containment spray pumps 51 C and 51 D are not available (System 2). Also, the stem shows that the System 1 mode select switch will not open the System 1 OW spray valve.

Thus, there are no containment sprays available on the operable emergency bus (1 C), and therefore, there are no containment sprays available to spray the OW at all.

D is Incorrect but plausible. Lowering RPV pressure is in the RPV water level leg, when RPV water cannot be maintained/restored above 61 ". The stem says that water level is 178" and lowering slowly.

Lesson 2621.845.0.0056, Primary Containment Control Plan 200-10445, Given a set of system indications or data, evaluate Learning and interpret them to determine limits, trends and system status.

Objective/

References ILT: None LORT: Open Provided Question Source (New, Modified, Bank) Bank Previous 2 NRC Exams (ILT Only) No Memory or Comprehension Fundamental 3:SPK Cognitive or Analysis Knowledge Level NUREG 1021 Appendix B: Solve a Problem using Knowledge and its meaning L-------~--------¥- _____________________,______________________

OCSOPS ILT 13-1 EXAM Page: 13 of 75 13 May 2014

EXAMINATION ANSWER KEY ILT 13w1 NRC Written Exam (SAO) 10CFR55 55.41b J I 55.43b I 5 Content Assessment of facility conditions and selection of appropriate procedures during normal, abnormal, and emergency situations.

Justification for LORT questions with KIA values< 3.0 I N/A Time to Complete: 1*2 minutes Point Value: 1 System ID No.: 295028 PRA: No Safety Function(s): 5 ~ ILT Category(s) (LORT Only): N/A ~ LORT OCS OPS ILT 13-1 EXAM Page: 14 of 75 13 May 2014

EXAMINATION ANSWER KEY ILT 13~1 NRC Written Exam (SAO) 6 10: 13*1 NSRO 06 Points: 1.00 The plant was at rated power when an event occurred. Present plant conditions are as follows:

  • RPV Pressure is 1000 psig
  • COND B FLOW HI POSSIBLE RUPTURE annunciator is in alarm
  • ISOL CONDENSER B LEVEL indicates 9.5'
  • Isolation Condenser B Shell Temperature is 350° F and rising
  • Isolation Condenser B CANNOT be isolated
  • RADIATION MONITORS- OFFGAS HI-HI annunciator is in alarm
  • RPS 1 and RPS 2 SCRAM SOLENOIDS lights are de-energized
  • Several Area Radiation Monitors in the Reactor Building are reading slightly above their high setpoint
  • The Site Emergency Director has declared aGE EAL (RG1, Radiological Effluent)

Which of the following actions shall the SAO direct?

A. Emergency Depressurize the RPV lAW the Radioactivity Release Control EOP B. Emergency Depressurize the RPV lAW the Secondary Containment Control EOP C. Depressurize the RPV to maintain the cooldown rate below 100 °F/hr lAW the RPV Control- No ATWS EOP D. Rapidly reduce RPV pressure with the Main Turbine Bypass Valves lAW ABN-26, High Main Steam/Offgas/Stack Effluent Activity Answer: A Answer Explanation Knowledge and Ability Reference Information Importance Rating K&A RO I SRO OCSOPS ILT 13*1 EXAM Page: 15 of 75 13 May 2014

EXAMINATION ANSWER KEY ILT 13*1 NRC Written Exam (SAO) 295038 High Off-site Release Rate I 9 2.4.45

  • Emergency Procedures I Plan: Ability to prioritize 4.1 4.3 and interpret the significance of each annunciator or alarm.

Level SRO I Tier I 1 I Group I 1 General RREOP References A is Correct. The plant was at rated power when an event occurred. The given conditions show the following: fuel failures as evidenced by offgas and main steam line radiation alarms; a tube break in Isolation Condenser B as evidenced by the high flow annunciator and high water level in the shell. The plant has scrammed as evidenced by the RPS solenoid lights de-energized.

The following procedures will be entered: RPV Control - No ATWS EOP (on RPV water level following the scram), Secondary Containment Control EOP (several ARMs in the RB are in alarm),

Radioactivity Release Control EOP (indications of an Isolation Condenser tube leak) and ABN-26 (high offgas and main steam radiation monitors).

The requirements to ED in the Rad Release EOP are: indications of fuel damage, and a General Emergency declared due to offsite dose (which has been declared and provided). Therefore, ED is Explanation required lAW the Rad Release EOP.

B is Incorrect but plausible. ED can be directed from the Secondary Containment Control EOP but there needs to be a discharge of a primary system into secondary containment, and a max safe value exceeded in ~ 2 areas. The indications provided show a general rise in radiation levels in the reactor Building but not to the extent of max safe.

Cis Incorrect but plausible. The RPV Control- No ATWS EOP does direct establishing a normal cooldown, but this is overridden by other EOPs.

Dis Incorrect but plausible. Reducing RPV pressure will reduce the driving head on the leak, but the Main Steam Lime Radiation Monitors shall be isolated lAW ABN-26, when greater than 800 mrlhr, ans thus the turbine bypass valves are not available.

OCS OPS ILT 13-1 EXAM Page: 16 of 75 13 May 2014

EXAMINATION ANSWER KEY ILT 13~1 NRC Written Exam (SRO)

Lesson 2624.845.0.0012, Radioactivity Release Control Plan Learning RRC-01667, Based upon specific plant parameters and Objective/ conditions, determine if entry conditions for Emergency Operating Procedures (EOPs) have been met and which EOP(s) are applicable to the conditions provided.

References ILT: None LORT: Open Provided Question Source (New, Modified, Bank) Bank Previous 2 NRC Exams (ILT Only) No Memory or Comprehension Fundamental 3:SPK Cognitive or Analysis Knowledge Level NUREG 1021 Appendix B: .Solve a Problem using Knowledge and its meaning 55.41b 55.43b 5 10CFR55 Content Assessment of facility conditions and selection of appropriate procedures during normal, abnormal, and emergency situations.

Justification for LORT questions with KIA values< 3.0 I N/A Time to Complete: 1-2 minutes Point Value: 1 System I D No.: 295038 PRA: No Safety Function(s): 9 ~ ILT Category(s) (LORT Only): N/A ~ LORT OCS OPS ILT 13-1 EXAM Page: 17 of 75 13 May 2014

EXAMINATION ANSWER KEY ILT 13-1 NRC Written Exam (SAO) 7 10: 13*1 NSRO 07 Points: 1.00 The plant is in SHUTDOWN due to a loss of condenser vacuum (which has NOT been restored).

The following conditions currently exist:

  • RPV water level is in the normal band
  • Unit Substation 1B2 is de-energized for emergent maintenance (it is expected the bus will be returned to service in 30 minutes)
  • RPV coolant temperature is 220°F and is trending down slowly The following component failure has just occurred:
  • TE-31 J (Reactor Recirculation Pump E suction temperature element) has failed upscale Which of the following states (1) the effect on the plant and (2) the required actions the SAO shall direct FIRST?

A. (1) Shutdown Cooling Pump A has tripped due to high SOC Pump A suction temperature (2) Start SOC Pump B or C to restore SOC System flow lAW procedure 305, Shutdown Cooling System Operation B. (1) Shutdown Cooling Pump A has tripped due to isolation valve closure (2) Bypass the failed temperature element and restore SOC Pump A lAW ABN-3, Loss of Shutdown Cooling C. (1) Shutdown Cooling Pump A has tripped due to isolation valve closure (2) Initiate Alternate Shutdown Cooling Using EMRVs and Core Spray lAW ABN-3, Loss of Shutdown Cooling D. (1) Shutdown Cooling Pump A has tripped due to high SOC Pump A suction temperature (2) Initiate Alternate RPV cooldown (cleanup system letdown) lAW procedure 303, Reactor Cleanup Demineralizer System Answer: B IAnswer Explanation OCS OPS ILT 13-1 EXAM Page: 18 of 75 13 May 2014

EXAMINATION ANSWER KEY ILT 13M1 NRC Written Exam (SRO)

Knowledge and Ability Reference Information Importance Rating K&A RO SRO 295021 Loss of Shutdown Cooling/ 4 2.2.44- Ability to interpret control room indications to verify the status and operation of a system, and 4.2 4.4 understand how operator actions and directives affect plant and system conditions.

Level SRO I Tier I 1 I Group I 1 General References B is Correct. A high temperature sensed on any reactor recirculation loop (350F) will isolate SOC. When SOC IV V-17-19 closes, all SOC pumps trip. Because it has been determined that the recirculation temperature sensor has failed, ABN-3 allows bypassing the sensor and restoring SOC flow.

A is Incorrect but plausible. It is true that 350F SOC suction temperature will isolate SOC and trip the SOC pump, this is not what was given in the question. Also, SOC Pumps Band Care powered by USS 1B2, which is de-energized. Answer a is incorrect.

Explanation C is Incorrect but plausible. As stated, the SOC IVs close on high Recirc. temperature. ABN-3 directs that the failed temperature thermocouple be bypassed and SOC restored in step 4.4.1. Later, in step 4.4.4, it directs alternate cooling with core spray and EMRVs. Step 4.4.1 would be the first action performed lAW the ABN.

0 is Incorrect but plausible. As stated, SOC pump trips from IV position, not SOC loop temperature. Also, cleanup letdown as an alternate path is not available since the condenser is not available (given in the question stem).

Lesson 2621.828.0.0045, Shutdown Cooling System Plan Learning SOC-10441, Given the system logic/electrical drawings, describe Objective/ the system trip signals, setpoints and expected system response including power loss or failed components.

References ILT: None LORT: Open Provided Question Source (New, Modified, Bank) Bank OCS OPS ILT 13-1 EXAM Page: 19 of 75 13 May 2014

EXAMINATION ANSWER KEY ILT 13~1 NRC Written Exam (SAO)

Previous 2 NRC Exams (ILT Only) I No Memory or Comprehension Fundamental 3:SPK Cognitive or Analysis Knowledge Level NUREG 1021 Appendix 8: Solve a Problem using Knowledge and its meaning 55.41b 55.43b 5 10CFR55 Content Assessment of facility conditions and selection of appropriate procedures during normal, abnormal, and emergency situations.

Justification for LORT questions with KIA values< 3.0 I N/A Time to Complete: 1*2 minutes Point Value: 1 System ID No.: 295021 PRA: No Safety Function(s): 4 ~ ILT Category(s) (LORT Only): N/A ~ LORT OCS OPS ILT 13-1 EXAM Page: 20 of 75 13 May 2014

EXAMINATION ANSWER KEY ILT 13~1 NRC Written Exam (SRO) 8 10: 13*1 NSRO 08 Points: 1.00 The plant was at rated power when the following annunciators alarmed:

  • ROPS ACTUATE A
  • RXLVLHIII The Operator reports that indication for RE05A in Panel 19R indicates upscale at 185".

lAW ABN-59, RPV Level Instrument Failures, the US shall confirm all automatic actions have occurred from the failed instrument and ...

A. insert a manual1/2 scram on RPS 1.

B. place RPS 1 Subchannel test Switches to TEST.

C. place the redundant GEMAC level instrument in control.

D. restore the failed instrument within 12 hrs or place it in the tripped condition.

Answer: D Answer Explanation Knowledge and Ability Reference Information Importance Rating K&A RO SRO 295008 High Reactor Water Level/2 2.1.20 - Conduct of Operations: Ability to interpret and 4.6 4.6 execute procedure steps.

Level SRO I Tier I 3 I Group I N/A General ABN-59 References OCS OPS ILT 13-1 EXAM Page: 21 of 75 13 May 2014

EXAMINATION ANSWER KEY ILT 13-1 NRC Written Exam (SRO)

The plant was at rated power when RPS RPV water level instrument REOSA failed upscale. This instrument inputs into the turbine trip circuit and Feedwater Pump trip on RPV high water level, and inputs into the reactor scram on RPV low water level.

When just one level instrument fails upscale, there are no automatic actions which occur.

D is Correct. ABN-59 says that if an instrument malfunction has occurred or is suspected (as compared to a failed indicator), then confirm all automatic actions have occurred due to the failed instrument and confirm compliance with TS 3.1. TS 3.1 states

.. With one required channel inoperable in one Trip System, within Explanation 12 hours1.388889e-4 days <br />0.00333 hours <br />1.984127e-5 weeks <br />4.566e-6 months <br />, restore the inoperable channel or place the inoperable channel and/or that Trip System in the tripped condition ...

The ABN action in answer C is Incorrect but plausible since if the failed instrument input into Feedwater Level Control system it would be correct, but the REOSA does not.

A & B are Incorrect but plausible if the applicant confuses a scram setpoint with RPV high water level, then the ABN actions in answers A or B might be correct. But even though it is a scram water level instrument, there is no scram setpoint on RPV water level high.

Lesson 2621.828.0.0030 Nuclear Steam Supply System Plan Learning 1032 Given tech specs, analyze each LCO for the NSSS system Objective/ under described conditions.

References ILT: None LORT: Open Provided Question Source (New, Modified, Bank) Modified Previous 2 NRC Exams (ILT Only) No Memory or Comprehension Fundamental 3:SPK Cognitive or Analysis Knowledge Level NUREG 1021 Appendix B: Solve a Problem using Knowledge and its meaning L-------~-----------------~----------------------------------------------------------------------------------------------------

OCS OPS ILT 13-1 EXAM Page: 22 of 75 13 May 2014

EXAMINATION ANSWER KEY ILT 13w1 NRC Written Exam (SRO) 10CFR55 55.41b I I 55.43b I 2 Content Facility operating limitations in the technical specifications and their bases.

Justification for LORT questions with KIA values< 3.0 I N/A Time to Complete: 1-2 minutes Point Value: 1 System I D No.: N/A PRA: No Safety Function(s): NIA ~ ILT Category(s) (LORT Only): N/A ~ LORT OCS OPS ILT 13-1 EXAM Page: 23 of 75 13 May 2014

EXAMINATION ANSWER KEY ILT 13-1 NRC Written Exam (SAO) 9 ID: 13-1 NSRO 09 Points: 1.00 The plant was at rated power when an automatic scram setpoint was exceeded.

Current plant conditions include the following:

  • All individual scram lights (Panel 4F) are energized
  • RPV water level is 141" and lowering
  • RPV pressure is 990 psig and steady
  • Torus water temperature is 94 °F and steady
  • Drywell pressure is 9 psig and rising slowly
  • Drywell temperature is 201 °F and rising slowly
  • TOTAL STEAM FLOW is 2.01 MLB/HR
  • All MFRV FLOW CONTROLLERs are in MAN and TOTAL FEEDWATER FLOW indicates 0 MLB/HR
  • ISOL SIGNAL BYPASS V-6-395 is in BYPASS lAW the EOP Users Guide, which of the following states the strategy of HIGHEST priority?

A. In the Pressure leg of the Primary Containment Control EOP, direct initiating Drywell Sprays.

B. In the Level/Power leg of the RPV Control- With ATWS EOP, direct an RPV water level band to below 30".

C. In the Power leg of the RPV Control- With ATWS EOP, direct inserting control rods by venting the scram air header.

D. In the Torus Water Temperature leg of the Primary Containment Control EOP, initiate Containment Spray in the Torus Cooling Mode.

Answer: B Answer Explanation Knowledge and Ability Reference Information Importance Rating K&A RO SAO 295009 Low Reactor Water Level/ 2 AA2.02 - Ability to determine and/or interpret the following 3.6 3.7 as they apply to LOW REACTOR WATER LEVEL : Steam flow/feed flow mismatch Level I SAO I Tier I 1 I Group I 2 OCS OPS ILT 13-1 EXAM Page: 24 of 75 13 May 2014

EXAMINATION ANSWER KEY ILT 13*1 NRC Written Exam (SRO)

General EOP Users Guide References B is Correct. The plant was at power when an ATWS occurred.

Control rod position has been lost and no direct power indication is provided. But with steam flow at 2 Mlb/hr, and steady RPV pressure, power is about 28% (rated steam flow is about 7.1 Mlb/hr).

The question stem also shows that the MSIV lo-lo isolation bypass (SP-16) has been performed (V-6-395 in bypass). Because reactor power is so high, the next task is to terminate/prevent injection lAW SP-17 by stopping all Feedwater Pumps. This also has been performed and accounts for the large steam/feedwater flow mismatch. The next direction is to direct an RPV water level to below 30" lAW the ATWS EOP.

A is Incorrect but plausible. Maintaining Primary Containment Explanation intact is certainly important, but Drywell Sprays cannot be initiated until Dryweii/Torus pressure exceeds 12 psig (and its only 9 psig currently). Therefore, Drywell Sprays cannot be initiated from the pressure leg of the Primary Containment Control EOP.

C is Incorrect but plausible. Inserting control rods is also of high priority, but the method to insert control rods is for an electric ATWS and indications given suggest a hydraulic ATWS.

D is Incorrect but plausible. With Torus water temperature at 93F and steady, the entry condition from this parameter into Primary Containment Control EOP has not been reached. Since temperature is < 95F and is steady, there is no hurry in initiating Torus Cooling and is of lower priority than lowering RPV water level through termination/prevention of RPV injection.

Lesson 2621.845.0.001 B, RPV Control with ATWS Plan Learning EWA-10445, Given a set of system indications or data, evaluate Objective/ and interpret them to determine limits, trends and system status.

References ILT: None LORT: Open Provided Question Source (New, Modified, Bank) Bank Previous 2 NRC Exams (ILT Only) No OCS OPS ILT 13-1 EXAM Page: 25 of 75 13 May 2014

EXAMINATION ANSWER KEY ILT 13~1 NRC Written Exam (SRO)

Memory or Comprehension Fundamental 3:SPK Cognitive or Analysis Knowledge Level NUREG 1021 Appendix B: Solve a Problem using Knowledge and its meaning 55.41b 55.43b 5 10CFR55 Content Assessment of facility conditions and selection of appropriate procedures during normal, abnormal, and emergency situations.

Justification for LORT questions with KIA values< 3.0 I N/A Time to Complete: 1*2 minutes Point Value: 1 System ID No.: 295009 PRA: No Safety Function(s): 2 ~ ILT Category(s) (LORT Only): NIA ~ LORT OCS OPS ILT 13-1 EXAM Page: 26 of 75 13 May 2014

EXAMINATION ANSWER KEY ILT 13~1 NRC Written Exam (SRO) 10 ID: 13*1 NSRO 10 Points: 1.00 The plant was at rated power when the station experienced a seismic event. All EOs were sent to investigate the plant for damage.

The Reactor Building EO reports that there is a large fire main pipe break in the NE Corner Room and water is above the red line on the wall and rising.

Which of the following Containment Spray pumps are inoperable AND what action is required?

INOPERABLE CQntainment The reactQr may remain in Spray Pumps operation not to exceed ...

A. 51 A and 51 B 7 days B. 51 A and 51 B 15 days C. 51 C and 51 D 7 days D. 51C and 510 15 days Answer: A Answer Explanation Knowledge and Ability Reference Information Importance Rating K&A RO SRO 295036 Secondary Containment High Sump/Area Water Level/5 EA2.01 - Ability to determine and/or interpret the following 3.0 3.2 as they apply to SECONDARY CONTAINMENT HIGH SUMP/AREA WATER LEVEL : Operability of components within the affected area Level SRO I Tier I 1 I Group I 2 General 2000-BAS-3200.02 References sec EOP (EOP Users Guide)

TS 3.4.C OCS OPS ILT 13-1 EXAM Page: 27 of 75 13 May 2014

EXAMINATION ANSWER KEY ILT 13~1 NRC Written Exam (SRO)

A is Correct. The question states there is a large leak in the NE corner room and level is above Max Safe (above the Red line -

16 11 ). Since the leak is from the fire protection system and not a primary system, the reactor may remain in operation. The question does not state any other area is above Max Safe so it can be assumed that this is the only corner room affected.

Containment Spray pumps 51 A nd 51 B are located in the NE corner room. The EOP Users Guide states that corner room level at 16 11 (red line) is the level of the Containment Spray pump motor housing. At a corner room level above 16 11 the Containment Spray pumps in that room cannot be considered operable. TS 3.4.C.4 states that if two Containment Spray pumps in the same loop become inoperable, the action requirement of TS 3.4.C.3 apply. TS 3.4.C.3 states that the reactor may remain in operation for a period not to exceed 7 days.

B is Incorrect. Due to Containment Spray pumps 51 A and 51 B being in the same loop, the action requirement of TS 3.4.C.3 Explanation applies. TS 3.4.C.3 states that the reactor may remain in operation for a period not to exceed 7 days, not 15 days. This distractor is plausible if the student misinterprets the TS action requirement.

C is Incorrect. The NE corner room is the location of Containment Spray pumps 51 A and 518, not 51C and 51 D. This distractor is plausible if the student does not remember which Containment Spray pumps are in the NE corner room.

D is Incorrect. The NE corner room is the location of Containment Spray pumps 51 A and 51 B, not 51 C and 51 D. This distractor is plausible if the student does not remember which Containment Spray pumps are in the NE corner room.

Additionally, since two Containment Spray pumps in the same loop are inoperable, the action requirement of TS 3.4.C.3 apply.

TS 3.4.C.3 states that the reactor may remain in operation for a period not to exceed 7 days, not 15 days. This distractor is plausible if the student misinterprets the TS action requirement.

Lesson 2621.845.0.0057, Secondary Containment Control Plan SCC-1667, Based upon specific plant parameters and conditions, determine if entry conditions for Emergency Operating Learning Procedures (EOPs) have been met and which EOPs are Objective/ applicable to the conditions provided.

References ILT: None LORT: Open Provided OCS OPS ILT 13-1 EXAM Page: 28 of 75 13 May 2014

EXAMINATION ANSWER KEY ILT 13-1 NRC Written Exam (SAO)

Question Source (New, Modified, Bank) Bank Previous 2 NRC Exams (ILT Only) No Memory or Comprehension Fundamental 3:SPK Cognitive or Analysis Knowledge Level NUREG 1021 Appendix B: .Solve a Problem using Knowledge and its meaning 55.41b 55.43b 2 10CFR55 Content Facility operating limitations in the technical specifications and their bases Justification for LORT questions with KIA values< 3.0 I NIA Time to Complete: 1*2 minutes Point Value: 1 System 10 No.: 295036 PRA: No Safety Function(s): 5 ~ ILT Category(s) (LORT Only): N/A ~ LORT OCS OPS ILT 13-1 EXAM Page: 29 of 75 13 May 2014

EXAMINATION ANSWER KEY ILT 13~1 NRC Written Exam (SAO) 11 10: 13-1 NSRO 11 Points: 1.00 A plant startup is in progress with the following conditions:

  • RPV pressure is 700 psig and rising slowly
  • RPV water level is in the normal band
  • Feedwater Pump A is in service An event then occurred. Plant conditions now include the following:
  • RPV water level swelled to 181" and continues to slowly rise Based on the above conditions, which of the following RPV pressure control strategies shall the SAO direct?

A. Use EMRVs lAW RPV Control- no ATWS B. Adjust the MPR setpoint lAW 201, Plant Startup C. Use the Isolation Condensers lAW RPV Control- no ATWS D. Use the Bypass Valve Opening Jack lAW 201, Plant Startup Answer: A Answer Explanation Knowledge and Ability Reference Information Importance Rating K&A RO SRO 212000 RPS A2.01 - Ability to (a) predict the impacts of the following on the REACTOR PROTECTION SYSTEM; and (b) based on 3.7 3.9 those predictions, use procedures to correct, control, or mitigate the consequences of those abnormal conditions or operations: RPS motor-generator set failure Level SRO I Tier I 2 I Group I 1 General RPV Control - no 237E566 Sh. 3 EOP User*s Guide References AWTS OCS OPS ILT 13-1 EXAM Page: 30 of 75 13 May 2014

EXAMINATION ANSWER KEY ILT 13~1 NRC Written Exam (SAO)

A is Correct. Under the conditions in the stem, with RPV pressure less that 825 psig (TS value), a single RPS Bus loss will result in a full reactor scram and closure of the MSIVs. With the closure of the MSIVs, changing the MPR setpoint or the Bypass Valve Opening Jack will have no impact on RPV pressure. The EMRVs can be still used to control RPV pressure. Even though the use of EMRVs require Torus water level above 90", the event started at normal level of approximately 150" and can be assumed to be the same.

Explanation Band Dare Incorrect but plausible if the applicant does not recall the RPV Control- no ATWS Pressure leg actions for the conditions provided.

Cis Incorrect. This distractor is plausible if the applicant does not recognize that the Isolation Condensers are not available due to RPV level > 160 11 and believes this is a valid RPV pressure control strategy within the RPV Control- no ATWS EOP.

Lesson 2621.828.0.0037, Reactor Protection System Plan Learning RPS-1 0445, Given a set of system indications or date, evaluate Objective/ and interpret them to determine limits, trends, and system status.

References ILT: None LORT: Open Provided Question Source (New, Modified, Bank) Bank Previous 2 NRC Exams (ILT Only) Yes Memory or Comprehension Fundamental 3:SPK Cognitive or Analysis Knowledge Level NUREG 1021 Appendix B: Solve a Problem using Knowledge and its meaning 55.41b 55.43b 5 10CFR55 Content Assessment of facility conditions and selection of appropriate procedures during normal, abnormal, and emergency situations.

Justification for LORT questions with KIA values< 3.0 I NIA Time to Complete: 1-2 minutes Point Value: 1 System ID No.: 212000 PRA: No Safety Function(s): 7 ~ ILT OCS OPS ILT 13-1 EXAM Page: 31 of 75 13 May 2014

EXAMINATION ANSWER KEY ILT 13w1 NRC Written Exam (SAO)

I Category(s) (LORT Only): I NIA I~ LORT OCS OPS ILT 13-1 EXAM Page: 32 of 75 13 May 2014

EXAMINATION ANSWER KEY ILT 13~1 NRC Written Exam (SAO) 12 10: 13*1 NSRO 12 Points: 1.00 The plant was at rated power when an event occurred.

Current plant conditions are as follows:

  • 4160 BUS 1B indicates 0 AC AMPERES
  • RPV water level is 40" and lowering
  • Feedwater is being injected into the RPV at 2.5 x 106 lb/hr lAW Support Procedure 8, Lineup for Condensate Injection
  • Core Spray System 1 is lined up for injection lAW Support Procedure 9, Lineup for Core Spray System Injection
  • RPV pressure is 400 psig and lowering slowly
  • Drywall pressure is 17.9 psig and rising
  • The leak rate into the primary containment has been quantified at 2.9 x 106 lb/hr For these conditions, which of the following states the correct RPV water level control strategy?

A. Lower RPV pressure as necessary to allow low pressure systems to inject into the RPV.

B. Manually raise feedwater flow to> 2.9 x 106 lb/hr lAW Support Procedure 8, Lineup for Condensate Injection.

C. Line-up and commence injection with Core Spray 2 lAW Support Procedure 9, Lineup for Core Spray System Injection.

D. Wait until RPV water level lowers to the top of active fuel, and then direct an ED to allow low pressure systems to inject into the RPV.

Answer: A Answer Explanation Knowledge and Ability Reference Information Importance Rating K&A RO SRO 259002 Reactor Water Level Control System A2.04 *Ability to (a) predict the impacts of the following on the REACTOR WATER LEVEL CONTROL SYSTEM and (b) 3.0 3.1 based on those predictions, use procedures to correct, control, or mitigate the consequences of those abnormal operation: RFP runout condition: Plant-Specific OCS OPS ILT 13-1 EXAM Page: 33 of 75 13 May 2014

EXAMINATION ANSWER KEY ILT 13w1 NRC Written Exam (SAO)

Level SRO I Tier I 2 I Group I 1 General RPV Control* no EOP User's Guide References ATWS A is Correct. The stem shows that a leak into the PC has occurred, and that FW pumps B and C are not available, due to the Bus 1B loss. RPV water level is 40" and lowering, and current RPV pressure is above all low pressure systems discharge head, and the reactor has scrammed. Because FW runout protection will cap flow through the one remaining FW pump at 2.67 x 106 lb/hr, FW flow cannot be raised to greater than the leak size (and other FW pumps are unavailable). Because OW pressure is> 2.9 psig, core spray has started and is running on minimum flow and NOT discharging into the RPV (which is at 400 psig). The EOP step should be to lower RPV pressure to allow low pressure systems (ie., core spray) to inject. This question also requires the SRO to choose the correct strategy between RPV Control - no ATWS level control and the Level Restoration contingency.

B is Incorrect. This distractor is plausible if the applicant does Explanation not recognize that current RPV pressure is above all low pressure systems discharge head, and the reactor has scrammed. Because FW runout protection will cap flow through the one remaining FW pump at 2.67 x 106 lb/hr, FW flow cannot be raised to greater than the leak size (and other FW pumps are unavailable).

C is Incorrect. This distractor is plausible if the applicant does not recognize that RPV pressure is greater than the discharge pressure of the Core Spray pumps. Injection will not be possible until RPV pressure lowers to < 310 psi g.

Dis Incorrect. Because core spray has started normally, when the SRO directs a lowered RPV pressure to allow core spray to inject, THEN the SRO will decide if this action can keep water level above 0".

Lesson 2621.845.0.0052, RPV Control* no ATWS Plan Learning EWA-3055, Given a copy of the EOP, describe each Objective/ step/statement, including the technical basis and how to verify or perform each step.

References ILT: None LORT: Open Provided Question Source (New, Modified, Bank) Bank OCS OPS ILT 13-1 EXAM Page: 34 of 75 13 May 2014

EXAMINATION ANSWER KEY ILT 13-1 NRC Written Exam (SRO)

Previous 2 NRC Exams (ILT Only) I Yes Memory or Comprehension Fundamental 3:SPK Cognitive or Analysis Knowledge Level NUREG 1021 Appendix 8: Solve a Problem using Knowledge and its meaning 55.41b 55.43b 5 10CFR55 Content Assessment of facility conditions and selection of appropriate procedures during normal, abnormal, and emergency situations.

Justification for LORT questions with KIA values< 3.0 T N/A Time to Complete: 1*2 minutes Point Value: 1 System 10 No.: 295002 PRA: No Safety Function(s): 2 [81 ILT Category(s) (LORT Only): N/A [81 LORT OCS OPS ILT 13-1 EXAM Page: 35 of 75 13 May 2014

EXAMINATION ANSWER KEY ILT 13-1 NRC Written Exam (SAO) 13 10: 13-1 NSRO 13 Points: 1.00 The plant was at rated power when an event occurred. Present plant conditions include the following:

  • Drywell temperature is 225 °F and rising slowly
  • Drywell pressure is 9 psig and rising slowly
  • Tor us water level is 190" and rising slowly
  • RPV water level is 11 0" and lowering slowly
  • RPV pressure is 500 psig and lowering slowly Which of the following actions is required NOW?

A. Exit the RPV Control - No ATWS EOP and enter the Steam Cooling EOP.

B. Emergency Depressurize the RPV and then inject into the RPV with Core Spray.

C. Initiate Drywell sprays lAW the pressure leg of the Primary Containment Control EOP.

D. Lower RPV pressure with the Turbine Bypass Valves and inject into the RPV with Core Spray.

Answer: B Answer Explanation Knowledge and Ability Reference Information Importance Rating K&A RO SRO 209001 Low Pressure Core Spray 2.4.47- Emergency Procedures I Plan: Ability to diagnose 4.2 4.2 and recognize trends in an accurate and timely manner utilizing the appropriate control room reference material.

Level SRO I Tier I 2 I Group I 1 General PCC EOP EMG-SP17 References OCS OPS ILT 13-1 EXAM Page: 36 of 75 13 May 2014

EXAMINATION ANSWER KEY ILT 13~1 NRC Written Exam (SRO)

B is Correct. The plant was at rated power when an event occurred resulting in the provided plant conditions. The conditions show that Torus water level and RPV pressure are above the Torus Load Limit. Under these conditions, ED is required. Also, since systems injecting from outside the Primary Containment are secured with adequate core cooling assured, then Core Spray should be used to maintain/control RPV water level in this case with the TLL exceeded, regardless of RPV pressure.

Explanation A is Incorrect but plausible. With no sources of injection lined up or available (i.e. Feedwater, CRD, and SLC unavailable), the correct action would be to enter Steam Cooling, but at 0 TAF.11 C is Incorrect but plausible. Drywell sprays can be initiated from the Drywell pressure leg when Drywell or Torus pressure exceeds 12 psig.

D is Incorrect but plausible. Lowering RPV pressure with a high Torus water level is an expected action. But in this area of the curve, lowering RPV pressure with TBVs will not result in reaching the good side of the TLL curve and ED is required.

Lesson 2621.845.0.0002, Primary Containment Control Plan Learning PCC-03000: Using Procedure EMG-3200.02, evaluate the Objective/ technical basis for each step in the procedure, and apply this evaluation to determine correct courses of action under emergency conditions.

References ILT: TLL Graph LORT: Open Provided Question Source (New, Modified, Bank) Modified Previous 2 NRC Exams (ILT Only) No Memory or Comprehension Fundamental 3:SPK Cognitive Knowledge or Analysis Level NUREG 1021 Appendix B: Solve a Problem using Knowledge and its meaning 55.41b 55.43b 5 10CFR55 Content Assessment of facility conditions and selection of appropriate procedures during normal, abnormal, and emergency situations.

OCS OPS ILT 13-1 EXAM Page: 37 of 75 13 May 2014

EXAMINATION ANSWER KEY ILT 13-1 NRC Written Exam (SAO)

Justification for LORT questions with KIA values< 3.0 I N/A Time to Complete: 1*2 minutes Point Value: 1 System ID No.: 209001 PRA: Yes Safety Function(s): 2&4 IZI ILT Category(s) (LORT Only): NIA IZI LORT OCS OPS ILT 13-1 EXAM Page: 38 of 75 13 May 2014

EXAMINATION ANSWER KEY ILT 13~1 NRC Written Exam (SAO) 14 ID: 13-1 NSRO 14 Points: 1.00 The plant was at rated power with the following abnormal lineup:

  • Emergency Service Water Pump 52A was tagged out of service as of 0400 this morning A grid disturbance occurred which resulted in the following annunciator at 0900:
  • LKOUT RELAY 86/S1 B TRIP lAW Tech Specs and procedure OP-OC-108-104-1001, Guidance for Limiting and Administrative Conditions for Operations, which of the following states the required action?

A. The reactor shall be placed in cold shutdown condition without delay.

B. The reactor shall be placed in cold shutdown within 7-days from the pump inoperability.

C. The reactor shall be placed in cold shutdown within 15-days from the pump inoperability.

D. The reactor shall be placed in cold shutdown within 7 days from receipt of the above annunciator.

Answer: A Answer Explanation Knowledge and Ability Reference Information Importance Rating K&A RO SRO 262001 AC Electrical Distribution 2.2.40- Equipment Control: Ability to apply Technical 3.4 4.7 Specifications for a system.

Level SRO I Tier I 2 I Group I 1 General UFSAR 7.3 TS 3.4.C TS 3.7.8 References OCS OPS ILT 13-1 EXAM Page: 39 of 75 13 May 2014

EXAMINATION ANSWER KEY ILT 13~1 NRC Written Exam (SRO)

A is Correct. The plant was at rated power with Emergency Service Water Pump 52A out of service (powered from 4160 Bus 1C, which can be supplied from Startup transformer S1A). A grid disturbance then results in the loss of Startup Transformer 51 B.

The UFSAR section 7.3 states that the Emergency Service Water System pumps are engineered safeguard loads.

TS 3.7.B provides the following: The reactor shall be PLACED IN the COLD SHUTDOWN CONDITION if the availability of power falls below that required by Specification A above, except that 1.

The reactor may remain in operation for a period not to exceed 7 days if a startup transformer is out of service. None of the engineered safety feature equipment fed by the remaining Explanation transformer may be out of service. If just the Startup Transformer were lost, a 7-day LCO would be correct. lAW OP-OC-108-104-1001, it states that the requirement is to place the plant in the cold shutdown condition without delay.

B & C are Incorrect but plausible. The TS for Containment Spray does allow a 7-day and a 15-day LCO for one loop and one pump out of service respectively. But since the times listed are incorrect for the given conditions.

D is Incorrect but plausible if the applicant confuses the LCO action time with one pump out of service and one Containment Spray Loop out of service.

OCS OPS ILT 13-1 EXAM Page: 40 of 75 13 May 2014

EXAMINATION ANSWER KEY ILT 13-1 NRC Written Exam (SAO)

Lesson 2621.828.0.0016, Electrical Distribution Plan Learning ACD-1 0451, Referencing plant Technical Specifications (*from Objective/ memory for Initial Candidates) and given a set of plant conditions, determine, as applicable, the:

a) Definitions*

b) Safety Limits and Bases*

c) Limiting Safety System Settings and Bases*

d) Limiting Conditions for Operation and Applicability e) LCO Action Requirements (SRO ONLY) f) Surveillance Requirements (SRO ONLY) g) Design Features, Containment, Auxiliary Equipment, Administrative Controls, and Appendix B Environmental Technical Specifications (SRO ONLY) h) Bases for Surveillance Requirements, Design Features, Containment, Auxiliary Equipment, Administrative Controls, and Appendix B Environmental Technical Specifications. (SRO ONLY)*

References ILT: None LORT: Open Provided Question Source (New, Modified, Bank) Bank Previous 2 NRC Exams (ILT Only) No Memory or Comprehension Fundamental 3:SPK Cognitive or Analysis Knowledge Level NUREG 1021 Appendix B: .S.olve a Problem using Knowledge and its meaning 55.41b 55.43b 2 10CFR55 Content Facility operating limitations in the technical specifications and their bases.

Justification for LORT questions with KIA values< 3.0 I NIA Time to Complete: 1*2 minutes Point Value: 1 System ID No.: 262001 PRA: No Safety Function(s): 6 ~ ILT Category(s) (LORT Only): N/A ~ LORT OCSOPS ILT 13-1 EXAM Page: 41 of 75 13 May 2014

EXAMINATION ANSWER KEY ILT 13~1 NRC Written Exam (SAO) 15 ID: 13-1 NSRO 15 Points: 1.00 The plant is in the cold shutdown condition with fuel shuffling in progress. An event then occurred that resulted in a loss of MCC 1A24 and MCC 1B24.

Based on these conditions, which of the following identifies (1) the plant equipment impacted by the loss of power and (2) the action required lAW Technical Specifications, if any?

A. (1) Refuel floor radiation monitors B9 and C5.

(2) Refueling activities are NOT impacted and may continue unrestricted.

B. (1) Refuel floor radiation monitors B9 and C5.

(2) Refueling activities must cease immediately.

C. (1) SGTS Exhaust Fans 1-8 and 1-9.

(2) Refueling activities are NOT impacted and may continue unrestricted.

D. (1) SGTS Exhaust Fans 1-8 and 1-9.

(2) Refueling activities must cease immediately.

Answer: D Answer Explanation Knowledge and Ability Reference Information Importance Rating K&A RO SAO 261000 SGTS A2.07 - Ability to (a) predict the impacts of the following on the STANDBY GAS TREATMENT SYSTEM ; and (b) based 2.7 2.8 on those predictions, use procedures to correct, control, or mitigate the consequences of those abnormal conditions or operations: A.C. electrical failure Level SAO I Tier I 2 I Group I 1 General 205 TS 3.5.8 TS 1.14 References 330 OCS OPS ILT 13-1 EXAM Page: 42 of 75 13 May 2014

EXAMINATION ANSWER KEY ILT 13-1 NRC Written Exam (SRO)

Dis Correct. The plant is shutdown with fuel shuffling in-progress. This activity requires Secondary Containment integrity, which includes the Standby Gas Treatment System operable.

SGTS Fan 1-8 is powered from MCC 1A24 and SGTS Fan 1-9 is powered from MCC 1B24. Refuel activities could continue for a limited time if only 1 fan were operable, but it is given that both MCCs 1A24 and 1B24 are de-energized, and are thus must be declared inoperable. Thus, there is no operable SGTS Fan. TS definition 1.14 for Secondary Containment integrity includes Explanation SGTS operable. Procedure 205 also requires Secondary Containment integrity. With no SGTS, there is no Secondary Containment integrity and core alterations must cease immediately.

All distractors are Incorrect but plausible since Refuel Floor radiation monitors are powered from vital AC power. In addition, since the stem states the plant is in the cold shutdown condition, the applicant may not recall that at least one SGTS train must be still be operable.

Lesson 2621.812.0.0003, Refueling Plan Learning RFL-10451, Referencing plant Technical Specifications(* from Objective/ memory for Initial Candidates) and given a set of plant conditions, determine, as applicable, the:

a) Definitions*

b) Safety Limits and Bases*

c) Limiting Safety System Settings and Bases*

d) Limiting Conditions for Operation and Applicability e) LCO Action Requirements (SRO ONLY) f) Surveillance Requirements (SRO ONLY) g) Design Features, Containment, Auxiliary Equipment, Administrative Controls, and Appendix B Environmental Technical Specifications (SRO ONLY) h) Bases for Surveillance Requirements, Design Features, Containment, Auxiliary Equipment, Administrative Controls, and Appendix B Environmental Technical Specifications. (SRO ONLY)*

References ILT* None LORT: Open Provided

  • Question Source (New, Modified, Bank) New Previous 2 NRC Exams (ILT Only) No OCS OPS ILT 13-1 EXAM Page: 43 of 75 13 May 2014

EXAMINATION ANSWER KEY ILT 13*1 NRC Written Exam (SRO)

Memory or Comprehension Fundamental 2:DR Cognitive or Analysis Knowledge Level NUREG 1021 Appendix B: Describing or recognizing Relationships 55.41b 55.43b 2 10CFR55 Content Facility operating limitations in the technical specifications and their bases.

Justification for LORT questions with KIA values< 3.0 I N/A Time to Complete: 1*2 minutes Point Value: 1 System ID No.: 261000 PRA: No Safety Function(s): 9 181 ILT Category(s) (LORT Only): N/A 0 LORT OCS OPS ILT 13-1 EXAM Page: 44 of 75 13 May 2014

EXAMINATION ANSWER KEY ILT 13~1 NRC Written Exam (SAO) 16 10: 13-1 NSRO 16 Points: 1.00 The plant is at power following an outage, with primary containment inerting still in-progress.

An event then occurred and the following annunciator (#1) came into alarm, followed by the other annunciators (#2) two minutes later:

(#1) TORUS/DRYWELL- DW PRESS HI/LO

(#2) DW PRESS- DW PRESS HI-HI I and DW PRESS HI-HI II Drywell pressure and temperature indications are rising very slowly.

Which of the following lists the (1) effect on the inerting process, if any, and (2) the required actions the SAO shall direct NEXT? (assume NO operator actions have been taken)

A. (1) Drywell inerting has automatically isolated (even though the Drywell Vent Purge Interlock Bypass Keylock Switch is in BYPASS per the inerting procedure)

(2) Confirm primary containment isolations in accordance with Support Procedure 1, Confirmation of Automatic Initiations and Isolations B. (1) Drywell inerting has automatically isolated (even though the Drywell Vent Purge Interlock Bypass Keylock Switch is in BYPASS per the inerting procedure)

(2) Vent the drywell in accordance with Support Procedure 33, Venting the Drywell for Primary Containment High Pressure C. (1) Drywell inerting has not isolated (because the Torus/Drywelllsolation Valve Bypass Permissive Keylock Switch is in the DRYWELL position per the inerting procedure)

(2) Vent the drywell in accordance with Support Procedure 33, Venting the Drywell for Primary Containment High Pressure D. (1) Drywell inerting has not isolated (because the Torus/Drywelllsolation Valve Bypass Permissive Keylock Switch is in the DRYWELL position per the inerting procedure)

(2) Isolate the primary containment vent and purge valves in accordance with Support Procedure 40, Isolation of the Primary Containment Vent and Purge Valves Answer: A OCS OPS ILT 13-1 EXAM Page: 45 of 75 13 May 2014

EXAMINATION ANSWER KEY ILT 13~1 NRC Written Exam (SRO)

Answer Explanation Knowledge and Ability Reference Information Importance Rating K&A RO SRO 223001 Primary CTMT and Aux.

A2.07 *Ability to (a) predict the impacts of the following on the PRIMARY CONTAINMENT SYSTEM AND AUXILIARIES; 4.2 4.3 and (b) based on those predictions, use procedures to correct, control, or mitigate the consequences of those abnormal conditions or operations: High drywell pressure Level SRO I Tier I 2 I Group I 2 General RPVC - no ATWS References A is Correct. The first listed alarm comes in at 1.4 psig OW pressure, and the second two come in at 2.9 psig. If the operators have time, they would have isolated the inerting process themselves - but the isolation signal came quickly after the initial high pressure alarm. At 2.9 psig drywell pressure, the primary containment will isolate. The Drywell Vent Purge Interlock Bypass Keylock Switch is in BYPASS allows inerting with the mode switch in RUN, and does not effect the OW isolation at 2.9 psig - the vent valves will automatically close. The Torus/Drywell Isolation Valve Bypass Permissive Keylock Switch is normally in the NORM position and is positioned to either DRYWELL or TORUS when venting of the drywell or torus is required, with a drywell isolation signal present. Answer A is in the pressure control leg of EMG-3200-02 (PC Control) and should be Explanation performed.

B is Incorrect but plausible. Part (1) is correct, but part (2) would not be performed until OW temperature nears 281 F or until PC pressure has exceeded 12 psig. As stated in the stem, OW pressure and temperature are rising very slowly, and drywell spray would not be the appropriate step at this time.

C & D are Incorrect but plausible since inerting will isolate, the Torus/Drywelllsolation Valve Bypass Permissive Keylock Switch is in the NORM position, and the actions are incorrect as well.

Venting the OW lAW SP-33 would not be performed until after ED.

SP-40 is performed in the combustible gas leg when there is detectable hydrogen in the PC.

OCS OPS ILT 13-1 EXAM Page: 46 of 75 13 May 2014

EXAMINATION ANSWER KEY ILT 13M1 NRC Written Exam (SAO)

Lesson 2621.828.0.0032, Primary Containment Plan Learning PCS-00394, Given auto isolation signals, list or identify cause(s),

Objective/ system response, and affected Primary Containment system components.

References ILT: None LORT: Open Provided Question Source (New, Modified, Bank) Bank Previous 2 NRC Exams (ILT Only) No Memory or Comprehension Fundamental 3:PEO Cognitive or Analysis Knowledge Level NUREG 1021 Appendix B: Predict an Event or .Outcome 55.41b 55.43b 5 10CFR55 Content Assessment of facility conditions and selection of appropriate procedures during normal, abnormal, and emergency situations.

Justification for LORT questions with KIA values< 3.0 I N/A Time to Complete: 1-2 minutes Point Value: 1 System ID No.: 223001 PRA: No Safety Function(s): 5 ~ ILT Category(s) (LORT Only): N/A ~ LORT OCS OPS ILT 13-1 EXAM Page: 47 of 75 13 May 2014

EXAMINATION ANSWER KEY ILT 13~1 NRC Written Exam (SAO) 17 10: 13*1 NSRO 17 Points: 1.00 A plant startup is in progress with reactor power at 8%. Control rod 26-31 was withdrawn from position 12 to 48 when the following occurred:

  • Position indication for control rod 26-31 indicated BLACK-BLACK Which of the following statements describes the procedurally required actions for this event?

NOTE:

B. lAW RAP-H5a, insert control rod 26-31 to position "00". If control rod 26-31 cannot be inserted to position "00", valve the rod out of service and continue with the startup.

C. lAW RAP-H5a, insert control rod 26-31 until a response on Nuclear Instruments is received, then contact Reator Engineering prior to proceeding with the startup.

D. lAW ABN-6, insert control rod 26-31 until a response on Nuclear Instruments is received, then continue with the startup. Reactor Engineering concurence is NOT required for this situation.

Answer: A Answer Explanation Knowledge and Ability Reference Information Importance Rating K&A RO SRO 201 003 Control Rod and Drive Mechanism 2.4.11 - Emergency Procedures I Plan: Knowledge of 4.0 4.2 abnormal condition procedures.

Level SRO I Tier I 2 I Group I 2 General RAP-H5a ABN-6 References OCS OPS ILT 13-1 EXAM Page: 48 of 75 13 May 2014

EXAMINATION ANSWER KEY ILT 13-1 NRC Written Exam (SAO)

A is Correct. lAW RAP-H5a, a rod overtravel alarm combined with a loss of control rod indication "black-black" indicates the rod is uncoupled. The RAP directs the operator to ABN-6. With reactor power <1 0%, immediate operator actions require the operator to drive the rod to the full in position until "00" is indicated.

Supplemental operator actions (SRO Only required knowledge},

require the operator to scram the affected rod from Panel 6XR if it cannot be inserted to position "00".

B is Incorrect but plausible since it is required to valve the rod Explanation out of service if it cannot be recoupled, but only after it has first been inserted to position "00".

C is Incorrect but plausible since it is the correct immediate and supplemental action for an uncoupled control rod but at

>1Oo/opower, not <1 0%.

Dis Incorrect but plausible if the applicant recalls actions for an uncoupled rod at >10% but does not recall the supplemental actions correctly. With only one control rod out of service the applicant may believe a startup could continue without RE guidance.

Lesson 2621.828.0.0011, Control Rod Drive and Hydraulics Plan Learning CRD-10460, Describe the CRDM design features and/or interlocks Objective/ which provide for the following:

a. Detection of an uncoupled control rod
b. Slowing the drive mechanism near the end of travel following a scram
c. The use of either the accumulator or reactor water to scram the control rod.
d. Maintaining the control rod at a given location.

References ILT: None LORT: Open Provided Question Source (New, Modified, Bank) New Previous 2 NRC Exams (ILT Only) No Memory or Comprehension Fundamental 1:P Cognitive Knowledge or Analysis Level NUREG 1021 Appendix B: Procedure steps and cautions OCS OPS ILT 13-1 EXAM Page: 49 of 75 13 May 2014

EXAMINATION ANSWER KEY ILT 13-1 NRC Written Exam (SAO) 10CFR55 55.41b I I 55.43b I 5 Content Assessment of facility conditions and selection of appropriate procedures during normal, abnormal, and emergency situations.

Justification for LORT questions with KIA values< 3.0 I N/A Time to Complete: 1-2 minutes Point Value: 1 System ID No.: 201003 PRA: No Safety Function(s): 1 ~ ILT Category(s) (LORT Only): NIA ~ LORT OCS OPS ILT 13-1 EXAM Page: 50 of 75 13 May 2014

EXAMINATION ANSWER KEY ILT 13-1 NRC Written Exam (SRO) 18 10: 13*1 NSRO 18 Points: 1.00 The plant is at rated power. Plant conditions include the following:

  • Drywell pressure indicates 1.2 psig at steady
  • Drywell unidentified leak rate indicates 0. 75 gpm and steady Three hours later plant conditions include the following:
  • Drywell pressure indicates 1.5 psig and steady
  • Drywell unidentified leakrate indicates 3.0 gpm and steady Which of the following is required per Technical Specifications?

A. Be in at least the shutdown condition within the next 12 hours1.388889e-4 days <br />0.00333 hours <br />1.984127e-5 weeks <br />4.566e-6 months <br /> and in the cold shutdown condition within the next 24 hours2.777778e-4 days <br />0.00667 hours <br />3.968254e-5 weeks <br />9.132e-6 months <br />.

B. Identify the source of leakage within 4 hours4.62963e-5 days <br />0.00111 hours <br />6.613757e-6 weeks <br />1.522e-6 months <br /> or be in the shutdown condition within the next 12 hours1.388889e-4 days <br />0.00333 hours <br />1.984127e-5 weeks <br />4.566e-6 months <br /> and be in the cold shutdown condition within the following 24 hours2.777778e-4 days <br />0.00667 hours <br />3.968254e-5 weeks <br />9.132e-6 months <br />.

C. Reduce the reactor coolant system leakrate within limits within 4 hours4.62963e-5 days <br />0.00111 hours <br />6.613757e-6 weeks <br />1.522e-6 months <br /> or be in at least the shutdown condition within the next 12 hours1.388889e-4 days <br />0.00333 hours <br />1.984127e-5 weeks <br />4.566e-6 months <br /> and the cold shutdown condition within the following 24 hours2.777778e-4 days <br />0.00667 hours <br />3.968254e-5 weeks <br />9.132e-6 months <br />.

D. Reduce the reactor coolant system leakrate within limits within 8 hours9.259259e-5 days <br />0.00222 hours <br />1.322751e-5 weeks <br />3.044e-6 months <br /> or be in the shutdown condition within the next 12 hours1.388889e-4 days <br />0.00333 hours <br />1.984127e-5 weeks <br />4.566e-6 months <br /> and the cold shutdown condition within the following 24 hours2.777778e-4 days <br />0.00667 hours <br />3.968254e-5 weeks <br />9.132e-6 months <br />.

Answer: B Answer Explanation Knowledge and Ability Reference Information Importance Rating K&A RO SRO 202001 Recirculation System A2.02 - Ability to (a) predict the impacts of the following on the RECIRCULATION SYSTEM; and (b) based on those 3.7 3.9 predictions, use procedures to correct, control, or mitigate the consequences of those abnormal conditions or operations: Recirculation system leak Level SRO I Tier I 1 I Group I 1 General TS 3.3.0 References OCS OPS ILT 13-1 EXAM Page: 51 of 75 13 May 2014

EXAMINATION ANSWER KEY ILT 13~1 NRC Written Exam (SRO) 8 is Correct. lAW TS 3.3.0.3, with any reactor coolant leakage greater than the limit in 3.3.0.1.c (2gpm increase in unidentified leakage rate within any 24hr period while operating at steady state power), identify the source of leakage within 4 hours4.62963e-5 days <br />0.00111 hours <br />6.613757e-6 weeks <br />1.522e-6 months <br /> or be in the shutdown condition within the next 12 hours1.388889e-4 days <br />0.00333 hours <br />1.984127e-5 weeks <br />4.566e-6 months <br /> and be in the cold shutdown condition within the following 24 hours2.777778e-4 days <br />0.00667 hours <br />3.968254e-5 weeks <br />9.132e-6 months <br />.

All distractors are Incorrect but plausible since they apply to Explanation different leak rates. Oistractor A only applies if the leakage is not identified. Oistractor C is plausible since reducing the leakage is correct for other leak rates. Oistractor 0 is plausible since it applies to other leak rates. Other leak rates defined in TS 3.3.0.1 are 5gpm unidentified leakage and 25 gpm total (unidentified and identified).

NOTE: Question meets part (b) of KIA.

Lesson 2621.850.0.0090, Overview/Highlights of Technical Specifications Plan Learning TSX-01920, Given various plant indications (and their values) or Objective/ copies of control room/plant logs, evaluate the indications to determine plant status with respect to operating license and technical specifications.

References ILT: None LORT: Open Provided Question Source (New, Modified, Bank) New Previous 2 NRC Exams (ILT Only) No Memory or Comprehension Fundamental 3:SPK Cognitive or Analysis Knowledge Level NUREG 1021 Appendix 8: Solve a Problem using Knowledge and its meaning 55.41b 55.43b 2 10CFR55 Content Facility operating limitations in the technical specifications and their bases.

Justification for LORT questions with KIA values< 3.0 I N/A Time to Complete: 1-2 minutes Point Value: 1 System 10 No.: 202001 PRA: No Safety Function(s): 1 181 ILT OCS OPS ILT 13-1 EXAM Page: 52 of 75 13 May 2014

EXAMINATION ANSWER KEY ILT 13-1 NRC Written Exam (SAO) l Category(s) (LORT Only): I N/A I~ LORT OCS OPS ILT 13-1 EXAM Page: 53 of 75 13 May 2014

EXAMINATION ANSWER KEY ILT 13~1 NRC Written Exam (SAO) 19 10: 13*1 NSRO 19 Points: 1.00 The plant is in a refuel outage and fuel movements are in-progress on the refuel floor.

The Refuel Platform Operator had just latched onto a spent fuel bundle in the fuel pool and the mast was being raised to the NORMAL UP position. The intended core location for the bundle is in the 4th core quadrant. The following report was then received from the Control Room Operator:

  • The SAM ALL IN light is extinguished Which of the following is correct?

NOTE:

  • 205, Reactor Refueling 2  ;

.** 1 North ... .' .. * * **** * *I*..* **** * * **** * ...

3 ..* 4 A. lAW 205, ALL core alterations shall be halted.

B. lAW 205, core alterations shall be halted in the 4th quadrant ONLY.

C. lAW TS 3.2, core alterations may continue ONLY if SAM 24 indicates>

1 CPS.

D. lAW TS 3.2, core alterations may continue with NO further confirmations/verifications.

Answer: A OCS OPS ILT 13-1 EXAM Page: 54 of 75 13 May 2014

EXAMINATION ANSWER KEY ILT 13-1 NRC Written Exam (SAO)

Answer Explanation Knowledge and Ability Reference Information Importance Rating K&A RO SRO Conduct of Operations 4.6 4.6 2.1.20 - Abilit 1 to interpret and execute procedure steps.

Level SRO I Tier I 3 I Group J N/A General 205.0 TS 3.9 TS 1.21 References OCS OPS ILT 13-1 EXAM Page: 55 of 75 13 May 2014

EXAMINATION ANSWER KEY ILT 13~1 NRC Written Exam (SRO)

The plant is shutdown with core alterations in progress. A spent fuel bundle is currently being moved within the spent fuel pool, to be inserted into the core area. The control room operator reports to the refuel operator that the SRM all in light is extinguished.

Under normal refueling conditions, this light is illuminated and shows that all SRMs are fully inserted. With the light extinguished, one or more SRMs are no longer fully inserted, and thus cannot be considered operable.

Procedure 205 (P&L) requires that a minimum of 2 SRMs are fully inserted, operable and reading at least 1 CPS. The procedure also requires one operable SRM in the quadrant where the core alteration is being performed and one shall be located in an adjacent quadrant.

Since not even the control room operator is able to tell from the panel indications which SRM is not fully inserted, the refuel SRO cannot assume which SRM(s) is/are affected either, and must therefore halt all core alterations - not just alterations in the 4th core quadrant.

TS 3.9 mirrors the same requirement for required SRMs during Explanation core alterations as in the procedure. TS 3.9.G goes on to state that if the TS requirements are not met, to cease all core alterations.

TS definition 1.21 defines core alteration, and it only applies to the core area. Loss of SRM operability has no impact on fuel movements in the spent fuel pool. Therefore, all core alterations must cease, but movements in the spent fuel pool may continue.

Answer A is Correct.

If the applicant believes the given malfunction only impacts the fourth quadrant, then answer B would appear correct, but it is not.

It is true that an operable SRM must be reading at least 1 CPS lAW procedure 205. But operable SRMs must also be fully inserted. Answer C is Incorrect.

If the Applicant thinks that the SRM ALL IN light is normally extinguished when all SRMs are fully inserted, then answer D would appear correct. Since the light is ON to show that all SRMs are fully inserted, answer D is Incorrect.

OCS OPS ILT 13-1 EXAM Page: 56 of 75 13 May 2014

EXAMINATION ANSWER KEY ILT 13~1 NRC Written Exam (SAO)

Lesson 2621.812.0.0003, Refueling Plan Learning 00323, State the responsibilities of the following personnel Objective/ during refueling operations lAW procedure 205.0: Fuel Handling Director.

01132, Using plant references, state and interpret the Technical Specification reQuirements associated with fuel handling.

References ILT: None LORT: Open Provided Question Source (New, Modified, Bank) Bank Previous 2 NRC Exams (ILT Only) No Memory or Comprehension Fundamental 2:DR Cognitive or Analysis Knowledge Level NUREG 1021 Appendix B: Describing or recognizing Relationships 55.41b 55.43b 7 10CFR55 Content Fuel handling facilities and procedures.

Justification for LORT questions with KIA values< 3.0 I N/A Time to Complete: 1-2 minutes Point Value: 1 System ID No.: NIA PRA: No Safety Function(s): N/A IZI ILT Category(s) (LORT Only): NIA IZI LORT OCS OPS ILT 13-1 EXAM Page: 57 of 75 13 May 2014

EXAMINATION ANSWER KEY ILT 13-1 NRC Written Exam (SAO) 20 10: 13-1 NSRO 20 Points: 1.00 The plant is at rated power with CAD Pump B tagged out of service to replace the pump oil.

The work order requires running surveillance test 617.4.001, CAD Pump Operability Test as a Post Maintenance Test (PMT) following work completion.

lAW MA-AA-716-0 12, Post Maintenance Testing, which of the following states an additional requirement for the PMT of this pump, if any?

A. NO other actions outside of the surveillance are required.

B. Motor current should also be monitored and documented.

C. Bearing temperatures should also be monitored and documented.

D. A VT-2 leakage inspection should be performed and documented.

Answer: C Answer Explanation Knowledge and Ability Reference Information Importance Rating K&A RO SAO Equipment Control 2.2.21 - Knowledge of pre- and post-maintenance 2.9 4.1 operability requirements.

Level SAO I Tier I 3 I Group I N/A General MA-AA-716-012 617.4.001 References Answer Cis Correct. lAW MA-AA-716-012, Post Maintenance Testing, Attachment 1, Generic Post Maintenance Test Matrix, there are 3 types of tests for pump lubricant changeout: bearing temperature, external leakage, and lubrication level checks. The surveillance test does not test or verify any of these recommended actions. Of those actions listed, only answer C Explanation specifies a one of the listed actions.

All distractors are Incorrect but plausible if the applicant does not correctly interpret the requirements within the surveillance since all could be actions required for pump oil replacement, however only answer Cis listed in the provided reference.

OCSOPSILT13-1 EXAM Page: 58 of 75 13 May 2014

EXAMINATION ANSWER KEY ILT 13-1 NRC Written Exam (SRO)

Lesson 2621.828.0.0011, CRD and Hydraulic Plan Learning CRD-00021 - Identify and interpret the test and surveillance Objective/ procedures for the CRD System, including personnel and equipment allocation.

SRO-Only Task: 3410102419: PROMOTE THE USE OF POST-MODIFICATION AND POST* MAINTENANCE TESTS TO DETERMINE EQUIPMENT OPERABILITY.

References ILT: MA-AA-716*012 Provided Attachment 1; 617.4.001 LORT: Open Attachments 1-3 Question Source (New, Modified, Bank) Bank Previous 2 NRC Exams (ILT Only) No Memory or Comprehension Fundamental 1:P Cognitive or Analysis Knowledge Level NUREG 1021 Appendix B: Procedure steps and cautions 55.41b 55.43b None 10CFR55 This question is not linked to 1O.CFR.55.43(b), however the Content knowledge of this KIA is "unique to the SRO position". SRO-Only Task 3410102419.

Justification for LORT questions with KIA values< 3.0 I NIA Time to Complete: 1*2 minutes Point Value: 1 System ID No.: NIA PRA: No Safety Function(s): NIA ~ ILT Category(s) (LORT Only): NIA 0 LORT OCS OPS ILT 13-1 EXAM Page: 59 of 75 13 May 2014

EXAMINATION ANSWER KEY ILT 13~1 NRC Written Exam (SAO) 21 10: 13-1 NSRO 21 Points: 1.00 The plant was at rated power when a scram and an RPV isolation occurred. Present plant conditions are as follows:

  • Isolation Condenser A was being used for cooldown
  • Both Isolation Condensers are currently in Standby
  • ISOL CONDENSER A LEVEL indicates 7.2' and steady
  • ISOL CONDENSER B LEVEL indicates 7.4' and steady
  • ISOL COND A SHELL indicates 208 °F and lowering
  • ISOL COND B SHELL indicates 89 oF and steady The following annunciators then alarmed:
  • ISOL COND- COND AREA TEMP HI
  • RADIATION MONITORS- AREA MON HI (ARM C3, ISOLATION COND AREA indicates 9 mr/hr)
  • RB L.\P LO The Operator reports NO CHANGE in the Isolation Condenser shell level indications.

Which of the following states the potential impact and the required SAO direction?

Impact SRO Direction A. Increase in dose to workers Isolate BOTH ICs lAW Secondary in the RB Containment Control EOP B. Increase in dose to workers Isolate IC-A ONLY lAW Secondary in the RB Containment Control EOP C. Increase in offsite Isolate BOTH ICs lAW Radioactivity radioactivity release Release Control EOP D. Increase in offsite Isolate IC-A ONLY Radioactivity radioactivity release Release Control EOP Answer: A Answer Explanation Knowledge and Ability Reference Information Importance Rating K&A RO I SRO OCS OPS ILT 13-1 EXAM Page: 60 of 75 13 May 2014

EXAMINATION ANSWER KEY ILT 13~1 NRC Written Exam (SRO)

Radiation Control 2.3.14 *Knowledge of radiation or contamination hazards 3.4 3.8 that may arise during normal, abnormal, or emergency conditions or activities.

Level SRO I Tier I 3 I Group I NIA General References sec EOP A is Correct and B is Incorrect. The plant was at rated power when a scram and RPV isolation occurred. With the MSIVs closed, Isolation Condenser is being used for cooldown (place in service, then remove, then place in service as required). The initial conditions given show that IC A was in service and is now back in standby.

The provided annunciators, combined with no changes to the initial trends, show a steam leak into the RB in the vicinity of the Isolation Condensers in the RB: high area temperature (at max normal temperature of 160 oF), high radiation in the vicinity of the isolation condensers, and low RB AP. Entry into the Secondary Containment Control EOP is required. With a steam leak in the RB, dose to workers in the RB may rise. JAW the EOP, the leak Explanation should be isolated. Because the indications do not point to one condenser or the other as the leak source, both condensers should be isolated.

C and D are Incorrect. The given indications are not indicative of a tube leak in an isolation condenser (shell water level rising, shell water temperature rising). The ARM, by itself, could be indicative of a tube leak. These indications (rising shell water level, rising shell water temperature, ARM) would require entry into the Radiological release EOP, which in this case is not required. A tube leak in a condenser could lead to an increase in offsite release. Although the actions are correct, the procedure guidance is not correct.

Lesson 2621.845.0.0011, Secondary Containment Control Plan SCC-3082, Using the Secondary Containment Control EOP, evaluate the technical basis for each step and apply this Learning evaluation to determine the correct course of action under Objective/ emergency conditions.

References ILT: None LORT: Open Provided Question Source (New, Modified, Bank) Bank Previous 2 NRC Exams (ILT Only) No OCS OPS ILT 13-1 EXAM Page: 61 of 75 13 May 2014

EXAMINATION ANSWER KEY ILT 13~1 NRC Written Exam (SRO)

Memory or Comprehension Fundamental 3:SPK Cognitive or Analysis Knowledge Level NUREG 1021 Appendix 8: Solve a Problem using Knowledge and its meaning 55.41b 55.43b 4 10CFR55 Radiation hazards that may arise during normal and abnormal Content situations, including maintenance activities and various contamination conditions.

Justification for LORT questions with KIA values< 3.0 I N/A Time to Complete: 1*2 minutes Point Value: 1 System ID No.: N/A PRA: No Safety Function(s): N/A 181 ILT Category(s) (LORT Only): N/A 181 LORT OCS OPS ILT 13-1 EXAM Page: 62 of 75 13 May 2014

EXAMINATION ANSWER KEY ILT 13-1 NRC Written Exam (SRO) 22 10: 13-1 NSRO 22 Points: 1.00 The plant is shutdown for a refuel outage, and fuel shuffling is underway. All SRMs are currently reading between 300-400 cps.

A new fuel bundle, which was just removed from the spent fuel pool, is being lowered into its core location next to SRM 21, when the following annunciators and indications alarmed (assume indication are observed for an extended period of time):

  • SRM PERIOD SHORT
  • Several amber PERIOD lights are energized
  • SRM 21 indicates full scale Complete the following:
1) The Refuel SRO will direct that the fuel bundle be removed and ....
2) The SM will/will not declare an emergency event.
1) Direct that the fuel bundle be .2) SM will/will not declare an removed and ... emergency event A. placed back into the fuel pool. Will B. placed into an empty core Will NOT periphery location.

C. placed into any empty core Will location.

D. placed back into the fuel pool. Will NOT Answer: A Answer Explanation Knowledge and Ability Reference Information Importance Rating K&A RO SRO Emergency Procedures/Plan 2.4.45 - Ability to prioritize and interpret the significance 4.1 4.3 of each annunciator or alarm.

Level I SRO I Tier I 3 I Group I N/A OCS OPS ILT 13-1 EXAM Page: 63 of 75 13 May 2014

EXAMINATION ANSWER KEY ILT 13~1 NRC Written Exam (SAO)

General ABN-7 EP-AA-1010 References ABN-7, Unexplained Reactivity Change, requires that if SRM count rates raise by 3 doublings (or 8 times) the beginning-of-shift SRM count rate while moving fuel, to remove the fuel assembly being inserted and to re-position back to its previous location. With initial count rate at 300-400 cps, and the annunciators for hi count rate (1 x1 OE5 cps) and hi-hi (5x1 OE5 cps), the SRM count rate has increased more than 8 times. The bundle should be removed from the core and placed back in its previous position -the fuel pool.

Explanation lAW EP-AA-1010, a UE (Inadvertent Criticality MU2) should be declared: an unplanned sustained positive period observed on nuclear instrumentation. With the indications provided, it is clear that the period response is more than merely due to subcritical multiplication (EAL bases for MU2). Answer A is correct.

All distractors are Incorrect but plausible if the applicant does not recall the correct action or does not classify the EAL correctly.

Lesson 2624.812.0.0003, Reactor Refueling Plan Learning RFL-01734, State how to recognize symptoms of inadvertent Objective/ criticality.

References ILT: EAL Cold Matrix LORT: Open Provided Question Source (New, Modified, Bank) Bank Previous 2 NRC Exams (ILT Only) No Memory or Comprehension Fundamental 3:SPR Cognitive or Analysis Knowledge Level NUREG 1021 Appendix B: Solve a Problem using References 55.41b 55.43b 5 10CFR55 Content Assessment of facility conditions and selection of appropriate procedures during normal, abnormal, and emergency situations.

Justification for LORT questions with KIA values< 3.0 N/A OCS OPS ILT 13*1 EXAM Page: 64 of 75 13 May 2014

EXAMINATION ANSWER KEY ILT 13~1 NRC Written Exam (SRO)

Time to Complete: 1*2 minutes Point Value: 1 System I D No.: N/A PRA: No Safety Function(s): N/A ~ ILT Category(s) (LORT Only): N/A 0 LORT OCS OPS ILT 13-1 EXAM Page: 65 of 75 13 May 2014

EXAMINATION ANSWER KEY ILT 13-1 NRC Written Exam (SRO) 23 10: 13-1 NSRO 23 Points: 1.00 The plant was at rated power when EMRV NR018C inadvertently opened. EMRV NR108C keylock was placed in DISABLE lAW ABN-40, Stuck Open EMRV.

Which of the following states whether EMRV NR108C is available to perform its RPV pressure protection function, ADS function, and what Tech Spec action is required?

Available for RPV Available for Action Required Pressure ADS?

Protection?

A. No Yes The reactor shall remain in operation not to exceed 3 days B. No No Reactor pressure must be reduced to <11 0 psig within 24 hours2.777778e-4 days <br />0.00667 hours <br />3.968254e-5 weeks <br />9.132e-6 months <br />

c. No No The reactor shall remain in operation not to exceed 3 days D. Yes No Reactor pressure must be reduced to <11 0 psig within 24 hours2.777778e-4 days <br />0.00667 hours <br />3.968254e-5 weeks <br />9.132e-6 months <br /> Answer: C Answer Explanation Knowledge and Ability Reference Information Importance Rating K&A RO SAO Equipment Control 2.2.37 - Ability to determine operability and I or availability 3.6 4.6 of safety related eQuipment.

Level SAO J Tier I 3 I Group I NIA General 729E182 sh1 RAP-B5g TS 3.4.8 References OCS OPS ILT 13-1 EXAM Page: 66 of 75 13 May 2014

EXAMINATION ANSWER KEY ILT 13~1 NRC Written Exam (SAO)

C is Correct. With EMRV NR1 08C Disabled, the valve is unavailable to perform either the RPV pressure protection function or ADS function. TS 3.4.B.2 allows the reactor to remain in operation for a period not to exceed 3 days provided the motor operated isolation condensated makeup valves in both Isolation Condensers are verified daily to be operable. The question does not state there are any abnormalities with the ICs and therefore these valves can be assumed to be operable.

A is Incorrect. With NR1 08A disabled, the ADS function is inhibited. This distractor is plausible if the student does not recall this fact.

Explanation B is Incorrect. The correct action requirement is the reactor may remain in operation for a period not to exceed 3 days. This distractor is plausible if the student does not interpret the TS action requirements correctly or believes that the ICs are not able to be verified operable as required by TS 3.4.B.2.

D is Incorrect. With NR1 08A disabled, the RPV overpressure protection function is inhibited. This distractor is plausible if the student does not recall this fact. The correct action requirement is the reactor may remain in operation for a period not to exceed 3 days. This distractor is plausible if the student does not interpret the TS action requirements correctly or believes that the ICs are not able to be verified operable as required by TS 3.4.B.2.

Lesson 2621.828.0.0005, Automatic Depressurization System Plan Learning ADS-375, Given ADS alarms and indications, evaluate them in Objective/ terms of limits and trends.

References ILT: None LORT: Open Provided r------~..........._.......................................................r-------------------------------------------~

Question Source (New, Modified, Bank) Bank Previous 2 NRC Exams (ILT Only) No Memory or Comprehension Fundamental 2:DR Cognitive Knowledge or Analysis Level NUREG 1021 Appendix B: Describing or recognizing Relationships OCS OPS ILT 13-1 EXAM Page: 67 of 75 13 May 2014

EXAMINATION ANSWER KEY ILT 13-1 NRC Written Exam (SRO) 10CFR55 55.41b I I 55.43b I 2 Content Facility operating limitations in the technical specifications and their bases.

Justification for LORT questions with KIA values< 3.0 I N/A Time to Complete: 1-2 minutes Point Value: 1 System I 0 No.: N/A PRA: No Safety Function(s): N/A ~

-ILT Category(s) (LORT Only): N/A ~ LORT OCS OPS ILT 13-1 EXAM Page: 68 of 75 13 May 2014

EXAMINATION ANSWER KEY ILT 13-1 NRC Written Exam (SRO) 24 10: 13*1 NSRO 24 Points: 1.00 The plant was at power when an event occurred resulting in an offsite radiological release.

lAW Emergency Procedures, which of the following lists the smallest release and the wind direction as indicated on the PPC, which would require the Shift Emergency Director to recommend evacuation of Point X on the map below?

The EP-AA-1010, Radiological Emergency Plan for Oyster Creek Station, Radiological Effluents thresholds is shown below. (Assume there are no offsite impediments to evacuation.)

Table R1 -Effluent Monitor Thresholds GE SAE Alert UE 4.0 E+01 4.0 E+OO 1.93 E+OO

~-tCi/cc ~-tCi/cc ~-tCi/cc Main HRM HRM HRM 7.92 E+03 Stack OR OR OR cps LRM RAG EMS 1.6 E-08 1.6 E-09 7.8 E-10 amps HRM amps HRM amps HRM Turbine 5.0 E-01 2.51 E+05 8.11 E+04 8.11 E+02 Bldg ~-tCi/cc cpm LRM cpm LRM cpm LRM RAG EMS HRM OCS OPS ILT 13-1 EXAM Page: 69 of 75 13 May 2014

EXAMINATION ANSWER KEY ILT 13* 1 NRC Written Exam (SAO) 5-Mile Radius from Oyster Creek Note that each radiological release listed below has been at the value indicated for > 15 minutes.

Release Wind Direction A. Main Stack RAGEMS 6.6 E+01 !!Ci/cc HAM B. Main Stack RAGEMS 4.3 E+OO !!Ci/cc HAM

c. TB RAGEMS 5.9 E-01 !!Ci/cc HAM D. TB RAGEMS 2.78 E+05 cpm LAM Answer: C OCSOPSILT13-1 EXAM Page: 70 of 75 13 May 2014

EXAMINATION ANSWER KEY ILT 13M1 NRC Written Exam (SRO)

Answer Explanation Knowledge and Ability Reference Information Importance Rating K&A RO SRO Radiation Control 2.3.15 - Knowledge of radiation monitoring systems, such 2.9 3.1 as fixed radiation monitors and alarms, portable survey instruments, personnel monitoring equipment, etc.

Level SRO I Tier 1 3 J Group I N/A General EP-AA-1010 EP-AA-112-1 00-F-01 EP-AA-111-F-10 References The plant was at rated power when an event occurred resulting in an offsite release. The question asks what the smallest release rate would require the Shift Emergency Director to recommend evacuation of Point X.

Evacuation and sheltering are components of Protective Action recommendations (PARs). Thus, the question asks when a PAR will be recommended. PARs are recommended at the General Emergency level.

Point X on the map is in the eastern section. For wind to blow in this section, it must come from a westerly direction. Indicated wind direction is .. from .. , not "to ... A wind from 270 degrees would require evacuation of sections ENE, E, & ESE. Point X is contained in the E section. Therefore, a GE must be declared and Explanation indicated wind is from the west or 270 degrees. For plant-based PARs, evacuation of a 2-mile radius and 5 miles downwind is required, and Point X is within 5 miles downwind. Answer Cis lists a release rate which would require a GE declaration and indicated wind is from the west at 270 degrees. Answer C is correct.

All distractors are plausible if the applicant confuses a GE/SAE or does not correctly interpret the wind direction.

Answer A is a GE, but the wind direction is incorrect. Answer A is incorrect.

Answer 8 is a SAE and the incorrect wind direction. Answer 8 is incorrect.

Answer D is a SAE and the correct wind direction. Answer D is incorrect.

OCS OPS ILT 13-1 EXAM Page: 71 of 75 13 May 2014

EXAMINATION ANSWER KEY ILT 13~1 NRC Written Exam (SRO)

Lesson G-1 01, Station Emergency Director Plan Learning G-101 Objective/

References ILT: None LORT: Open Provided Question Source (New, Modified, Bank) Bank Previous 2 NRC Exams (ILT Only) No Memory or Comprehension Fundamental 3:SPK Cognitive or Analysis Knowledge Level NUREG 1021 Appendix B: .Solve a Problem using Knowledge and its meaning 55.41b 55.43b 5 10CFR55 Content Assessment of facility conditions and selection of appropriate procedures during normal, abnormal, and emergency situations.

Justification for LORT questions with KIA values< 3.0 I NIA Time to Complete: 1-2 minutes Point Value: 1 System I D No.: N/A PRA: No Safety Function(s): N/A IZI ILT Category(s) (LORT Only): N/A IZI LORT OCS OPS ILT 13-1 EXAM Page: 72 of 75 13 May 2014

EXAMINATION ANSWER KEY ILT 13~1 NRC Written Exam (SAO) 25 10: 13-1 NSRO 25 Points: 1.00 Which ONE of the following is a violation of the facility staffing requirements of Technical Specifications?

A. A non-licensed ST A is alone in the control room while there is no fuel in the reactor vessel.

B. The only ST A-qualified person on~site goes home sick and is not replaced for 1 hour1.157407e-5 days <br />2.777778e-4 hours <br />1.653439e-6 weeks <br />3.805e-7 months <br /> during POWER OPERATION.

C. There is ONLY one (1) SAO~ licensed individual and two (2) RO-Iicensed individuals onsite and they are all in the control room while the reactor is in a SHUTDOWN CONDITION.

D. The Unit Supervisor is in the Reactor Building and the Reactor Operator is alone in the control room at the controls while the reactor is in COLD SHUTDOWN.

Answer: C Answer Explanation I Knowledge and Ability Reference Information Importance Rating K&A RO SRO Conduct of Operations 2.1.5 - Ability to use procedures related to shift staffing, 2.9 3.9 such as minimum crew complement, overtime limitations, etc.

Level SRO I Tier I 3 I Group I N/A General TS 6.2.2.c References OCS OPS ILT 13-1 EXAM Page: 73 of 75 13 May 2014

EXAMINATION ANSWER KEY ILT 13-1 NRC Written Exam (SAO)

C is Correct. This does not meet the requirements of TS 6.2.2.c, which states: "At all times when there is fuel in the vessel, except when the reactor is in COLD SHUTDOWN or REFUEL modes, two licensed senior reactor operators and two licensed reactor operators shall be on site, with at least one licensed senior reactor operator in the control room and one licensed reactor operator at the controls."

A is Incorrect but plausible. There are no requirements in TS 6.2.2 for control room staffing when there is no fuel in the reactor vessel.

Explanation B is Incorrect but plausible. Although one STA is required to meet the minimum staffing, this condition still meets the requirements of TS 6.2.2.a, which states: "Except for the Shift Manager, shift crew composition may be one less than the minimum requirements, for a period of time not to exceed two hours, in order to accommodate unexpected absence of on-duty shift crew members."

D is Incorrect but plausible. This meets the requirement of TS 6.2.2.b, which states: "At all times when there is fuel in the vessel, at least one licensed senior reactor operator shall be on site and one licensed reactor operator should be at the controls."

Lesson 2621.850.0.0090, Overview/Highlights of Technical Specifications Plan Learning TSX-1 0451, Referencing plant Technical Specifications (*from Objective/ memory for Initial Candidates) and given a set of plant conditions, determine, as applicable, the:

a) Definitions*

b) Safety Limits and Bases*

c) Limiting Safety System Settings and Bases*

d) Limiting Conditions for Operation and Applicability e) LCO Action Requirements (SRO ONLY) f) Surveillance Requirements (SRO ONLY) g) Design Features, Containment, Auxiliary Equipment, Administrative Controls, and Appendix B Environmental Technical Specifications (SRO ONLY) h) Bases for Surveillance Requirements, Design Features, Containment, Auxiliary Equipment, Administrative Controls, and Appendix B Environmental Technical Specifications. (SRO ONLY)*

OCS OPS ILT 13-1 EXAM Page: 74 of 75 13 May 2014

EXAMINATION ANSWER KEY ILT 13~1 NRC Written Exam (SRO)

References ILT: None LORT: Open Provided Question Source (New, Modified, Bank) Bank Previous 2 NRC Exams (ILT Only) No Memory or Comprehension Fundamental 1:F Cognitive or Analysis Knowledge Level NUREG 1021 Appendix B: Facts 55.41b 55.43b 2 10CFR55 Content Facility operating limitations in the technical specifications and their bases.

Justification for LORT questions with KIA values< 3.0 I N/A Time to Complete: 1*2 minutes Point Value: 1 System 10 No.: NIA PRA: No Safety Function(s): N/A ~ ILT Category(s) (LORT Only): NIA 0 LORT OCS OPS ILT 13-1 EXAM Page: 75 of 75 13 May 2014